HESI 201-799

Lakukan tugas rumah & ujian kamu dengan baik sekarang menggunakan Quizwiz!

A client is receiving an IV solution of nitroglycerin 100mg/500ml D5W at 10 mcg/ minute. The nurse should program the infusion pump to deliver how many ml/hour? ( Enter numeric value only)

3 ml/hour Rationale : 0.01 x 500 x 60 / 100 = 3

A male client with COPD smokes two packs of cigarettes per day and is admitted to the hospital for a respiratory infection. He complains that he has trouble controlling respiratory distress at home when using his rescue inhaler. Which comment from the client indicates to the nurse that he is not using his inhaler properly? a. "I have a hard time inhaling and holding my breath after I squeeze the inhaler, but I do my best" b. "I never use the inhaler unless I am feeling really short of breath" c. I always shake the inhaler several times before I start" d. "After I squeeze the inhaler and swallow, I always feel a slight wave of nausea, bit it goes away"

"After I squeeze the inhaler and swallow, I always feel a slight wave of nausea, bit it goes away" Rationale: It is vital for the nurse to emphasize to the client that the mist should be inhaled, not swallowed. This assessment should be done for all COPD clients, not just those who complain about their inhalers.

A 3-year-old boy with a congenital heart defect is brought to the clinic by his mother... During the assessment, the mother asks the nurse why her child is at the 5th percent...response is best for the nurse to provide? a. Does your child seem mentally slower than his peers also? b. "His smaller size is probably due to the heart disease" c. Haven't you been feeding him according to recommended daily allowances for children? d. You should not worry about the growth tables. They are only averages for children

"His smaller size is probably due to the heart disease" Rationale: Poor growth patterns are associated with heart disease.

While the school nurse is teaching a group of 14-year-olds, one of the participants remarks, "You are too young to be our teacher! You're not much older than we are!" How should the nurse respond?

"We need to stay focused on the topic."

An adolescent receives a prescription for an injection of s-matriptan succinate 4 mg subcutaneously for a migraine headache. Using a vial labeled, 6 mg/ 0.5 ml, how many ml should the nurse administer? (Enter the numerical value only. If rounding is required, round to the nearest hundredth.

0.33 mL Rationale: 4mg x 0.5 ml=2/6=0.33 ml

An infant is receiving penicillin G procaine 220,000 units IM. The drug is supplied as 600,000 units/ml. How many ml should the nurse administer? (Enter numeric value only. If rounding is required, round to the nearest tenth)

0.4 Rationale: Calsulate using the formula, desired dose (220,000 units) over dose on hand (600,000 units) x the volume of the available dose (1 ml). 220,000 / 600,000 x 1 ml = 0.36 = 0.4 ml

A school-age child who weighs 42 pounds receives a post-tonsillectomy prescription for promethazine (Phenergan) 0.5 mg/kg IM to prevent postoperative nausea. The medication is available in 25 mg/ml ampules. How many ml should the nurse administer? (Enter numeric value only. If rounding is required, round to the nearest tenth).

0.4 Rationale: Convert pounds to kg 42lbs = 19.09 kg Next calculate to prescribed dose, 0.5 mg x 1909 kg = 9.545 Then use the desired dose/ dose on hand x volume on hand (9.545/25x1ml =0.3818=0.4 ml) Or use ratio proportion (9.545 mg: x ml = 25 mg: 1ml 25x = 9.545 X= 0.3818 = 0.4)

A client with a serum sodium level of 125 meq/mL should benefit most from the administration of which intravenous solution? a. 0.9% sodium chloride solution (normal saline) b. 0.45% sodium chloride solution (half normal saline) c. 10% Dextrose in 0.45% sodium chloride d. 5% dextrose in 0.2% sodium chloride

0.9% sodium chloride solution (normal saline)

A client admitted to the telemetry unit is having unrelieved chest pain after receiving 3 sublingual nitroglycerin tablets and morphine 8 mg IV. The electrocardiogram reveals sinus bradycardia with ST elevation. In what order should the nurse implement the nursing actions? (Arrange first to last) Move the crash cart to the client room Call the rapid response team to assist Inform the family of the critical situation Notify the client's healthcare provider

1. Call the rapid response team to assist 2. Move the crash cart to the client room 3. Notify the client's healthcare provider 4. Inform the family of the critical situation

The nurse is collecting a sterile urine specimen using a straight catheter tray for culture.... (Arrange from first action to last). Use forceps and swaps to clean the urinary meatus Drape the client in a recumbent position for privacy Open the urinary catheterization tray Don sterile gloves using aseptic technique

1. Drape the client in a recumbent position for privacy 2. Open the urinary catheterization tray 3. Don sterile gloves using aseptic technique 4. Use forceps and swaps to clean the urinary meatus

Which actions should the nurse implement with auscultating anterior breath sounds? (Place the first action on top and last action on the bottom.)

1. Place stethoscope in suprasternal area to auscultate from bronchial sounds 2. Auscultate bronchovesicular sounds from side to side of the first and second intercostal spaces 3. Displace female breast tissue and apply stethoscope directly on chest wall to hear vesicular sounds 4. Document normal breath sounds and location of adventitious breath sounds

When washing soiled hands, the nurse first wets the hands and applies soap. The nurse should complete additional actions in which sequence? (Arrange from first action on top last action on bottom.)

1. Rub hands palm to palm. 2. Interlace the fingers, 3. Dry hands with paper towel. 4. Turn off the water faucet.

An IV antibiotic is prescribed for a client with a postoperative infection. The medication is to be administered in 4 divided doses. What schedule is best for administering this prescription

1000, 1600, 2200, 0400

Nurses working on a surgical unit are concerned about the physicians treatment of clients during invasive procedures, such as dressing changes and insertion of IV lines. Clients are often crying during the procedures, and the physician is usually unconcerned or annoyed by the client's response. To resolve this problem, what actions should the nurses take? (Arrange from the first action on the top of the list on the bottom) Talk to the physician as a group in a non-confrontational manner. File a formal complaint with the state medical board. Submit a written report to the director of nursing. Contact the hospital's chief of medical services. File a formal complaint with the state medical board. Document concerns and report them to the charge nurse.

1. Talk to the physician as a group in a non-confrontational manner. 2. Document concerns and report them to the charge nurse. 3. Submit a written report to the director of nursing. 4. Contact the hospital's chief of medical services. 5. File a formal complaint with the state medical board. Rational: nurses have both an ethical and legal responsibility to advocate for clients' physical and emotional safety. Talking with the physician in a non-confrontational manner is the first step in conflict resolution. If this is not effective, the organizational chain of ineffective, a formal complaint with the state medical board should be implemented.

The HCP prescribes methotrexate 7.5 mg PO weekly, in 3 divides doses for a child with rheumatoid arthritis whose body surface area (BSA) is 0.6 m2. The therapeutic dosage of methotrexate PO is 5 to 15 mg/m2/week. How many mg should the nurse administer in each of the three doses given weekly? (Enter the numeric value only. If round is required, round to the nearest tenth.)

1.5

The healthcare provider prescribes Morphine Sulfate Oral Solution 38 mg PO q4 hours for a client who is opioid-tolerant. The available 30 mL bottle is labeled, 100 mg/5 mL (20mg/mL), and is packaged with a calibrated oral syringe to provide to provide accurate dose measurements. How many mL should the nurse administer? (Enter the numerical value only. If rounding is required, round to the nearest tenth.)

1.9 Rationale: 38/20x1=1.9 m

A client is receiving an IV solution labeled Heparin Sodium 20,000 Units in 5% dextrose injection 500 ml at 25 ml/hour. How many units of heparin is the client receiving each hour?

1000 units/hour Rationale:20000/500=40x25=1000

The healthcare provider prescribes potassium chloride 25 mEq in 500 ml D_5W to infuse over 6 hours. The available 20 ml vial of potassium chloride is labeled, "10 mEq/5ml." how many ml of potassium chloride should the nurse add the IV fluid? (Enter numeric value only. If is rounding is required, round to the nearest tenth.

12.5 Rationale: Using the formula D / H X Q: 25 mEq / 10 mEq x 5ml ꞊12.5ml

A client currently receiving an infusion labeled Heparin Sodium 25,000 Units in 5% Dextrose Injection 500 mL at 14 mL/hour. A prescription is received to change the rate of the infusion to 900 units of Heparin per hour. The nurse should set the infusion pump to deliver how many mL/hour? (Enter numeric value only)

18 Rationale: 450000/25000=18

The nurse is conducting health assessments. Which assessment finding increases a 56 year-old woman's risk for developing osteoporosis? a. Body mass index of (BMI) of 31 b. 20 pack-year history of cigarette smoking c. Birth control pill usage until age 45 d. Diabetes mellitus in family history

20 pack-year history of cigarette smoking Rationale: Cigarette smoking (2 packs/day x 310 years = 20 packs-year) increases the risk of osteoporosis. BMI of 30 or greater falls in the category of obesity which increase weight bearing that is protective against osteoporosis. C contain estrogens and are also protective against development of osteoporosis. D is not related to the development of osteoporosis.

The nurse notes the client receiving heparin infusion labeled, Heparin Na 25,000 Units in 5% Dextrose injection 500 ml at 50ml/hr. What dose of Heparin is the client receiving per hour?

2500

The nurse mixes 250 mg of debutamine in 250 ml of D5W and plans to administer the solution at rate client weighing 110 pounds. The nurse should set the infusion pump to administer how many ml per hour only. If rounding is required, round the nearest whole number.)

45

Dopamine protocol is prescribed for a male client who weigh 198 pounds to maintain the mean arterial pressure (MAP) greater than 65 mmHg. His current MAP is 50 mmHg, so the nurse increases the infusion to 7 mcg/kg/minute. The infusion is labeled dextrose 5% in water (D5W) 500 ml with dopamine 400 mg. The nurse should program the infusion pump to deliver how many ml/hour?

47

A 154 pound client with diabetic ketoacidosis is receiving an IV of normal saline 100 ML with regular insulin 100 units. The healthcare provider prescribes a rate of 0.1 units/kg/hour. To deliver the correct dosage, the nurse should set the infusion pump to Infuse how many ml/hour? enter numeric value only

7 Rationale: Convert the client's weight to kg, 2.2 pound: 1 kg:: 154 pounds: x kg = 154/2.2 = 70kg. Calculate the client infusion rate, 0.1 x 70 kg = 7 units/hour. Using the formula, D/H x Q = 7 units/hour / 100 units x 100 ml = 7ml / hour

A client is receiving an IV of heparin sodium 25000 units in 5% dextrose injection 500 ml at 14 ml/hour...verify that the client is receiving the prescribed amount of heparin. How many units is the client receiving

700 Rationale: 25000/500x14=700

A female client reports that she drank a liter of a solution to cleanse her intestines... immediately. How many ml of fluid intake should the nurse document? Whole number

760 Rationale: 1L=1000ml Subtract the emesis, 1 cup (8 oz)=240ml 1000-240=760 ml

The nurse is preparing a heparin bolus dose of 80 units/kg for a client who weighs 220 pounds. Heparin sodium injection, USP is available in a 3o ml multidose vial with the concentration of 1,000 USP units/ml. how many ml of heparin should the nurse administer? (Enter numeric value only)

8 Calculate the client's weigh in kg: 220 pounds divides by 2.2 pounds/kg ꞊100 kg Calculate the client's dose, 80 units x 100 kg ꞊ 8,000 units Use the formula, D / H X Q ꞊ 8,000 units / 1,000 units x 1ml ꞊ 8

The charge nurse of the Intensive Care Unit is making assignments for the permanent staff and one RN who was floated from a medical unit. The client with which condition is the best to assign to the float nurse? a. Diabetic ketoacidosis and titrated IV insulin infusion b. Emphysema extubated 3 hours ago receiving heated mist c. Subdural hematoma with an intracranial monitoring device d. Acute coronary syndrome treated with vasopressors

A

The health care provider prescribes atenolol 50 mg daily for a client with angina pectoris...to the health care provider before administering this medication? a. Irregular pulse b. Tachycardia c. Chest pain d. Urinary frequency

A

A newly admitted client vomits into an emesis basin as seen in the picture. The nurse should consult with the healthcare provider before administering which of the client's prescribes medications? a. Clopidogrel (Plavix), an antiplatelet agent, given orally b. Methylprednisolone (solu-medrol), a corticosteroid, to be given IV c. Nitroglycerin (Nitro-Dur) an antianginal, to be given transdermally. d. Enoxaparin (lovenox), a low-molecular weight heparin to be given subcutaneous. e. Furosemide (Lasix), a loop diuretic, to be given intravenously.

A Rationale: Because of the emesis is coffee brown appearance, which is an indicator of bleeding in GI track, the nurse should consult the health care provider because increase the risk of bleeding.

The healthcare provider prescribes a low-fiber diet for a client with ulcerative colitis. Which food selection would indicate to the nurse the client understands they prescribed diet? a. Roasted turkey canned vegetables b. Baked potatoes with skin raw carrots c. Pancakes whole-grain cereal's d. Roast pork fresh strawberries Rationale: Foods allowed on a low-fiber diet includes roasted or baked turkey and canned vegetables the foods in the other options are not low in fiber

A Rationale: Foods allowed on a low-fiber diet includes roasted or baked turkey and canned vegetables the foods in the other options are not low in fiber

The home health nurse is preparing to make daily visits to a group of clients. Which client should the nurse visit first?

A client with congestive heart failure who reports a 3 pound weight gain in the last two days

A school-aged child was recently diagnosed with celiac disease. Which instruction should the nurse give the classroom teacher? a- The child should avoid eating homemade cookies and cupcakes during parties. b- No products containing any form of peanuts should be allowed in the classroom c- Report a runny nose or head cold to the nurse immediately for further revaluation. d- Avoiding direct contact sports and games will reduce the child's risk of bruising.

A Rationale: Celiac disease is an intolerance to products containing gluten. Wheat flour,

An adult male was diagnosed with stage IV lung cancer three weeks ago. His wife approaches the nurse and asks how she will know that her husband's death is imminent because their two adult children want to be there when he dies. What is the best response by the nurse? a. Explain that the client will start to lose consciousness and his body system will slow down b. Reassure the spouse that the healthcare provider will let her know when to call the children c. Offer to discuss the client's health status with each of the adult children d. Gather information regarding how long it will take for the children to arrive

A Rationale: Expected signs of approaching death include noticeable changes in the client's level of consciousness and a slowing down of body systems. The nurse should answer the spouse's questions about the signs of imminent death rather than offering reassurance that may or may not be true. Other options listed may be implemented but the nurse should first answer the spouse's question directly.

An adult client with severe depression was admitted to the psychiatric unit yesterday evening. Although the client ran one year ago, his spouse states that the client no longer runs, bur sits and watches television most of the day. Which is most important for the nurse to include in this client's plan of care for today? a. Assist client in identifying goals for the day. b. Encourage client to participate for one hour in a team sport. c. Schedule client for a group that focuses on self-esteem. d. Help client to develop a list of daily affirmations.

A Rationale: clients with severe depression have low energy and benefit from structured activities because concentration is decreased. The client participates in care by identifying goals for the day is the most important intervention for the client's first day at the unit. Other options can be implemented over time, as the depression decreases.

A client is admitted with acute pancreatitis. The client admits to drinking a pint of bourbon daily. The nurse medicates the client for pain and monitors vital signs q2 hours. Which finding should the nurse report immediately to the healthcare provider? a. Confusion and tremors b. Yellowing and itching of skin. c. Abdominal pain and vomiting d. Anorexia and abdominal distention

A Rationale: daily alcohol is the likely etiology for the client's pancreatitis. Abrupt cessation of alcohol can result in delirium tremens (DT) causing confusion and tremors, which can precipitate cardiovascular complications and should be reported immediately to avoid life-threatening complications. The other options are expected findings in those with liver dysfunction or pancreatitis, but do not require immediate action.

A young adult female college student visits the health clinic in early winter to obtain birth control pills. The clinic nurse asks if the student has received an influenza vaccination. The student stated she did not receive vaccination because she has asthma. How should the nurse respond? a. Offer to provide the influenza vaccination to the student while she is at the clinic b. Encourage the student to obtain a vaccination prior to the next influenza season. c. Confirm that a history of asthma can increase risks associated with the vaccine. d. Advise the student that the nasal spray vaccine reduces side effects for people with asthma.

A Rationale: person with asthma are at increased risk related to influenza and should receive the influenza vaccination prior to or during influenza season. Waiting until the start of the next season places the student at risk for the current season. The vaccination does not increase risk for persons with asthma, but the nasal spray may result in increased wheezing after receiving that form of the vaccination.

A client is being discharged home after being treated for heart failure (HF). What instruction should the nurse include in this client's discharge teaching plan? a. Weigh every morning b. Eat a high protein diet c. Perform range of motion exercises d. Limit fluid intake to 1,500 ml daily

A should be instructed to weight each morning before breakfast with approximately the same clothing. A is not specifically to HF and fluid retention.

The nurse is assigned to care for clients on a medical unit. Based on the notes taken during the shift report, which client situation warrants the nurse's immediate attention? a. A young adult with Crohn's disease who reports having diarrheal stools b. An older adult with type 2 diabetes whose breakfast tray arrives 20 minutes late. c. A 10-year-old who is receiving chemotherapy and the infusion pump is beeping. d. A teenager who reports continued pain 30 minutes after receiving an oral analgesic.

A 10-year-old who is receiving chemotherapy and the infusion pump is beeping Rationale: The nurse should immediately assess the child whose infusion pump is alarming during chemotherapy administration because infiltration of a caustic agent can cause tissue damage and children are at greater risk for fluid volume imbalance. Diarrhea is a common occurrence for Crohn's disease. Late consumption of food for a diabetic is of concern, but 20 minutes late is usually not life-threatening. Treatment of pain is most important but has been only 30 mints since the client was medicated and this issue can be assessed in 10 mints or delegated to another nurse.

The charge nurse is planning for the shift and has a registered nurse (RN) and a practical nurse (PN) on the team. Which client should the charge nurse assign to the RN? a. A 64-year-old client who had a total hip replacement the previous day. b. A 75-year-old client with renal calculi who requires urine straining. c. An adolescent with multiple contusions due to a fall that occurred 2 days ago. d. A 30-year-old depressed client who admits to suicide ideation.

A 30 year old depressed client who admits to suicide ideation RATIONALE: A client who is suicidal requires psychological assessment, therapeutic communication and knowledge beyond the educational level of a practical nurse (RN). Other clients could be cared for by the PN or the UAP, with supervision by the registered nurse.

A nurse working on an endocrine unit should see which client first? a. An adolescent male with diabetes who is arguing about his insulin dose. b. An older client with Addison's disease whose current blood sugar level is 62mg/dl (3.44 mmol/l). c. An adult with a blood sugar of 384mg/dl (21.31mmol/l) and urine output of 350 ml in the last hour. d. A client taking corticosteroids who has become disoriented in the last two hours.

A client taking corticosteroids who has become disoriented in the last two hours. Rational: meeting the client's need for safety is a priority intervention. Mania and psychosis can occur during corticosteroids therapy, places the client at risk for injury, so the patient taking corticosteroids should be seen first.

A client with multiple sclerosis is receiving beta-1b interferon every other day. To assess for possible bone marrow suppression caused by the medication, which serum laboratory test findings should the nurse monitor? (Select all that apply)

A. Platelet count B. Red blood cell count (RBC) C. White blood cell count (WBC).

When assessing a male client, the nurse notes that he has unequal lung expansion. What conclusion regarding this finding is most likely to be accurate? The client has: a- A collapsed lung b- A history of COPD c- A chronic lung infection d- Normally functioning lungs

A collapsed lung Rationale: Unilateral absence of chest movement (or unequal lung expansion because one lung is not moving at all) may be indicative previous surgical removal of that lung, a bronchial obstruction, or a collapsed lung caused by air or fluid in the pleural space.

The nurse suspect may be hemorrhaging internally. Which findings of an orthostatic test may indicate to the nurse of major bleed?

A decrease in the systolic b/p of 10mm/hg with a corresponding increase of heart rate of 20.

A client is admitted with an exacerbation of heart failure secondary to COPD. Which observations by the nurse require immediate intervention to reduce the likelihood of harm to this client? (Select all that apply).

A full pitcher of water is on the bedside table The client is lying in a supine position in bed

When organizing home visits for the day, which older client should the home health nurse plan to visit first? a. A woman who takes naproxen (Naprosyn) and reports a recent onset of dark, tarry stools. b. A man who receives weekly injections of epoetin (Procrit) for a low serum iron level c. A man with emphysema who smokes and is complaining of white patches in his mouth d. A frail woman with heart failure who reported a 2 pounds' weight gain in the last week.

A woman who takes naproxen (Naprosyn) and reports a recent onset of dark, tarry stools

Following an esophagogastroduodenoscopy (EGD) a male client is drowsy and difficult to arouse, and his respiration are slow and shallow. Which action should the nurse implement? Select all that apply. a. Prepare medication reversal agent b. Check oxygen saturation level c. Apply oxygen via nasal cannula d. Initiate bag- valve mask ventilation. e. Begin cardiopulmonary resuscitation

A,B,C Rationale: Sedation, given during the procedure may need to be reverse if the client does not easily wake up. Oxygen saturation level should be asses, and oxygen applied to support respiratory effort and oxygenation. The client is still breathing so the bag- valve mask ventilation and CPR are not necessary.

The nurse is caring for a group of clients with the help of a practical nurse (PN). Which nursing actions should the nurse assign to the PN? (Select all that apply.)

A. Administer a dose of insulin per sliding scale for a client with type 2 diabetes mellitus (DM). B. Obtain postoperative vital signs for a client one day following unilateral knee arthroplasty C. Perform daily surgical dressing change for a client who had an abdominal hysterectomy

The nurse requests a meals tray for a client follows Mormon beliefs and who is on clear liquid diet following abdominal surgery. Which meal item should the nurse request for this client? (Select all that apply) a- Apple juice b- Chicken broth. c- Hot chocolate d- Orange juice e- Black coffee

AB

The home care nurse provide self-care instruction for a client chronic venous insufficiency cause by deep vein thrombosis. Which instructions should the nurse include in the client's discharge teaching plan? Select all that apply a. Avoid prolonged standing or sitting b. Use recliner for long period of sitting c. continue wearing elastic stocking d. Maintain the bed flat while sleeping e. Cross legs at knee but not at ankle

ABC

When conducting diet teaching for a client who is on a postoperative soft diet, which foods should eat? (Select all that apply) a. Pasta, noodles, rice. b. Egg, tofu, ground meat. c. Mashed, potatoes, pudding, milk. d. Brussel sprouts, blackberries, seeds. e. Corn bran, whole wheat bread, whole grains.

ABC Rational: a client's postoperative diet is commonly progressed as tolerated. A soft diet includes foods that are mechanically soft in texture (pasta, egg, ground meat, potatoes, and pudding. High fiber foods that require thorough chewing and gas forming foods, such as cruciferous vegetables and fresh fruits with skin, grains and seeds are omitted.

A client is receiving oxytocin (Pitocin) to augment early labor. Which assessment is most important time the infusion rate is increases? a. Contraction pattern b. Blood pressure c. Infusion site d. Pain level

Contraction pattern

The husband of a client with advanced ovarian cancer wants his wife to have every treatment available. When the husband leaves, the client tells the nurse that she has had enough chemotherapy and wants to stop all treatments but knows her husband will sign the consent form for more treatment. The nurse's response should include which information? a- The husband cannot sign the consent for the client, her signature is required b- The client's specific wishes should be discussed with her healthcare provider c- Counseling should be sought to resolve the husband's desire to control his wife d- The healthcare team will formulate a plan of care to keep the client comfortable e- The client should seek a second medical opinion before deciding to stop treatment.

ABD Rationale: An adult client who is mentally competent has the autonomy and the client's right to make her own decision regarding her treatment.

The nurse is preparing a client who had a below-the-knee (BKA) amputation for discharge to home. Which recommendations should the nurse provide this client? (Select all that apply) a. Inspect skin for redness b. Use a residual limb shrinker c. Apply alcohol to the stump after bathing d. Wash the stump with soap and water e. Avoid range of motion exercises

ABD Rationale: Several actions are recommended for home care following an amputation. The skin should be inspected regularly for abnormalities such as redness, blistering, or abrasions. A residual limb shrinker should be applied over the stump to protect it and reduce edema. The stump should be washed daily with a mild soap and carefully rinse and dried. The client should avoid cleansing with alcohol because it can dry and crack the skin. Range of motion should be done daily.

What is the primary goal when planning nursing care for a client with degenerative joint disease (DJD)? a. Obtain adequate rest and sleep b. Achieve satisfactory pain control. c. Improve stress management skills d. Reduce risk for infection.

Achieve satisfactory pain control.

An older female who ambulate with a quad-cane prefer to use a wheel chair because she has a halting and unsteady gait at times. Which interventions should the nurse implement? (Select all that apply) a. Move personal items within client's reach b. Lower bed to the lower possible position c. Raise all bed rails when the client is resting d. Give directions to call for assistance e. Assist client to the bathroom in 2 hours. f. Encourage the use of the wheelchair

ABDE Rationale: A client who needs assistive devices, such as quad-cane is at risk for falls. Precautions that should implement include ensuring that personal items are within reach the bed is in the lowest position and directions are given to call assistance to minimize the risk for falls. Frequently assisting the client to the bathroom help ensure this client does not go the bathroom by herself, thereby decreasing the possibility of falling.

The nurse is preparing to discharge an older adult female client who is at risk for hyPOCALCEMIA nurse include with this client's discharge teaching? a- Report any muscle twitching or seizures b- Take vitamin D with calcium daily c- Avoid seafood, particularly selfish d- Low fat yogurt is a good source of calcium e- Keep a diet record to monitor calcium intake

ABDE Rationale: Twitching and seizure are signs of low calcium. (A) Vit D supplement with calcium to enhance calcium absorption, especially in older adults. Dairy product should be included in the diet. Keeping a food record is a good healthcare practice. Foods high in calcium are recommended to maintain normal calcium level and it is important to verify if the client has allergy to shellfish.

A client with acute pancreatitis is complaining of pain and nausea. Which interventions should the nurse implement (Select all that apply) a- Monitor heart, lung, and kidney function. b- Notify healthcare provider of serum amylase and lipase levels. c- Position client on abdomen to provide organ stability d- Encourage an increased intake of clear oral fluids e- Review client's abdominal ultrasound findings.

ABE

During a left femoral artery aortogram, the healthcare provider inserts an arterial sheath and initiate. Through the sheath to dissolve an occluded artery. Which interventions should the nurse implement? a- Instruct the client to keep the left leg straight b- Keep the head of bed at 60-degree angle. c- Observe the insertion site for a hematoma d- Manually flush the arterial sheath hourly e- Circle first noted drainage on the dressing

ACE

A client with type 2 diabetes mellitus is admitted for frequent hyperglycemic episodes and a glycosylated hemoglobin (HbA1c) of 10%. Insulin glargine 10 units subcutaneously once a day at bedtime and a sliding scale with insulin aspart q6h are prescribed. What action should the nurse include in this client's plan of care? a. Fingerstick glucose assessment q6h with meals b. Mix bedtime dose of insulin glargine with insulin aspart sliding scale dose c. Review with the client proper foot care and prevention of injury d. Do not contaminate the insulin aspart so that it is available for iv use e. Coordinate carbohydrate controlled meals at consistent times and intervals Teach subcutaneous injection technique, site rotation and insulin management

ACEF

An older adult resident of a long-term care facility has a 5-year history of hypertension. The client has a headache and rate the pain 5 on a pain scale 0 to 10. The client's blood pressure is currently 142/89. Which interventions should the nurse implement? (Select all that apply) a. Administer a daily dose of lisinopril as scheduled. b. Assess the client for postural hypotension. c. Notify the healthcare provider immediately d. Provide a PRN dose of acetaminophen for headache e. Withhold the next scheduled daily dose of warfarin.

AD Rational: the client' routinely scheduled medication, lisinopril, is an antihypertensive medication and should be administered as scheduled to maintain the client's blood pressure. A PRN dose of acetaminophen should be given for the client's headache. The other options are not indicated for this situation.

After teaching a male client with chronic kidney disease (CKD) about therapeutic diet... which menu of foods indicates that the teaching was effective? Select all that apply a- A slice of whole grain toast b- Half cup of black beans c- A ham and cheese sandwich d- A bowl of cream of wheat e- Two bananas.

AD Rationale: Patient with CKD have elevated serum potassium, sodium and protein levels. A and D are low in potassium, sodium and protein, Beans are rich in proteins. C are high in sodium and potassium and E are rich in potassium.

n adult man reports that he recently experienced an episode of chest pressure and breathlessness when he was jogging in the neighborhood. He expresses concern because both of his deceased parents had heart disease and his father was a diabetic. He lives with his male partner, is a vegetarian, and takes atenolol which maintain his blood pressure at 138/74. Which risk factors should the nurse explore further with the client? Select all that apply a- History of hypertension. b- Homosexual lifestyle c- Vegetarian diet d- Excessive aerobic exercise e- Family heath history.

AE Rationale: Based on the client's family history and medication for management of hypertension, the nurse should further explore these risk for ischemic heart disease.

A client with eczema is experiencing severe pruritus. Which PRN prescriptions should the nurse administer? (Select all that apply) a. Topical corticosteroid. b. Topical scabicide. c. Topical alcohol rub. d. Transdermal analgesic. e. Oral antihistamine

AE Rationale: anti-inflammatory actions of topical corticosteroids and oral antihistamines provide relief from severe pruritus (itching). Other options are not indicated.

A female client comes to the clinic complaining of fatigue and inability to sleep because she is the full-time caretaker for 22-year-old son who was paralyzed by a motor vehicle collision. She adds that her husband left her because he says he can't take her behavior any more since all she does is care for their son. What intervention should the nurse implement? a- Schedule a home visit in the afternoon to assess the son and client role as caregiver. b- Acknowledge the client's stress and suggest that she consider respite care. c- Provide feedback to the client about her atonement for guilt about her son's impairment. d- Teach the client to problem-solve for herself and establish her own priorities.

Acknowledge the client's stress and suggest that she consider respite care. Rationale: When this amount of disclosure is offered, the client is usually seeking information focuses on the client's expression of worry, concern and stress and addresses the client's need to initiate a request for assistance with respite care.

To reduce the risk of being named in malpractice lawsuit, which action is most important for the nurse to take?

Adhere consistently to standards of care.

After checking the fingerstick glucose at 1630, what action should the nurse implement? a. Notify the healthcare provider b. Administer 8 units of insulin aspart SubQ c. Gives an IV bolus of Dextrose 50% 50 ml d. Perform quality control on the glucometer.

Administer 8 units of insulin aspart SubQ

A primigravida client is 36 weeks gestation is admitted to labor and delivery unit because her membranes ruptured 30minutes ago. Initial assessment indicates 2cm dilation, 50% effaced, -2 station, vertex presentation greenish colored amniotic fluid, and contractions occurring 3-5 minutes with a low FHR after the last contraction peaks: a. Administer Oxygen via face mask b. Apply an internal fetal heart monitor c. Notify the healthcare provider d. Use a vibroacoustic stimulator

Administer Oxygen via face mask Rationale: The nurse should administer oxygen to increase the amount of oxygen available for the fetus, because is presenting characteristics of late decelerations, caused by uteroplacental insufficiency.

A male client is admitted with a severe asthma attack. For the last 3 hours he has experienced increased shortness of breath. His arterial blood gas results are: pH 7.22 PaCO2 55 mmHg; HCO3 25 mEq/L or mmol/L (SI). Which intervention should the nurse implement?

Administer PRN dose of albuterol

While making rounds, the charge nurse notices that a young adult client with asthma who was admitted yesterday is sitting on the side of the bed and leaning over the bed-side-table. The client is currently receiving at 2 litters/minute via nasal cannula. The client is wheezing and is using pursed-lip breathing. Which intervention should the nurse implement? a. Assist the client to lie back in bed b. Call for an Ambu resuscitating bag c. Increase oxygen to 6 litters/minute d. Administer a nebulizer Treatment

Administer a nebulizer Treatment Rationale: The client needs an immediate medicated nebulizer treatment. Sitting in an upright position with head and arms resting on the over-bed table is an ideal position to promote breathing because it promotes lung expansion. Other actions me be accurate but not yet indicated.

A male adult is admitted because of an acetaminophen overdose. After transfer to the mental health unit, the client is told he has liver damage. Which information is most important for the nurse to include in the client's discharge plan?

Avoid exposure to large crowds

A 6 -years-old who has asthma is demonstrating a prolonged expiratory phase and wheezing, and has 35% personal best peak expiratory flow rate (PEFR). Based on these finding, which action should the nurse implement first? a. Administer a prescribed bronchodilator. b. Report finding to the healthcare provider. c. Encourage the child to cough and deep breath d. Determine what trigger precipitated this attack.

Administer a prescribed bronchodilator. Rationale: If the PEFR is below 50% in as asthmatic child, there is severe narrowing of the airway, and a bronchodilator should be administered immediately. B should be implemented after A. C will not alleviate the symptoms and D is not a priority.

A client receives a new prescription for simvastatin (Zocor) 5 mg PO daily at bedtime. What action should the nurse take? a. Provide a bedtime snack to be eaten before taking the medication. b. Administer the medication as prescribed with a glass of water c. Contact the prescriber about changing the time of administration. d. Check the client's blood pressure prior to administering the med.

Administer the medication as prescribed with a glass of water Rationale: Simvastatin (Zocor), a HMG co-enzyme A reductase inhibitor, interferes with cholesterol synthesis pathway. Zocor can be taken at any time.

A toddler presents to the clinic with a barking cough, strider, refractions with respiration, the child's skin is pink with capillary refill of 2 seconds. Which intervention should the nurse implement? a. Encourage the child to cough b. Obtain a throat specimen for culture c. Administer nebulized epinephrine d. Collect blood for arterial blood gasses

Administered Nebulized Epinephrine

A client with a traumatic brain injury becomes progressively less responsive to stimuli. The client has a "Do Not Resuscitate" prescription, and the nurse observes that the unlicensed assistive personnel (UAP) has stopped turning the client from side to side as previously schedules. What action should the nurse take?

Advise the UAP to resume positioning the client on schedule

A female client with chronic urinary retention explains double voiding technique to the nurse by stating she voids partially, hold the remaining urine in her bladder for three minutes, then voids again to empty her bladder fully. How should the nurse respond? a. Affirm that the client is effectively performing the double voiding. b. Advise the client to empty her bladder fully when she first voids c. Suggest that the client drink water between the two voiding. d. Explain that Kegel exercises help promote full bladder empty.

Advise the client to empty her bladder fully when she first voids

The nurse provides feeding tube instructions to the wife of a client with end stage cancer. The client's wife performs a return demonstration correctly, but begins crying and tells the nurse, "I just don't think I can do this every day." The nurse should direct further teaching strategies toward which learning domain? a- Cognitive b- Affective c- Comprehension d- Psychomotor

Affective

The nurse makes a supervisory home visit to observe an unlicensed assistive personnel (UAP) who is providing personal care for a client with Alzheimer's disease. The nurse observes that whenever the client gets upset, the UAP changes the subject. What action should the nurse take in response to this observation? a. Tell the UAP to offer more choices during the personal care to prevent anxiety b. Meet with the UAP later to role model more assertive communication techniques c. Assume care of the client to ensure that effective communication is maintained. d. Affirm that the UAP is using and effective strategy to reduce the client's anxiety.

Affirm that the UAP is using and effective strategy to reduce the client's anxiety.

A client arrives in the emergency center with a blood alcohol level of 500 mg/dl. When transferred to the observation unit, the client becomes demanding, aggressive, and shouts at the staff. Which assessments finding is most important for the nurse to identify in the first 24 hours?

Agitation and threats to harms staff

A male Korean-American client looks away when asked by the nurse to describe his problem. What is the best initial nursing action? a- Ask social services to dins a Korean interpreter b- Establish direct eye contact with the client. c- Allow several minutes for the client to respond. d- Repeat the question slowly and distinctly.

Allow several minutes for the client to respond

The nurse observes a newly hired unlicensed assistive personnel (UAP) performing a fingestick to obtain a client's blood glucose. Prior to sticking the client's finger, the UAP explains the procedure and tell the client that it I painless. What action should the nurse take? a. Allow the UAP to complete the procedure, then discuss the painless comment privately with the UAP. b. Stop the UAP before the procedure and explain to the client that some discomfort may be felt c. Interject that while the procedure is not extremely painful, the client will feel a prick on the finger. d. Report the incident to the education director and request additional instruction for the UAP.

Allow the UAP to complete the procedure, then discuss the painless comment privately with the UAP.

A female nurse who took drugs from the unit for personal use was temporarily released from duty. After completion of mandatory counseling, the nurse has asked administration to allow her to return to work. When the nurse administrator approaches the charge nurse with the impaired nurse request, which action is best for the charge nurse to take? a. Since treatment is completed, assign the nurse to the route RN responsibilities b. Ask to meet with impaired nurse's therapist before allowing her back on the unit. c. Allow the impaired nurse to return to work and monitor medication administration d. Meet with staff to assess their feelings about the impaired nurse's return to the unit.

Allow the impaired nurse to return to work and monitor medication administration

A nurse is preparing to feed a 2-month-old male infant with heart failure who was born with congenital heart defect. Which intervention should the nurse implement? a. Feed the infant when he cries b. Allow the infant to rest before feeding c. Weigh before and after feeding. d. Insert a nasogastric feeding tube.

Allow the infant to rest before feeding

A young adult who is hit with a baseball bat on the temporal area of the left skull is conscious when admitted to the ED and is transferred to the Neurological Unit to be monitored for signs of closed head injury. Which assessment finding is indicative of a developing epidural hematoma? a. Altered consciousness within the first 24 hours after injury. b. Cushing reflex and cerebral edema after 24 hours c. Fever, nuchal rigidity and opisthotonos within hours d. Headache and pupillary changes 48 hours after a head injury

Altered consciousness within the first 24 hours after injury.

The nurse is triaging several children as they present to the emergency room after an accident. Which child requires the most immediate intervention by the nurse?

An 11-year-old with a headache, nausea, and projectile vomiting

In making client care assignment, which client is best to assign to the practical nurse (PN) working on the unit with the nurse? a. An immobile client receiving low molecular weight heparin q12 h. b. A client who is receiving a continuous infusion of heparin and gets out of bed BID c. A client who is being titrated off heparin infusion and started on PO warfarin (Coumadin) d. An ambulatory client receiving warfarin (Coumadin) with INR of 5 second.

An immobile client receiving low molecular weight heparin q12 h. Rationale: A describe the most stable client. The other ones are at high risk for bleeding problems and require the assessment skills.

In preparing assignments for the shift, which client is best for the charge nurse to assign to a practical nurse (PN)? a- An older client who fell yesterday and is now complaining of diplopia b- An adult newly diagnosed with type 1 diabetes and high cholesterol c- A client with pancreatic cancer who is experience intractable pain. d- An older client post-stroke who is aphasic with right-sided hemiplegia

An older client post-stroke who is aphasic with right-sided hemiplegia

After receiving the Braden scale findings of residents at a long-term facility, the charge nurse should to tell the unlicensed assistive personnel (UAP) to prioritize the skin care for which client? a. An older adult who is unable to communicate elimination needs. b. An older man whose sheets are damped each time he is turned. c. A woman with osteoporosis who is unable to bear weight. d. A poorly nourished client who requires liquid supplement.

An older man whose sheets are damped each time he is turned. Rational: a Braden score of less than 18 indicates a risk for skin breakdown, and clients with such score require intensive nursing care. Constant moisture places the client at a high risk for skin breakdown, and interventions should be implemented to pull moisture away from the client's skin. Other options may be risk factors but do not have as high a risk as constant exposure to moisture.

The healthcare provider prescribes heparin protocol at 18 units/kg/hr for a client with a possible pulmonary embolism. This client weighs 144 pounds. The available solution is labeled, heparin sodium 25,000 units in 5% dextrose 250 ml. the nurse should program the pump to deliver how many ml/hr? (Enter numeric value only. If rounding is require round to the nearest whole number.)

Answer 12 Rationale: 144/2.2= 65kg 18units/kg/hr 65 kg x 18units/kg/hr= 1170 units/hr 25000 units heparin/250 ml of D5W = 100 units heparin per ml of solution

A client is receiving and oral antibiotic suspension labeled 250 mg/2ml. The healthcare provider prescribes 200mg every 6 hours. How many ml should the nurse administer at each dose? (Enter numerical value only. If rounding is required, round to the nearest tenth)

Answer: 1.6 Rational: using the formula D/H x Q 200mg/250 mg x 2ml = 200/250 = 1.6 ml

An antacid is prescribed for a client with gastroesophageal (GERD). The client asks the nurse, "How does this help my GERD?" What is the best response by the nurse?

Antacids will neutralize the acid in your stomach

Which class of drugs is the only source of a cure for septic shock?

Antiinfectives

The nurse is caring for a client with acute kidney injury (AKI) secondary to gentamicin therapy the client's serum blood potassium is elevated, which finding requires immediate action by the nurse?

Anuria for the last 12 hours.

A male client with ulcerative colitis received a prescription for a corticosteroid last month, but because of the side effect he stopped taking the medication 6 year ago. Which finding warrants immediate intervention by the nurse?

Anxiety and restlessness.

The nurse reviews the laboratory findings of a client with an open fracture of the tibia. The white blood cell (WBC) count and erythrocyte sedimentation rate (ESR) are elevated. Before reporting this information to the healthcare provider, what assessment should the nurse obtain?

Appearance of wound

A multigravida, full-term, laboring client complains of "back labor". Vaginal examination reveals that the client's 3 cm with 50% effacement and the fetal head is at -1 station. What should the nurse implement? a. Turn the client to a lateral position b. Apply counter-pressure to the sacral area c. Notify the scrub nurse to prepare the OR d. Ambulate the client between contractions

Apply counter-pressure to the sacral area Rationale: B provides pain relief during labor.

The nurse is teaching a male client with multiple sclerosis how to empty his bladder using the Crede Method. When performing a return demonstration, the client applies pressure to the umbilical areas of his abdomen. What instruction should the nurse provide? a- Stroke the inner thigh below the perineum to initiate urinary flow b- Contract, hold, and then relax the pubococcygeal muscle c- Pour warm water over the external sphincter at the distal glans d- Apply downward manual pressure at the suprapubic regions.

Apply downward manual pressure at the suprapubic regions. Rationale: The Crede Method is used for those clients with atonic bladders, which is a concomitant of demyelinating disorders like multiple sclerosis. The client is applying pressure in the wrong region (umbilical Are) and should be instructed to apply pressure at the suprapubic are.

A client present at the clinic with blepharitis. What instructions should the nurse provide for home care? a- Use bilateral eyes patches while sleeping to prevent injury to eyes. b- Wear sunglasses when out of doors to prevent photophobia c- Apply warm moist compresses then gently scrub eyelids with dilute baby shampoo d- Apply warm moist compresses then gently scrub eyelids with dilute baby shampoo.

Apply warm moist compresses then gently scrub eyelids with dilute baby shampoo Rationale: This condition is an inflammation of the eyelids edges that occurs in older adults. Is controlled with eyelid care using warm moist compresses followed by gently scrub eyelids.

When should intimate partner violence (IPV) screening occur? a. As soon as the clinician suspects a problem b. Only when a client presents with an unexplained injury c. As a routine part of each healthcare encounter d. Once the clinician confirms a history of abuse

As a routine part of each healthcare encounter

The healthcare provider explains through an interpreter the risks and benefits of a scheduled surgical procedure to a non-English speaking female client. The client gives verbal consent and the healthcare provider leaves, instructing the nurse to witness the signature on the consent form. The client and the interpreter then speak together in the foreign language for an additional 2 minutes until the interpreter concludes, "She says it is OK." What action should the nurse take next?

Ask for a full explanation from the interpreter of the witnessed discussion

After administering a proton pump inhibitor (PPI), which action should the nurse take to evaluate the effectiveness of the medication?

Ask the client about gastrointestinal pain

A male client with cancer is admired to the oncology unit and tells the nurse that he is in the hospital for palliative care measures. The nurse notes that the client's admission prescription include radiation therapy. What action should the nurse implement? a. Ask the client about his expected goals for the hospitalization b. Explain the palliative care measures can be provided at home c. Notify do radiation department to withhold the treatment for now d. Determine if the client wishes to cancel further radiation treatment

Ask the client about his expected goals for the hospitalization Rationale: Palliative care measures provide relief or control of symptoms, so it is important for the nurse to determine the client's goals for symptom control while receiving treatment in the hospital. Although home care is available the client may not be legible for palliative care at home. Radiation therapy is an effective positive care measure used to manage symptoms and would be appropriate unless the radiation conflicts with the client goals.

A young adult male who is being seen at the employee health care clinic for an annual assessment tell the nurse that his mother was diagnosed with schizophrenia when she was his age and that life with a schizophrenic mother was difficulty indeed. Which response is best for the nurse to provide? a. Ask the client if he is worried about becoming schizophrenic at the age his mother was diagnosed. b. Encourage the client to seek genetic counseling to determine his risk for mental illness. c. Informed the client that his mother schizophrenic has affected his psychological development. d. Tell the client that mental illness has a familial predisposition, so he should see a psychiatrist.

Ask the client if he is worried about becoming schizophrenic at the age his mother was diagnosed.

An older female client tells the nurse that her muscles have gradually been getting weak...what is the best initial response by the nurse? a- Explain that this is an expected occurrence with aging. b- Observe the lower extremity for signs of muscle atrophy c- Review the medical record for recent diagnosis test results. d- Ask the client to describe the changes that have occurred

Ask the client to describe the changes that have occurred

While the nurse is conducting a daily assessment of an older woman who resides in a long-term facility, the client begins to cry and tells the nurse that her family has stopped calling and visiting. What action should the nurse take first? a. Ask the client when a family member last visited her. b. Determine the client's orientation to time and space c. Review the client's record regarding social interactions d. Reassure the client of her family's love for her

Ask the client when a family member last visited her.

When five family members arrive at the hospital, they all begin asking the nurse questions regarding the prognosis of their critically ill mother. What intervention should the nurse implement first? a- Include the family in client's care b- Request the chaplain's presence c- Ask the family to identify a specific spokesperson d- Page the healthcare provider to speak with family.

Ask the family to identify a specific spokesperson

On a busy day, one hour after the shift report is completed, the charge nurse learns that a female staff nurse who lives one hour away from the hospital forgot her prescription eye glasses at home. What action should the charge nurse take? a. Encourage the nurse purchase the reading glasses in the hospital gift shop b. Request another nurse to assist the staff nurse with her documentation c. Ask the nurse to return home and get her prescription eyeglasses for work. d. Tell the staff nurse to take a day off and change her weekly work schedule.

Ask the nurse to return home and get her prescription eyeglasses for work

The healthcare provider prescribes oxycodone/ aspirin 1 tab PO every 4h as needed for pain, for a client with polycystic kidney disease. Before administering this medication, which component of the prescription should the nurse question? a- Aspirin content. b- Dose c- Route d- Risk for addiction

Aspirin content. Rationale: Aspirin content medication are contraindicated for client with polycystic kidney disease because the risk for bleeding.

A male client notifies the nurse that he feels short of breath and has chest pressure radiating down his left arm. A STAT 12-lead electrocardiogram (ECG) is obtained and shows ST segment elevation in leads II, II, aVF and V4R. The nurse collects blood samples and gives a normal saline bolus. What action is most important for the nurse to implement?

Asses for contraindications for thrombolytic therapy

A preoperative client states he is not allergic to any medications. What is the most important nursing action for the nurse to implement next?

Assess client's knowledge of an allergy response

A client who had a percutaneous transluminal coronary angioplasty (PTCA) two weeks ago returns to the clinic for a follow up visit. The client has a postoperative ejection fraction ejection fraction of 30%. Today the client has lungs which are clear, +1 pedal edema, and a 5pound weight gain. Which intervention the nurse implement? a. Arrange transport for admission to the hospital. b. Insert saline lock for IV diuretic therapy. c. Assess compliance with routine prescriptions. d. Instruct the client to monitor daily caloric intake.

Assess compliance with routine prescriptions. Rationale: Fluid retention may be a sign that the client is not taking the medication as prescribed or that the prescriptions may need adjustment to manage cardiac function post-PTCA (normal ejection fraction range is 50 to 75%)

Which nursing intervention has the highest priority for a multigravida who delivered? a. Maintain cold packs to the perineum for 24 hrs. b. Assess the client pain level frequently c. Observe for appropriate interaction with the infants. d. Assess fundal tone and lochia flow

Assess fundal tone and lochia flow Rationale D is the priority intervention because is a multigravida and this pregnancy predisposes the client to uterine atony which could result in hemorrhage.

The nurse is arranging home care for an older client who has a new colostomy following a large bowel resection three day. The clients plan to live with a family member. Which action should the nurse implement? Select all that apply

Assess the client for self-care ability Provide pain medication instructions Teach care of ostomy to care provider

A male client with rheumatoid arthritis is schedule for a procedure in the morning. The... unable to complete the procedure because of early morning stiffness. Which intervention... implement?

Assign a UAP to assist the client with a warm shower early in the morning

The nurse who works in labor and delivery is reassigned to the cardiac care unit for the day because of a low census in labor and delivery. Which assignments is best for the nurse to give this nurse? a. Transfer a client to another unit b. Monitor the central telemetry c. Perform the admission d. Assist cardiac nurses with their assignments

Assist cardiac nurses with their assignments

A client with gestational diabetes, at 39 weeks of gestation, is in the second stage of labor. After delivering of the fetal head, the nurse recognizes that shoulder dystocia is occurring. What intervention should the nurse implement first? a- Prepare the client for an emergency cesarean birth b- Encourage the client to move to a hands-and-knees position. c- Assist the client to sharply flex her thighs up again the abdomen. d- Lower the head of the bed an apply suprapubic pressure.

Assist the client to sharply flex her thighs up again the abdomen.

A confused, older client with Alzheimer's disease becomes incontinent of urine when attempting to find the bathroom. Which action should the nurse implement?

Assist the client's to a bedside commode every two hours

While removing staples from a male client's postoperative wound site, the nurse observes that the client's eyes are closed and his face and hands are clenched. The client states, "I just hate having staples removed." After acknowledging the client's anxiety, what action should the nurse implement?

Attempt to distract the client with general conversation

A client who had an open cholecystectomy two weeks ago comes to the emergency department with complaints of nausea, abdominal distention, and pain. Which assessment should the nurse implement? a. Auscultate all quadrant of the abdomen. b. Perform a digital rectal exam c. Palpate the liver and spleen d. Obtain a hemoccult of the client's stool

Auscultate all quadrant of the abdomen.

A client in the intensive care unit is being mechanically ventilated, has an indwelling urinary catheter in place, an exhibiting signs of restlessness. Which action should the nurse take fist?

Auscultate bilateral breath sounds

A male client, who is 24 hours postoperative for an exploratory laparotomy, complains that he is "starving" because he has had no "real food" since before the surgery. Prior to advancing his diet, which intervention should the nurse implement? a. Discontinue intravenous therapy b. Obtain a prescription for a diet change c. Assess for abdominal distention and tenderness. d. Auscultate bowel sounds in all four quadrants

Auscultate bowel sounds in all four quadrants

The nurse identifies an electrolyte imbalance, an elevated pulse rate, and elevated BP for a client with chronic kidney disease. Which is the most important action for the nurse to take? a. Monitor daily sodium intake. b. Record usual eating patterns. c. Measure ankle circumference. d. Auscultate for irregular heart rate.

Auscultate for irregular heart rate. Rational: Chronic kidney failure (CKF) is a progressive, irreversible loss of kidney functions, decreasing glomerular filtration rate (GFR), and the kidney's inability to excrete metabolic waste products and water, resulting in fluid overload, elevated pulse, elevated BP and electrolytes imbalances. The most important action for the nurse to implement is to auscultate for irregular heart rate (D) due to the decreased excretion of potassium by the kidneys. (A, B, and C) are not as important as monitoring for fatal cardiac dysrhythmias related to hyperkalemia.

A client in the intensive care unit is being mechanically ventilated, has an indwelling urinary catheter in place, and is exhibiting signs of restlessness. Which action should the nurse take first? a. Review the heart rhythm on cardiac monitors b. Check urinary catheter for obstruction c. Auscultated bilateral breath sounds d. Give PRN dose of lorazepam (Ativan)

Auscultated bilateral breath sounds Rationale: Restlessness often results from decreased oxygenation, so breath sounds should be assessed first. Giving an anxiolytic such as lorazepam, might be indicated but first the client should be assessed for the cause of the restlessness. An obstruction in the urinary drainage system can cause a distended bladder that may result in restlessness, but patent airway is the priority intervention. The client should be assessed before evaluating the cardiac rhythm on the monitor.

Oxygen at 5l/min per nasal cannula is being administered to a 10 year old child with pneumonia. When planning care for this child, what principle of oxygen administration should the nurse consider?

Avoid administration of oxygen at high levels for extended periods.

The nurse is preparing a discharge teaching plan for a client who had a liver transplant. Which instruction is most important to include in this plan? a. Limit intake fatty foods for one month after surgery. b. Notify the healthcare provider if edema occurs. c. Increase activity and exercise gradually, as tolerated. d. Avoid crowds for first two months after surgery.

Avoid crowds for first two months after surgery.

Which instruction is most important for the nurse to provide a client who is being discharge following treatment for Guillain-Barre syndrome? a. Avoid exposure to respiratory infections. b. Use relaxation exercise when anxious c. Continue physical therapy at home d. Plan short, frequent rest periods.

Avoid exposure to respiratory infection

A client with a recent colostomy expresses concern about the ability to control flatus. Which intervention is most important for the nurse to include in the client's plan of care? a. Adhere to a bland diet whenever planning to eat out b. Decrease fluid intake at meal times c. Avoid foods that caused gas before the colostomy d. Eliminate foods high in cellulose

Avoid foods that caused gas before the colostomy

An adult female client is admitted to the psychiatric unit with a diagnosis of major depressive...medication therapy, the nurse notices the client has more energy, is giving her belongings...mood. Which intervention is best for the nurse to implement a- Support the client by telling her what wonderful progress she is making. b- Ask the client if she has had any recent thoughts of harming herself. c- Reassure the client that the antidepressant drugs are apparently effective d- Tell the client to keep her belongings because she will need hem at discharge.

B

The nurse is preparing an older client for discharge following cataract extraction. Which instruction should be include in the discharge teaching? a. Do not read without direct lighting for 6 weeks. b. Avoid straining at stool, bending, or lifting heavy objects. c. Irrigate conjunctiva with ophthalmic saline prior to installing antibiotic ointment. d. Limit exposure to sunlight during the first 2 weeks when the cornea is healing.

Avoid straining at stool, bending, or lifting heavy objects Rationale: after cataract surgery, the client should avoid activities which increase pressure and place strain on the suture line.

595. A client admitted to the emergency center had inspiratory and expiratory wheezing, nasal flaring, and thick, tenacious sputum secretions observed during the physical examination. Based on these assessment findings, what classification of pharmacologic agents should the nurse anticipate administering? a. Beta blockers b. Bronchodilators c. Corticosteroids d. Beta-adrenergic

B

A male client with impaired renal function who takes ibuprofen daily for chronic arthritis...gastrointestinal (GI) bleeding. After administering IV fluids and a blood transfusion, his blood pressure is 100/70, and his renal output is 20 ml / hour. Which intervention should the nurse include in hours? a. Maintain the client NPO during the diuresis phase b. Evaluate daily serial renal laboratory studies for progressive elevations. c. Observe the urine character for sedimentation and cloudy appearance. d. Monitor for onset of polyuria greater than 150ml/hr.

B

The nurse is assessing a postpartum client who is 36 hours post-delivery. Which finding should the nurse report to the healthcare provider? a. White blood count of 19,000 mm3 b. Oral temperature of 100.6 F c. Fundus deviated to the right side d. Breasts are firm when palpated

B Rationale: A temperature greater than 100.4 F (38 C) (B), which is indicative of endometriosis (infection of the lining of the uterus), should be reported to the health care provider. (A and D) are findings that are within normal limits in the postpartum period. Fundal deviation to one side (C) is an expected finding related to a full bladder, so the nurse should encourage the client to void.

A nurse is planning discharge care for a male client with metastatic cancer. The client tells the nurse that he plans to return to work despite pain, fatigue, and impending death. Which goals is most important to include in this client's plan of care? a. Implements decisions about future hospices services within the next 3 months. b. Maintaining pain level below 4 when implementing outpatient pain clinic strategies. c. Request home health care if independence become compromised for 5 days. d. Arranges for short term counseling stressors impact work schedule for 2 weeks.

B Rationale: An outpatient pain clinic provides the interdisciplinary services needed to manage chronic pain. Also, the client has a terminal disease and is being discharge home, hospice and health care are not indicating currently. Short term counseling is not an option.

53- A male client with a long history of alcoholism is admitted because of mild confusion and fine motor tremors. He reports that he quit drinking alcohol and stopped smoking cigarettes one month ago after his brother died of lung cancer. Which intervention is most important for the nurses to include in the client's plan of care? a. Determine client's level current blood alcohol level. b. Observe for changes in level of consciousness. c. Involve the client's family in healthcare decisions. d. Provide grief counseling for client and his family.

B Rationale: Based on the client's history of drinking, he may be exhibiting sign of hepatic involvement and encephalopathy. Changes in the client's level of consciousness should be monitored to determine if he able to maintain consciousness, so neurological assessment has the highest priority.

After the risk and benefits of having a cardiac catheterization are reviewed by the healthcare provider, an older adult with unstable angina is scheduled for the procedure. When the nurse presents the consent form for signature, the client asks how the wires will keep a heart heating during the procedure. What action should the nurse take? a. Explain the procedure again in detail and clarify any misconceptions. b. Notify the healthcare provider of the client's lack of understanding. c. Call the client's next of kin and have them provide verbal consent. d. Postpone the procedure until the client understands the risk and benefits.

B Rational: the nurse is only witnessing the signature and is not responsible for the client's understanding of the procedure. The healthcare provider needs to clarify any questions and misconceptions. Explaining the procedure again is the healthcare provider's legal responsibility. The other options are not indicated.

In caring for a client receiving the amino glycoside antibiotic gentamicin, it is most important for the nurse to monitor which diagnostic test? a- Urinalysis b- Serum creatinine c- Serum osmolarity d- Liver enzymes.

B Rationale: Aminoglycosides can cause nephrotoxicity, so it is important for the nurse to monitor the serum creatinine level can monitor the renal function.

A woman just learned that she was infected with Heliobacter pylori. Based on this finding, which health promotion practice should the nurse suggest? a. Schedule a colonoscopy within the next month. b. Encourage screening for a peptic ulcer. c. Screen all family member for hepatitis A d. Eat small, frequent meals thought the day.

B Rationale: Helicobacter pylori is a gram- negative organism than can colonize in the stomach and is associated with peptic ulcers formation.

The nurse weighs a 6-month-old infant during a well-baby check-up and determines that the baby's weight has tripled compared to the birth weight of 7 pounds 8 ounces. The mother asks if the baby is gaining enough weight. What response should the nurse offer? a- Your baby is gaining weight right on schedule b- What food does your baby usually eat in a normal day? c- The baby is below the normal percentile for weight gain What was the baby's weight at the last well-baby clinic visit

B Rationale: The normal weight gain in the first year of life is approx. twice the birth weight

An adult who is 5 feet 5 inches (165.1 cm) tall and weighs 90 lb. (40.8 Kg) is admitted with a diagnosis of chronic anorexia. The client receives a regular diet for 2 days, and the client's medical records indicates that 100% of the diet provided has been consumed. However the client's weight on the third day morning after admission is 89 lb. (40.4 Kg). What action should the nurse implement? a. Examine the client's room for hidden food. b. Assign staff to monitor what the client eats. c. Ask the client if the food provided is being eaten or discarded. d. Provide the client with a high calorie diet.

B Rationale: clients with an eating disorder have an unhealthy obsession with food. The client's continued weight loss, despites indication that the client has consumed 100% of the diet, should raise questions about the client's intake of the food provided, so the client should be observed during meals to prevent hiding or throwing away food. Other options may be accurate but ineffective and unnecessary.

A child is diagnosed with acquired aplastic anemia. The nurse knows that this child has the best prognosis with which treatment regimen?

Bone marrow transplantation

After removing a left femoral arterial sheath, which assessment finding warrant immediately interventions by the nurse? (Select all that applied.) a- Tenderness over insertion b- Unrelieved back and flank pain. c- Cool and pale left leg and foot. d- Left groin egg-size hematoma. e- Quarter size red drainage at site.

BCD

A male client recently released from a correctional facility arrives at the clinic with a cough, fever, and chills. His history reveals active tuberculosis (TB) 10 years ago. What action should the nurse implement? (Select all that apply) a- Administer a PPD test b- Schedule the client for the chest radiograph c- Obtain sputum for acid fast bacillus (AFB) testing d- Place a mask on the client until he is moved to isolation. e- Send the client home with instructions for a prescribe antibiotic.

BCD Rationale: Client with history of TB a chest x-ray and sputum are indicated. The client sign and symptoms indicate the pt should wear mask to protect others.

Assessment by the home health nurse of an older client who lives alone indicates that client has chronic constipations. Daily medications include furosemide for hypertension and heart failure and laxatives. To manage the client's constipation, which suggestions should the nurse provide? (Select all that apply) a. Decrease laxative use to every other day and use oil retention enemas as needed. b. Include oatmeal with stewed pruned for breakfast as often as possible. c. Increase fluid intake by keeping water glass next to recliner. d. Recommend seeking help with regular shopping and meal preparation. e. Report constipation to healthcare provider related to cardiac medication side effects.

BCD Rational: older adult are at higher risk for chronic constipation due to decreased gastrointestinal muscle tone leading to reduce motility. Oatmeal with prunes increases dietary fiber and bowel stimulation, thereby decreasing need for laxatives. Increased fluid intake also decreases constipations. Assistance with food preparation might help the client eat more fresh fruits and vegetables and result on less reliance on microwaved and fast foods, which are usually high in sodium and fat with little fiber. Laxatives can be reduced gradually by improving the diet, without resorting to using enemas.

A client with type 2 diabetes mellitus is admitted for antibiotic treatment for a leg ulcer. To monitor the client for the onset of hyperosmolar hyperglycemic nonketotic syndrome (HHNS), what actions should the nurse take? (Select all that apply) a. Check urine for ketones b. Measure blood glucose c. Monitor vital signs d. Assessed level of consciousness e. Obtain culture of wound

BCD Rationale: Blood glucose greater than 600 mg/dl (33.3 mmol/L SI), vital sign changes in mental awareness are indicators of possible HHNS. Urine ketones are monitored in diabetic ketoacidosis. Wound culture is performed prior to treating the wound infection but is not useful in monitoring for HHNS.

The nurse is teaching a client with atrial fibrillation about a newly prescribed medication, dronedarone. Which information should the nurse include in client interactions? (Select all that apply) a- Discontinue medication when palpitation subside. b- Avoid eating grapefruit or drinking grapefruit juice. c- Report changes in the use of daily supplements d- Notify your health care provider if your skin looks yellow e- If a dose is missed, the next dose should be double.

BCD Rationale: Side effects can increase if the client consume grapefruit. OTC medications or herbal should be reported for possible drugs interactions. Hepatic injury can occur, and the client should report sign of jaundice or itching, or right upper quadrant pain.

An older adult male who had an abdominal cholecystectomy has become increasingly confused and disoriented over the past 24 hours. He is found wandering into another client's room and is return to his room by the unlicensed assistive personnel (UAP). What actions should the nurse take? (Select all that apply). a. Apply soft upper limb restrains and raise all four bed rails b. Report mental status change to the healthcare provider c. Assess the client's breath sounds and oxygen saturation d. Assign the UAP to re-assess the client's risk for falls e. Review the client's most recent serum electrolyte values

BCE Rationale: The healthcare provider should be informed of changes in the client's condition (B) because this behavior may indicate a postoperative complication. Diminished oxygenation (C) and electrolyte imbalance (E) may cause increased confusion in the older adult. Raising all four bed rails (A) may lead to further injury if the client climbs over the rails and falls and restrains should not be applied until other measures such as re-orientation are implemented. The nurse should assess the client's increased risk for falls, rather than assigning this to the UAP (D).

The nurse assesses a female client with obstructive sleep apnea syndrome (OSAS) who is 5 feet tall (152 cm) and weighs 155 pounds (70 kg), the client's 24 hour diet history includes: no breakfast, cheeseburger and fries for lunch; lasagna, chocolate ice cream and a cola drink for dinner, and 2 glasses of wine in the evening before going to bed for a total caloric intake of 3500 calories. What instructions should the nurse provide? (Select all that apply) a. Maintain current caloric intake b. Avoid use of alcohol as a sleep aide at bedtime c. Reduce intake of dairy products d. Start a weight loss program e. Set a goal of increasing BMI (Body Mass Index)

BD

The daughter of an older female client tells the clinic nurse that she is no longer able to care for her mother since her mother has lost the ability to perform activities of daily living (ADLs) due to aging. Which options should the nurse discuss with the daughter? a. Home hospice agency b. Long-term care facility c. Rehabilitation facility d. Independent senior apartment e. Home health agency

BE Long term care facilities and home health agencies performs ADLs. Hospice provides empathetic, attentive care for dying. C provide physical therapy to strengthen a part of the body.

When conducting diet teaching for a client who was diagnosed with hypoparathyroidism, which foods should the nurse encourage the client to eat? a- Nuts b- Yogurt. c- Fresh turkey d- Fresh chicken e- Processed cheese.

BE Rationale: In hypoparathyroidism, the client's diet should be supplemented with calcium rich foods which include dairy products.

Which information is more important for the nurse to obtain when determining a client's risk for (OSAS)? a- Body mass index b- Level of consciousness c- Self-description of pain d- Breath sounds

BMI

A male client has received a prescription for orlistat for weight and nutrition management. In addition to the medication, the client states he plans to take a multivitamin. What teaching should the nurse provide? a. As a nutritional supplement, orlistat already contains all the recommended daily vitamins and minerals. b. Multivitamins are contraindicated. During treatment with weight-control medications such as orlistat c. Be sure to take the multivitamin and the medication at least two hours apart for best absorption and effectiveness. d. Following a well-balanced diet is a much healthier approach to good nutrition than depending on a multivitamin.

Be sure to take the multivitamin and the medication at least two hours apart for best absorption and effectiveness

An unconscious client is admitted to the intensive care unit and is placed on a ventilator. The ventilator alarms continuously and the client's oxygen saturation level is 62%. What action should the nurse take first?

Begin manual ventilation immediately.

A client has a prescription for lorazepam 2mg for alcohol withdrawal symptoms. Which finding... the client? a. Blood pressure 149/101 b. Irregular pulse rate of 80 c. Oral temperature is 98.9 F (37.1 C) d. Pain rated 7 on scale 1-10

Blood pressure 149/101

After a routine physical examination, the healthcare admits a woman with a history of Systemic Lupus Erythematous (SLE) to the hospital because she has 3+ pitting ankle edema and blood in her urine. Which assessment finding warrants immediate intervention by the nurse? a. Dark, rust-colored urine b. Urine output 300 ml/hr c. Joint and muscle aches d. Blood pressure 170/98

Blood pressure 170/98 Rationale: SLE can result in renal complication such as glomerulonephritis, which can cause a critically high blood pressure that necessitates immediate intervention. A, B and C are symptoms of glomerulonephritis and should be treated once the blood pressure is under control

A female client who is admitted to the mental health unit for opiate dependency is receiving clonidine 0.1 mg PO for withdrawal symptoms. The client begins to complain of feeling nervous and tells the nurse that her bones are itching. Which finding should the nurse identify as a contraindication for administering the medication?

Blood pressure 90/76 mm Hg

A client with C-6 spinal cord injury rehabilitation. In the middle of the night the client reports a severe, pounding headache, and has observable piloerection or "goosebumps". The nurse should asses for which trigger? a. Loud hallway noise. b. Fever c. Full bladder d. Frequent cough.

C Rational: a pounding headache is a sign of autonomic hyperreflexia, an acute emergency that occurs because of an exaggerated sympathetic response in a client with a high level spinal cord injury. Any stimulus below the level of injury can trigger autonomic hyperreflexia, but the most common cause is an overly distended bladder. The other options are unlikely to produce the manifestation of autonomic hyperreflexia.

The nurse is assessing an older adult with type 2 diabetes mellitus. Which assessment finding indicates that the client understands long- term control of diabetes? a. The fasting blood sugar was 120 mg/dl this morning. b. Urine ketones have been negative for the past 6 months c. The hemoglobin A1C was 6.5g/100 ml last week d. No diabetic ketoacidosis has occurred in 6 months.

C Rationale: A hemoglobin A1C level reflects he average blood sugar the client had over the previous 2 to 3 month, and level of 6.5 g/100 ml suggest that the client understand long-term diabetes control. Normal value in a diabetic patient is up to 6.5 g/100 ml.

A client with emphysema is being discharged from the hospital. The nurse enters the client's room to complete discharge teaching. The client reports feeling a little short of breath and is anxious about going home. What is the best course of action? a. Postpone discharge instructions at this time and offer to contact the client by phone in a few days b. Invite the client to return to the unit for discharge teaching in a few days, when there is less anxiety c. Provide only necessary information in short, simple explanations with written instructions to take home d. Give detailed instructions speaking slowly and clearly while looking directly at the client when speaking

C Rationale: Simple, short explanations should be provided. Information is not retained when the recipient is anxious, and too much information can increase worry. Ethically, discharge instructions may not be postponed.

After receiving report, the nurse can most safely plan to assess which client last? The client with... a. A rectal tube draining clear, pale red liquid drainage b. A distended abdomen and no drainage from the nasogastric tube c. No postoperative drainage in the Jackson-Pratt drain with the bulb compressed d. Dark red drainage on a postoperative dressing, but no drainage in the Hemovac®.

C Rationale: The most stable client is the one with a functioning drainage device and no drainage. This client can most safely be assessing last. Other clients are either actively bleeding, have an obstruction in the nasogastric tube which may result in vomiting, or may be bleeding and / or may have a malfunction in the Hemovac® drain.

When assessing a multigravida the first postpartum day, the nurse finds a moderate amount of lochia rubra, with the uterus firm, and three fingerbreadths above the umbilicus. What action should the nurse implement first? a. Massage the uterus to decrease atony b. Check for a distended bladder c. Increase intravenous infusion d. Review the hemoglobin to determined hemorrhage

C Rationale: a fundus that is dextroverted (up to the right) and elevated above the umbilicus is indicative of bladder distension/urine retention.

The nurse is ready to insert an indwelling urinary catheter as seen in the picture. At this point in the procedure, what actions should the nurse take before inserting the catheter? (Select all that apply)

C. Gently palpate the client's bladder for distention D. Hold the catheter 3 - 4 inches (7.5 - 10 cm) from its tip E. Secure the urinary drainage bag to the bed frame

During the transfer of a client who had major abdominal surgery this morning, the post anesthesia care unit (PACU) nurse reports that the client, who is awake and responsive continues to report pain and nausea after receiving morphine 2 mg IV and ondansetron 4 mg IV 45 mints ago. Which elements of SBAR communication are missing from the report given by the PACU nurse? (Select all that apply) a- Situation b- Background c- Assessment d- Recommendation e- Rationales.

CDE

The unit clerk reports to the charge nurse that a healthcare provider has written several prescriptions that are illegible and it appears the healthcare provider used several unapproved abbreviations in the prescriptions. What actions should the charge nurse take?

Call the healthcare provider who wrote the prescription

Which statement is accurate regarding the pathological changes in the pulmonary system associated with acute (adult) respiratory distress syndrome (ARDS)? a. Capillary hydrostatic pressure exceeds colloid osmotic pressure, producing interstitial edema b. A high ventilation-to-perfusion ratio is characteristic of affected lung fields in ARDS c. Functional residual capacity and lung compliance increase as the disease progresses d. Interstitial edema that occurs due to capillary fluid shifts is usually more serious than alveolar edema.

Capillary hydrostatic pressure exceeds colloid osmotic pressure, producing interstitial edema

A postpartal client complains that she has the urge to urinate every hour but is only able to void a small amount. What interventions provides the nurse with the most useful information? a. Initiate a perineal pad count b. Catheterize for residual urine after next voiding c. Assess for a perineal hematoma d. Determine the client's usual voiding pattern

Catheterize for residual urine after next voiding

When assessing a multigravida the first postpartum day, the nurse finds a moderate amount of lochia rubra, with the uterus firm, and three fingerbreadths above the umbilicus. What action should the nurse implement first

Check for a distended bladder

The nurse is preparing to administer an infusion of amino acid-dextrose total parenteral nutrition (TPN) through a central venous catheter (CVC) line. Which action should the nurse implement first? 53- The nurse is preparing to administer an infusion of amino acid-dextrose total parenteral nutrition (TPN) through a central venous catheter (CVC) line. Which action should the nurse implement first? a- Attached de IV tubing to the central line. b- Check the TPN solution for cloudiness c- Set the infusion PUMP at the prescribed rate. d- Prime the IV tubbing with the TPN solution.

Check the TPN solution for cloudiness

A nurse who is working in the emergency department triage area is presented with four clients at the same time. The client presented with which symptoms requires the most immediate intervention by the nurse? a. Low-grade fever, headache, and malaise for the past 72 hours b. Unable to bear weight on the left foot, with the swelling and bruising c. Chest discomfort one hour after consuming a large, spicy meal d. One-inch bleeding laceration on the chain of the crying five-year-old

Chest discomfort one hour after consuming a large, spicy meal Rationale: Emergency triage involves quick assessment to prioritize the need for further evaluation and care. Those with trauma, chest pain, respiratory distress, or acute neurological changes are priority. In this example, while clients with other conditions require attention, the client with chest discomfort is at greatest risk and is a priority.

A client with bleeding esophageal varices receives vasopressin (Pitressin) IV. What should the nurse monitor for during the IV infusion of this medication?

Chest pain and dysrhythmia

After diagnosis and initial treatment of a 3 year old with Cystic fibrosis, the nurse provides home care instructions to the mother, which statement by the child's mother indicates that she understands home care treatment to promote pulmonary functions?

Chest physiotherapy should be performed twice a day before a meal.

The nurse is administering a 750 ml cleansing enema to an adult client. After approximately150 ml of enema has informed, the client states, 'stop I can't hold anymore." What action should the nurse take? a. Clamp the tubing and instruct the client to breathe deeply before continuing. b. Discontinue infusing the enema and record the client's response. c. Slow infusion of the enema and instruct the client to use paint breathing d. Place the client on the bedpan and continue infusion of the enema.

Clamp the tubing and instruct the client to breathe deeply before continuing. Rationale: Clamping the tube momentarily allows the muscle to relax and prevents expulsion of the solution prematurely. B may be eventually necessary but A should be tried first.

An older woman who was recently diagnosed with end stage metastatic breast cancer is admitted because she is experiencing shortness of breath and confusion. The client refuses to eat and continuously asks to go home. Arterial blood gases indicate hypoxia. Which intervention is most important for the nurse to implement?

Clarify end of life desires

The mother of a child recently diagnosed with asthma asks the nurse how to help protect her child from having asthmatic attacks. To avoid triggers for asthmatic attacks, which instructions should the nurse provide the mother? (Select all that apply)

Close car windows and use air conditioner Avoid sudden changes in temperature Keep away from pets with long hair Stay indoors when grass is being cut

The home health nurse is assessing a male client who has started peritoneal dialysis (PD) 5 days ago. Which assessment finding warrants immediate intervention by the nurse? a. Finger stick blood glucose 120 mg/dL post exchange b. Arteriovenous (AV) graft surgical site pulsations. c. Anorexia and poor intake of adequate dietary protein d. Cloudy dialysate output and rebound abdominal pain

Cloudy dialysate output and rebound abdominal pain

Which assessment finding of a postmenopausal woman necessitates a referral by the nurse to the healthcare provider for evaluation of thyroid functioning

Cold sensitivity

A client's subjective data includes dysuria, urgency, and urinary frequency. What action should the nurse implement next?

Collect a clean-catch specimen

A 7-year-old boy is brought to the clinic because of facial edema. He reports that he has been voiding small amounts of dark, cloudy, tea-colored urine. The parents state that their son had a sore throat 2 weeks earlier, but it has resolved. After assessing the child's vital signs and weight, what intervention should the nurse implement next? a. Perform an otoscopic examination b. Measure the child's abdominal girth c. Collect a urine specimen for routine urinalysis d. Obtain a blood specimen for serum electrolytes.

Collect a urine specimen for routine urinalysis Rationale: Acute glomerulonephritis is an auto-immune reaction to a precursory streptococcus. Manifestation of AGN include oliguria, edema, hypertension.

A client who is at 10-weeks gestation calls the clinic because she has been vomiting for the past 24 hours. The nurse determines that the client has no fever. Which instructions should the nurse give to this client?

Come to the clinic to be seen by a healthcare provider

The nurse is caring for a client following a myelogram. Which assessment finding should the nurse report to the healthcare provider immediately?

Complain of headaches and stiff neck

The nurse is changing a client's IV tubing and closes the roller clamp on the new tubing setup when the bag of solution is....which action should the nurse take to ensure adequate filling of the drip chamber? a. Lower the IV bag to a flat surface b. Compress the drip chamber c. Open the roller clamp d. Squeeze the bag of IV solution

Compress the drip chamber

An adult female client is admitted to the psychiatric unit because of a complex handwashing ritual she performs daily that takes two hours or longer to complete. She worries about staying clean and refuses to sit on any of the chairs in the day area. This client's handwashing is an example of which clinical behavior? a. Addiction b. Phobia c. Compulsion d. Obsession

Compulsion

A nurse is conducting a physical assessment of a young adult. Which information provides the best indication of the individual nutritional status? a. A 24-hour diet history b. History of a recent weight loss c. Status of current petite d. Condition of hair, nails, and skin

Condition of hair, nails, and skin Rationale: The assessment of hair, nails and skin is most indicative of long-term nutritional status, which is important in the healing process.

A nurse plans to call the healthcare provider to report an 0600 serum potassium level of 2 mEq/L or mmol/L (SI), but the charge nurse tells the nurse that the healthcare provider does not like to receive early morning calls and will make rounds later in the morning. What action should the nurse make?

Contact the healthcare provider immediately to report the laboratory value regardless of the advice

A client with end-stage liver failure is declared brain dead. The family wants to discontinue feeding and donate any viable organs. Which action should the nurse take? a- Contact the regional organ procurement agency b- Convene a multidisciplinary care conference c- Explain that client may not be an organ donor candidate d- Discontinue feeding and fluids per the family's request.

Contact the regional organ procurement agency

A 6-year-old child with acute infectious diarrhea is placed on a rehydration therapy... Which action should the nurse instruct the parents to take if the child begins to vomit? a. Continue giving ORS frequently in small amounts b. Withhold all oral intake c. Supplement ORS with gelatin or chicken broth d. Provide only bottle water.

Continue giving ORS frequently in small amounts

A primigravida a 40-weeks gestation with preeclampsia is admitted after having a seizure in the hot tub at a midwife's birthing center. Based on documentation in the medical record, which action should the nurse implement? (Click on each chart tab for additional information. Please be sure to scroll to the bottom right corner of each tab to view all information contained in the client's medical record.) a. Continue to monitor the client's blood pressure hourly. b. Inform the healthcare provider of CBC results c. Update the nursery staff on the client's status d. Give a dose of calcium gluconate per preeclampsia protocol.

Continue to monitor the client's blood pressure hourly Rationale: The laboratory results, urinary output, FHR, and vital signs are within expected ranges for a client who is receiving magnesium sulfate for preeclampsia. The client remains hypertensive, son continued hourly monitoring A is indicated Client magnesium center therapeutic range (5 to 7 mEq/L)

The nurse is assessing the thorax and lungs of a client who is having respiratory difficulty. Which finding is most indicative of respiratory distress? a- Contractions of the sternocleidomastoid muscle. b- Respiratory rate of 20 breath/mints c- Downward movement of diaphragm with inspiration d- A pulse oximetry reading of SpO2 95%

Contractions of the sternocleidomastoid muscle Rationale: Force inspiration needs to use accessories muscle and rib cage.

The nurse is using a straight urinary catheter kit to collect a sterile urine specimen from a female client. After positioning am prepping this client, rank the actions in the sequence they should be implemented. (Place to first action on the top on the last action on the bottom.) 1. Open the sterile catheter kit close to the client's perineum. 2. Don sterile gloves and prepare to sterile field 3. Cleanse the urinary meatus using the solution, swabs, and forceps provided 4. Place distal end of the catheter in sterile specimen cup and insert catheter into meatus

Correct : ODCP 1. Open the sterile catheter kit close to the client's perineum. 2. Don sterile gloves and prepare to sterile field 3. Cleanse the urinary meatus using the solution, swabs, and forceps provided 4. Place distal end of the catheter in sterile specimen cup and insert catheter into meatus Rationale: First the kit should be open near the clients to minimize the risk of contamination during the collection of the sterile specimen. Once the kit is opened, sterile gloves should be donned to prepare the sterile field. Then the clients' meatus should be cleansed, and the catheter inserted while to distal end of the catheter drains urine into the sterile specimen cup or receptacle.

Which action should the nurse implement with auscultating anterior breath sounds? (Place the first action on top and last action on the bottom) Auscultate bronchovesicular sounds from side to side the first and second intercostal spaces Place stethoscope in suprasternal area to auscultate for bronchial sounds Document normal breath sounds and location of adventitious breath sounds Displace female breast tissue and apply stethoscope directly on chest wall to hear vesicular sounds

Correct order: (PADD) 1. Place stethoscope in suprasternal area to auscultate for bronchial sounds 2. Auscultate bronchovesicular sounds from side to side the first and second intercostal spaces 3. Displace female breast tissue and apply stethoscope directly on chest wall to hear vesicular sounds 4. Document normal breath sounds and location of adventitious breath sounds

A client is complaining of intermittent, left, lower abdominal pain that began two days ago...implement the following interventions?

Correct orders: (DPIA) 1. Determine when the client had last bowel movement 2. Position client supine with knees bent 3. Inspect abdominal contour 4. Auscultate all four abdominal quadrants

A client with osteoporosis related to long-term corticosteroid therapy receives a prescription for calcium carbonate. Which client's serum laboratory values requires intervention by the nurse? a. Total calcium 9 mg/dl (2.25 mmol/L SI) b. Creatinine 4 mg/dl (354 micromol/L SI) c. Phosphate 4 mg/dl (1.293 mmol/L SI) d. Fasting glucose 95 mg/dl (5.3 mmol/L SI)

Creatinine 4 mg/dl (354 micromol/L SI

The nurse is assessing the emotional status of a client with Parkinson's disease. Which client finding is most helpful in planning goals to meet the client's emotional needs? a- Stares straight ahead without blinking b- Face does not convey any emotion c- Cries frequently during the interview d- Uses a monotone when speaking

Cries frequently during the interview

While caring for a client's postoperative dressing, the nurse observes purulent drainage at the wound. Before reporting this finding to the healthcare provider, the nurse should review which of the client's laboratory values? a. Serum albumin b. Creatinine level c. Culture for sensitive organisms. d. Serum blood glucose (BG) level

Culture for sensitive organisms. RATIONALE: A client who has a postoperative dressing with purulent drainage from the wound is experiencing an infection. The nurse should review the client's laboratory culture for sensitive organisms (C) before reporting to the healthcare provider. (A, B and D) are not indicated at this time.

A client exposed to tuberculosis is scheduled to begin prophylactic treatment with isoniazid. Which information is most important for the nurse to note before administering the initial dose? a. Conversion of the client's PPD test from negative to positive. b. Length of time of the exposure to tuberculosis. c. Current diagnosis of hepatitis B. d. History of intravenous drug abuse.

Current diagnosis of hepatitis B Rationale: prophylactic treatment of tuberculosis with isoniazid is contraindicated for persons with liver disease because it may cause liver damage. The nurse should withhold the prescribed dose and contact the healthcare provider. Other options do not provide data indicating the need to question or withhold the prescribed treatment.

A client is admitted with a wound on the right hand and associated cellulitis. In assessing the client's hand, which finding required most immediate follow-up by the nurse?

Cyanotic nailbeds

A clinical trial is recommended for a client with metastatic breast cancer, but she refuses to participate and tells her family that she does not wish to have further treatments. The client's son and daughter ask the nurse to try and convince their mother to reconsider this decision. How should the nurse respond? a. Ask the client with her children present if she fully understands the decision she has made. b. Discuss success of clinical trials and ask the client to consider participating for one month. c. Explain to the family that they must accept their mother's decision. d. Explore the client's decision to refuse treatment and offer support

D Rationale: as long as the client is alert, oriented and aware of the disease prognosis, the healthcare team must abide by her decisions. Exploring the decision with the client and offering support provides a therapeutic interaction and allows the client to express her fears and concerns about her quality of life. Other options are essentially arguing with the client's decisions regarding her end of life treatment or diminish the opportunity for the client to discuss her feelings

The nurse is teaching a mother of a newborn with a cleft lip how to bottle feed her baby using medela haberman feeder, which has a valve to control the release of milk and a slit nipple opening. The nurse discusses placing the nipple's elongated tip in the back of the oral cavity. What instructions should the nurse provide the mother about feedings? a. Squeeze the nipple base to introduce milk into the mouth b. Position the baby in the left lateral position after feeding c. Alternate milk with water during feeding d. Hold the newborn in an upright position

D Rationale: the mother should be instructed to hold the infant during feedings in a sitting or upright position to prevent aspiration. Impaired sucking is compensated using special feeding appliances and nipples such as the Haberman feeder that prevents aspiration by adjusting the flow of mild according to the effort of the neonate. Squeezing the nipple base may introduce a volume that is greater than the neonate can coordinate swallowing. The preferred position of an infant after feeding is on the right side to facilitate stomach emptying. Sucking difficulty impedes the neonate's intake of adequate nutrient needed for weight gain and water should be provided after the feeding to cleanse the oral cavity and not fill up the neonate's stomach.

When conducting diet teaching for a client who was diagnosed with nutritional anemia in pregnancy, which foods should the nurse encourage the client to eat? (Select all that apply) a. Seeds, spices, lettuce b. Consomme, celery, carrot c. Oranges, orange juice, bananas d. Fortified whole wheat cereals, whole-grain pasta, brown rice e. Spinach, kale, dried raisins and apricots

DE Rationale: Nutritional anemia in pregnancy should be supplemented with additional iron in the diet. Foods that are high in iron content are often protein based, whole grains (D), green leafy vegetables and dried fruits (E). (A, B, and C) are not iron rich sources

Following insertion of a LeVeen shunt in a client with cirrhosis of the liver, which assessment finding indicates to the nurse that the shunt is effective? a- Decrease abdominal girth b- Increased blood pressure c- Clear breath sounds d- Decrease serum albumin.

Decrease abdominal girth

A client with hyperthyroidism is receiving propranolol (Inderal). Which finding indicates that the medication is having the desired effect a. Decrease in serum T4 levels b. Increase in blood pressure c. Decrease in pulse rate d. Goiter no longer palpable.

Decrease in pulse rate Rationale: Beta blockers such as propranolol help control the symptoms of hyperthyroidism, such as palpitations or tachycardia, but do not alter thyroid hormone levels, B is not a desired effect in hyperthyroidism. Beta blocker do not impact the presence of a goiter.

An elderly female client with osteoarthritis reports increasing pain and stiffness in her right knee and asks how to reduce these symptoms. In responding to the client, the nurse recognizes what pathology as the cause of her symptoms?

Destruction of joint cartilage.

The nurse is triaging victims of a tornado at an emergency shelter. An adult woman who has been wandering and crying comes to the nurse. What action should the nurse take? a- Check the client's temperature, blood sugar, and urine output. b- Transport the client for laboratory client for laboratory test and electrocardiogram (EKG) c- Delegate care of the crying client to an unlicensed assistant d- Send the client to the shelter's nutrient center to obtain water and food.

Delegate care of the crying client to an unlicensed assistant

When administering an immunization in an adult client, the nurse palpates and administer the injection one inch below the acromion process into the center of the muscle mass. The nurse should document that the vaccine was administered at what site? a. Rectus femenis b. Ventrogluteous c. Vastus lateralis d. Deltoid

Deltoid

A female client is admitted for diabetic crisis resulting from inadequate dietary practices. After stabilization, the nurse talks to the client about her prescribed diet. What client characteristic is most import for successful adherence to the diabetic diet?

Demonstrates willingness to adhere to the diet consistently

The nurse is caring for a client who is experiencing a tonic-clonic seizure. Which actions should the nurse implement? (Select all that apply)

Ease the client to the floor Loosen restrictive clothing Note the duration of the seizure

A young couple who has been unsuccessful in conceiving a child for over a year is seen in the family planning clinic. During an initial visit, which intervention is most important for the nurse to implement? a. Determine current sexual practice b. Prepare a female client for an ultrasound c. Request a sperm sample for ovulation d. Evaluate hormone levels on both clients

Determine current sexual practice Rationale: First a history should be obtained including practices that might be related to the infertility, such as douching, daily ejaculation or the male partner's exposure to heat, such as frequent sauna or work environment which can decrease sperm production (A B or C) may be indicated after a complete assessment is obtained.

A newly hired home health care nurse is planning the initial visit to an adult client who has had multiple sclerosis (MS) for the past 20 years and is currently bed-bound and is lifted by a hoist. And unlicensed caregiver provides care 8 hours/ daily, 5 days/week. During the initial visit to this client, which intervention is most important to the nurse to implement? a. Determine how the client is cared for when caregiver is not present. b. Develop a client needs assessment and review with the caregiver c. Evaluate the caregiver's ability to care for the client's needs. d. Review with the care giver the interventions provided each day.

Determine how the client is cared for when caregiver is not present

The nurse is preparing an intravenous (IV) fluid infusion using an IV pump. Within 30 seconds of turning on the machine, the pump's alarm beeps "occlusion". What action should the nurse implement first? a. Flush the vein with 3 ml of sterile normal saline. b. Assess the IV catheter insertion site for infiltration. c. Verify the threading of the tubing through the IV pump. d. Determine if the clamp on the IV tubing is released

Determine if the clamp on the IV tubing is released Rational: When the pump immediately beeps, it is often because the IV tubing clamp is occluding the flow, so the clamp should be checked first to ensure that it is open. If the alarm is not eliminated after the tubing clamp is released, flushing the IV site with saline is a common practice to clean the needle or to identify resistance due to another source. Local signs of infiltration may indicate the need to select another vein, but the pump's beeping-this early in the procedure is likely due to a mechanical problem. If beeping continues after verifying that the clamp is released the placement or threading of the tubing through the pump should be verified.

An adult male who lives alone is brought to the Emergency Department by his daughter who is unresponsive. Initial assessment indicated that the client has minimal respiratory effort, and his pupils are fixed and dilated. At the daughter's request, the client is intubated and... Which nursing intervention has the highest priority? a. Offer to notify the client's minister of his condition. b. Determine if the client has an executed living will c. Provide the family with information about palliative care d. Explore the possibility of organ donation with the family.

Determine if the client has an executed living will Rationale: Once the client is intubated and ventilated, emergency intervention should continue until patient t be stable check if the client has an executed living will.

A client with persistent low back pain has received a prescription for electronic stimulator (TENS) unit. After the nurse applies the electrodes and turns on the power, the client reports feeling a tingling sensation. How should the nurse respond? a. Determine if the sensation feels uncomfortable. b. Decrease the strength of the electrical signals. c. Remove electrodes and observe for skin redness. d. Check the amount of gel coating on the electrodes.

Determine if the sensation feels uncomfortable. Rational: electronic stimulators, such as a transelectrical nerve stimulator (TENS) unit, have been found to be effective in reducing low back pain by "closing the gate" to pain stimuli. A tingling sensation should be felt when the power is turned on, and the nurse should assess whether the sensation is too strong, causing discomfort or muscle twitching. Decreasing the electrical signal may be indicated if the sensation is too strong. Other options are not necessary because the tingling sensation is expected.

The nurse is caring for four clients...postoperative hemoglobin of 8.7 mg/dl; client C, newly admitted with potassium...an appendectomy who has a white blood cell count of 15,000mm3. What intervention...

Determine the availability of two units of packed cells in the blood bank for client B

A male client was transferred yesterday from the emergency department to the telemetry unit because he had ST depression and resolved chest pain. When his EKG monitor alarms for ventricular tachycardia (VT), what action should the nurse take first?a. Determine the client's responsiveness and respirations b. Bring the crash cart to the room to defibrillate the client. c. Immediately initiate chest compressions. d. Notify the emergency response team

Determine the client's responsiveness and respirations Rationale: Activities, such as brushing teeth, can mimic the waveform of VI, so first he client should be assessed (A) to determine if the alarm is accurate. The crash cart can be brought to the room by someone else and defibrillation (B) delivered as indicated by the client's rhythm. Based on as assessment of the client, CPR© as summoning the emergency response team (D) may be indicated.

An infant is placed in a radiant warmer immediately after birth. At one hour of age, the nurse finds the infant tachypneic, and hypotonic. What is the first action that the nurse should take? a. Notify the healthcare provider immediately b. Increase the temperature of the radiant warmer c. Assess the infant's heart rate. d. Determine the infant's blood sugar level.

Determine the infant's blood sugar level

A young boy who is in a chronic vegetative state and living at home is readmitted to the hospital with pneumonia and pressure ulcers. The mother insists that she is capable of caring for her son and which action should the nurse implement next? a. Report the incident to the local child protective services. b. Find a home health agency that specializes in brain injuries. c. Determine the mother's basic skill level in providing care. d. Consult the ethics committee to determine how to proceed.

Determine the mother's basic skill level in providing care Rational: Although the mother states she is a capable caregiver, the client is manifesting disuse syndrome complications, and the mother's skill in providing basic care should be determined. Further assessment is needed before implementing other nursing actions.

A mother runs into the emergency department with s toddler in her arms and tells the nurse that her child got into some cleaning products. The child smells of chemicals on hands, face, and on the front of the child's clothes. After ensuring the airway is patent, what action should the nurse implement first? a. Call poison control emergency number. b. Determine type of chemical exposure. c. Obtain equipment for gastric lavage. d. Assess child for altered sensorium.

Determine type of chemical exposure. Rational: once the type of chemical is determined, poison control should be called even if the chemical is unknown. If lavage is recommended by poison control, intubation and nasogastric tube may be needed as directed by poison control. Altered sensorium, such as lethargy, may occur if hydrocarbons are ingested

The nurse is presenting information about fetal development to a group of parents with...when discussing cephalocaudal fetal development, which information should the nurse gives the parents? a- set order in fetal development is expected b- Growth normally occurs within one organ at a time c- Development progress from head to rump d- Organ formation is directed by brain development

Development progress from head to rump

During a clinic visit, a client with a kidney transplant ask, "What will happen if chronic rejection develops?" which response is best for the nurse to provide

Dialysis would need to be resumed if chronic rejection becomes a reality

The nurse reviews the signs of hypoglycemia with the parents of a child with Type I diabetes mellitus. The parents correctly understand signs of hypoglycemia if they include which symptoms? a- Fruity breath odor b- Polyphagia c- Diaphoresis d- Polydipsia

Diaphoresis

The nurse and an unlicensed assistive personnel (UAP) are providing care for a client with a nasogastric tube (NGT) when the client begins to vomit. How should the nurse manage this situation?

Direct the UAP to measure the emesis while the nurse irrigates the NGT

The nurse is caring a client with NG tube. Which task can the nurse delegate to the UAP? a- Replace the NG tube as prescribed by the healthcare provider b- Secure the NG tube if it slides out of the client's nasal passage c- Disconnect the NG suction so the client can ambulate in the hallway. d- Reconnect the NG suction when the client returns form ambulating.

Disconnect the NG suction so the client can ambulate in the hallway

While the nurse is preparing a scheduled intravenous (IV) medication, the client states that the IV site hurts and refuses to allow the nurse to administer a flush to assess the site. Which intervention should the nurse implement?

Discontinue the painful IV after a new IV is inserted

During a routine clinic visit, an older female adult tells the nurse that she is concerned that the flu season is coming soon, but is reluctant to obtain the vaccination. What action should the nurse take first? a. Determine when the client last had an influenza vaccination. b. Discuss the concerns expressed by the client about the vaccination. c. Ask about any recent exposure to persons with the flu or other viruses. d. Review the informed consent form for the vaccination with the client.

Discuss the concerns expressed by the client about the vaccination Rationale: the nurse should first address the concerns identified by the client, before taking other actions, such as obtaining information about past vaccinations, exposure to the flu, or reviewing the informed consent form.

A client diagnosed with bipolar disorder is going home on a week-end pass. Which suggestions should give the client's family to help them prepare for the visit? 1. Discuss the importance of continuing the usual at-home activities 2. Encourage the family to plan daily activities to keep the client busy 3. Have friends and family visit the client at a welcome party. 4. Instruct family to monitor the client's choice of television programs.

Discuss the importance of continuing the usual at-home activities Rationale: Week-end pass are schedules to help the client ease back into the family's routine, so the client can back to normal activities.

The nurse assigned unlicensed assistive personnel (UAP) to apply antiembolism stockings to a client. The nurse and UAP enters the room, the nurse observes the stockings that were applying by the UAP. The UAP states that the client requested application of the stockings as seen on the picture, for increased comfort. What action should the nurse take? a. Ask the client if the stocking feel comfortable. b. Supervise the UAP in the removal of the stockings. c. Place a cover over the client's toes to keep them warm. d. Discussed effective use of the stockings with the client and UAP

Discussed effective use of the stockings with the client on UA Rational: antiembolism stockings are designed to fit securely and should be applied so that there are no bands of the fabric constricting venous return. The nurse should discuss the need for correct and effective use of the stockings with both the client and UAP to improve compliance. Other options do not correct the incorrect application of the stockings.

Which assessment is more important for the nurse to include in the daily plan of care for a client with a burned extremity?

Distal pulse intensity

The nurse is preparing to administer 1.6 ml of medication IM to a 4 month old infant. Which action should the nurse include? a. Select a 22 gauge 1 ½ inch (3.8 cm) needle for the intramuscular injection b. Administer into the deltoid muscle while the parent holds the infant securely c. Divide the medication into two injections with volumes under 1ml d. Use a quick dart-like motion to inject into the dorsogluteal site.

Divide the medication into two injection with volumes under 1ml Rationale: IM injection for children under 3 of age should not exceed 1ml. divide the dose into smaller volumes for injection in two different sites.

A woman just received the Rubella vaccine after a delivery of a normal new born, has two children at home, ages 13 months and 3 years. Which instruction is most important to provide to the client? a- Refrain from eating foods containing eggs for 24 hors b- Breast feeding is recommended to prevent ovulation. c- Do not get pregnant for at least 3 months. d- Avoid exposure to the sunlight for 36 hours.

Do not get pregnant for at least 3 months Rationale: The rubella vaccine can be harmful to an unborn child who is conceived within 3 month of the vaccination.

In assessing a client 48 hours following a fracture, the nurse observes ecchymosis at the fracture site, and recognizes that hematoma formation at the bone fragment site has occurred. What action should the nurse implement?

Document the extend of the bruising in the medical record

During the initial newborn assessment, the nurse finds that a newborn's heart rate is irregular. Which intervention should the nurse implement? a- Notify the pediatrician immediately. b- Teach the parents about congenital heart defects. c- Document the finding in the infant's record. d- Apply oxygen per nasal cannula at 3 L/min.

Document the finding in the infant's record

While completing an admission assessment for a client with unstable angina, which closed questions should the nurse ask about the client's pain?

Does your pain occur when walking short distances?

To prevent infection by auto contamination during the acute phase of recovery from multiple burns, which intervention is most important for the nurse to implement? a. Dress each wound separately. b. Avoid sharing equipment between multiple clients. c. Use gown, mask and gloves with dressing change. d. Implement protective isolation.

Dress each wound separately. Rational: each wound should be dressed separately using a new pair of sterile glove to avoid auto contamination (the transfer of microorganisms' form one infected wound to a non-infected wound). The other choices do not prevent auto contamination.

The nurse assesses a 78-year-old male client who has left sides heart failure. Which symptoms would the nurse expect this client to exhibit? a- Dyspnea, cough, and fatigue. b- Hepatomegaly and distended neck veins c- Pain over the pericardium and friction rub. d- Narrowing pulse pressure and distant heart sounds.

Dyspnea, cough, and fatigue.

A client in her first trimester of pregnancy complains of nausea. Which complementary therapy should the nurse recommend? a. Eat food high in garlic with the evening meal b. Drink chamomile tea at breakfast and in the evening. c. Increase cocoa in the diet and drink before bedtime d. Join a yoga class that meets at least weekly

Drink chamomile tea at breakfast and in the evening. Rationale: Chamomile tea is used to aid with digestion and is in fact sometimes used for indigestion. C should not be used by breastfeeding woman or at night when trying to go to sleep. D is for improve circulation, stimulate the internal organs, stretch the body, restore....

A client with chronic alcoholism is admitted with a decreased serum magnesium level. Which snack option should the nurse recommend to this client? a. Cheddar cheese and crackers. b. Carrot and celery sticks. c. Beef bologna sausage slices. d. Dry roasted almonds.

Dry roasted almonds. Rational: alcoholism promotes inadequate food intake and gastrointestinal loss of magnesium include green leafy vegetables and nuts and seeds. Other snacks listed provide much lower amounts of magnesium per serving.

A male client with cancer, who is receiving antineoplastic drugs, is admitted to the... what findings is most often manifest this condition? a. Ecchymosis and hematemesis b. Weight loss and alopecia c. Weakness and activity intolerance d. Sore throat and fever

Ecchymosis and hematemesis

A client is admitted to the surgical unit with symptoms of a possible intestinal obstruction. When preparing to insert a nasogastric (NG) tube, which intervention should the nurse implement?

Elevate the head of the bed 60 to 90 degrees

While a child is hospitalized with acute glomerulonephritis, the parents ask why blood pressure readings are taken so often. Which response by the nurse is most accurate?

Elevated blood pressure must be anticipated and identified quickly

The nurse administers an oral antiviral to a client with shingles. Which finding is most important for the nurse report to the health care provider? a. Decreased white blood cell count b. Pruritus and muscle aches c. Elevated liver function tests d. Vomiting and diarrhea

Elevated liver function tests

A female client is extremely anxious after being informed that her mammogram was abnormal and needs to be repeated. Client is tearful and tells the nurse her mother died of breast cancer. What action should the nurse take? a. Provide the client with information about treatment options for breast cancer. b. Reassure the client that the final diagnosis has not been made. c. Encourage the client to continue expressing her fears and concerns. d. Suggest to the client that she seek a second opinion.

Encourage the client to continue expressing her fears and concerns. Rational: the nurse should show support for the client by encouraging her to continue expressing her concerns. A diagnosis has not yet been made, so it is too early to discuss treatment options. Other options dismiss the client's feelings or are premature given that the diagnosis is not yet made.

An adult woman who is seen in the clinic with possible neuropathic pain of the right leg rates her pain as a 7 on a 10 point scale. What action should the nurse take? a. Elevate the foot and leg on two pillows b. Measure the client's capillary glucose c. Ask the client to dorsiflex the right foot. d. Encourage the client to describe the pain.

Encourage the client to describe the pain. Rationale: Neuropathic pain is caused by damage within the nervous system. Description of the pain such as burning or numbness helps identify the pain as neuropathic, allowing appropriate treatment to be initiated. Elevation is to unlikely to impact the pain. Persons with diabetes mellitus may develop peripheral neuropathy, nut there is no immediate need to measure this client's capillary glucose. (C) is not a useful intervention in assessing or managing neuropathic pain.

A male client tells the nurse that he is concerned that he may have a stomach ulcer, because he is experiencing heartburn and a dull growing pain that is relieved when he eats. What is the best response by the nurse?

Encourage the client to obtain a complete physical exam since these symptoms are consistent with an ulcer

One day following a total knee replacement, a male client tells the nurse that he is unable to transfer because it is too painful. What action should the nurse implement? a- Encourage use of analgesics before position change b- Assess anxiety about transferring to commode chair c- Assist client during transfer on the first two days d- Review use of assistive devices for weight bearing.

Encourage use of analgesics before position change

A client is being treated for syndrome of inappropriate antidiuretic hormone (SIADH). On examination, the client has a weight gain of 4.4 lbs (2 kg) in 24 hours and an elev ated blood pressure. Which intervention should the nurse implement first?

Ensure client takes a diuretic q AM

A nurse is planning to teach infant care and preventive measures for sudden infant death syndrome (SIDS) to a group of new parents. What information is most important for the nurse to include? a. Swaddle the infant in a blanket for sleeping b. Place the infant in a prone position whenever possible c. Prop that the infant's crib matter is firm d. Ensure that the infant's crib mattress is firm.

Ensure that the infant's crib mattress is firm Rationale: Sudden infant death syndrome is the unexplained death of infants under the age of one year. Parents should be educated about the methods to reduce the risk of SIDS, which include use of a firm crib mattress, maternal smoking cessation before and after pregnancy, avoidance of pillows in the crib, and placing the infant in the supine position. (Back to Sleep Campaign)

The nurse plans to use an electronic digital scale to weight a client who is able to stand. Which intervention should the nurse implement to ensure that measurement of the client's weight is accurate? a. Ask the client to remove shoes before stepping on the scale b. Ensure that the scale is calibrated before a weight is obtained. c. Slide the balancing weights until the scale is at zero. d. Compare client's weight at various time of the day.

Ensure that the scale is calibrated before a weight is obtained

he nurse manager is conducting an in-services education program on the fire evacuation of the newborn recovery. What intervention should the nurse manager disseminate to the staff? a. Place infants on a blanket for evacuation via stairwell b. Secure three infants on a stretcher for transport c. Evacuate each infant with mother via wheelchair d. Use the bassinet in evacuate two infants at a time.

Evacuate each infant with mother via wheelchair Rationale: Rooming-in and newborn babies are counted with their mothers. To exposure safety and accountability during the evacuations newborns should be evacuated with their mother in a wheelchair while maneuver with fire extinguisher are performed (PASS)...

A client with rapid respirations and audible rhonchi is admitted to the intensive care unit because of a pulmonary embolism (PE). Low-flow oxygen by nasal cannula and weight based heparin protocol is initiated. Which intervention is most important for the nurse to include in this client's plan of care?

Evaluate daily blood clotting factors.

A client who is admitted to the intensive care unit with syndrome of inappropriate antidiuretic hormone (SIADH) has developed osmotic demyelination. Which intervention should the nurse implement first? a. Patch one eye. b. Reorient often. c. Range of motion. d. Evaluate swallow

Evaluate swallow Rational: Osmotic demyelination, also known as central pontine myelinolysis, is nerve damage caused by the destruction of the myelin sheath covering nerve cells in the brainstem. The most common cause is a rapid, drastic change in sodium levels when a client is being treated for hyponatremia, a common occurrence in SIADH. Difficulty swallowing due to brainstem nerve damage should be care but determining the client's risk for aspiration is most important.

The nurse ends the assessment of a client by performing a mental status exam. Which statement correctly describes the purpose of the mental status exam? a. Determine the client's level of emotional functioning' b. Assess functional ability of the primary support system. c. Evaluate the client's mood, cognition and orientation. d. Review the client's pattern of adaptive coping skill

Evaluate the client's mood, cognition and orientation. Rational: the mental status exam assesses the client for abnormalities in cognitive functioning; potential thought processes, mood and reasoning, the other options listed are all components of the client's psychosocial assessment.

The nurse enters a client's room to administer scheduled daily medications and observes the client leaning forward and using pursed lip breathing. Which action is most important for the nurse to implement first?

Evaluate the oxygen saturation

A young adult male was admitted 36 hours ago for a head injury that occurred as the result of a motorcycle accident. In the last 4 hours, his urine output has increased to over 200 ml/H. Before reporting the finding to the healthcare provider, which intervention should the nurse implement? a- Evaluate the urine osmolality and the serum osmolality values. b- Obtain blood pressure and assess for dependent edema c- Measure oral secretions suctioned during last hours d- Obtain capillary blood samples q2 hours for glucose monitoring.

Evaluate the urine osmolality and the serum osmolality values. Rationale: With a known head injury, sudden inadequate secretion of antidiuretic hormone (ADH) can cause excessive output of diluted urine. Evaluating laboratory results should de determined to identify findings of neurogenic diabetes insipidus (DI0, such as low urine osmolarity and normal serum osmolarity (A) prior to notify the healthcare provider so that these finding can be included in the report. Massive diuresis, dehydration, and thirst manifest hypotension, irregular tachycardia, decrease skin turgor, but B or C are not related to DI.

Progressive kyphoscoliosis leading to respiratory distress is evident in a client with muscul...Which finding warrants immediate intervention by the nurse? a. Extremity muscle weakness b. Bilateral eyelid drooping c. Inability to swallow pills d. Evidence of hypoventilation

Evidence of hypoventilation Rationale: Hypoventilation indicates respiratory muscle weakness, and if the client is unable to breath... respiratory distress and life-threatening.

The nurse ask the parent to stay during the examination of a male toddler's genital area. Which intervention should the nurse implement? a. Examine the genitalia as the last part of the total exam. b. Use soothing statements to facilitate cooperation c. Allow the child to keep underpants on to examine genitalia d. Work slowly and methodically so not to stress the child

Examine the genitalia as the last part of the total exam. Rationale: Examination of a child's genitalia is particularly stressful to toddles, so this assessment is best left as the last part of the examination. B are best done by a parent, not the nurse. The genitals must be completely visualized and sometimes palpates underwear for a brief period of.

The nurse is developing a plan of care for a middle-aged woman who is diagnosed with type 2 diabetes mellitus (DM). To lower her blood glucose and increase her serum high-density lipoprotein (HDL) levels, which instruction is most important for the nurse to provide? a- Exercise at least three times weekly b- Monitor blood glucose levels daily c- Limit intake of foods high in saturated fat d- Learn to read all food product labels

Exercise at least three times weekly

A 2-year-old girl is brought to the clinic for a routine assessment and all findings are within the normal limits. However, the mom expresses concern over her daughter's protruding abdomen and tells the nurse that she is worry that her child is becoming overweight. How should the nurse respond to the mother's comment? a- Tell the mother to keep a twenty- four-hour food diary for the child. b- Explain that a protruding abdomen is typical for toddlers. c- Discuss way to increase the child's daily activity level d- Ask the mother is she has weight problems when she was a child.

Explain that a protruding abdomen is typical for toddler

A young adult woman visits the clinic and learns that she is positive for BRCA1 gene mutation and asks the nurse what to expect next. How should the nurse respond? a. Explain that counseling will be provided to give her information about her cancer risk. b. Gather additional information about the client's family history for all types of cancer. c. Offer assurance that there are a variety of effective treatments for breast cancer. d. Provide information about survival rates for women who have this genetic mutation.

Explain that counseling will be provided to give her information about her cancer risk Rational: BRACA1or BRACA2 genetic mutation indicates an increased risk for developing breast or ovarian cancer and genetic counseling should be provided to explain the increased risk (A)to the client along with options for increased screening or preventative measures. (B) Is completed by the genetic counselor before the client undergoes genetic testing. a positive BRACA1test is not an indicator of the presence of cancer and (C and D) are not appropriate responses prior to genetic counseling.

The husband of an older woman, diagnosed with pernicious anemia, calls the clinic to report that his wife still has memory loss and some confusion since she received the first dose of nasal cyanocobalamin two days ago. He tells the nurse that he is worried that she might be getting Alzheimer's disease. What action should the nurse take?

Explain that memory loss and confusion are common with vitamin B12 deficiency

The nurse observes an unlicensed assistive personnel (UAP) using an alcohol-based clean...tray to the room. The UAP rub both hands thoroughly for 2 minutes while standing at the...should the nurse take? a. Encourage the UAP to remain in the client's room, until completed b. Explain that the hand rub can be completed in less than 2 minutes. c. Inform the UAP that handwashing helps to promote better asepsis. d. Determine why the UAP was not wearing gloves in the client's room

Explain that the hand rub can be completed in less than 2 minute

A female client on the mental health unit frequently asks the nurse when she can be discharged. Then, becoming more anxious, she begins to pace the hallway. What intervention should the nurse implement first?

Explore the client's reasons for wanting to be discharged.

Which assessment finding indicates to the nurse a client's readiness for pulmonary function tests? a- Expresses an understanding of the procedure. b- NPO for 6 hrs. c- No known drug allergies d- Intravenous access intact.

Expresses an understanding of the procedure

Following a motor vehicle collision (MCV), a male adult in severe pain is brought to the emergency department via ambulance. His injured left leg is edematous, ecchymotic around the impact of injury on the thigh, and shorter than his right leg. Based on these findings, the client is at greatest risk for which complication? a. Arterial ischemia b. Tissue necrosis c. Fat embolism d. Nerve damage

Fat embolism

The nurse is assessing a primigravida a 39-weeks gestation during a weekly prenatal visit. Which finding is most important for the nurse to report to the healthcare provider?

Fetal heart rate of 200 beats/minute

The nurse caring for a client with dysphagia is attempting to insert an NG tube, but the client will not swallow and is not gagging. What action should the nurse implement to facilitate the NGT passage into the esophagus? a. Push the NGT beyond the oropharynx gently yet swiftly. b. Offer the client sips of water or ice and coax to swallow c. Elevate the bed 90 degree and hyperextend the head. d. Flex the client's head with chin to the chest and insert.

Flex the client's head with chin to the chest and insert.

The nurse discovers that an elderly client with no history of cardiac or renal disease has an elevated serum magnesium level. To further investigate the cause of this electrolyte imbalance, what information is most important for the nurse to obtain from the client's medical history

Frequency of laxative use for chronic constipation

The nurse discovers that an elderly client with no history of cardiac or renal disease has an elevated serum magnesium level. To further investigate the cause of this electrolyte imbalance, what information is most important for the nurse to obtain from the client's medical history? a. Genetically inherited disorders of family members b. Length and frequency of the client's tobacco use. c. Ingestion of selfish or fish oil capsules daily. d. Frequency of laxative use for chronic constipation

Frequency of laxative use for chronic constipation Rationale: Elderly clients are at risk of developing hypermagnesemia as a result of chronic laxative abuse.

A male client who was admitted with an acute myocardial infarction receives a cardiac diet with sodium restriction and complains that his hamburger is flavorless. Which condiment should the nurse offer?

Fresh horseradish

When conducting diet teaching for a client who was diagnosed with a myocardial infarction, which snack foods should the nurse encourage the client to eat? (Select all that apply).

Fresh turkey slices and berries raw unsalted almonds and apples

A young adult female with chronic kidney disease (CKD) due to recurring pyelonephritis is hospitalized with basilar crackles and peripheral edema. She is complaining of severe nausea and the cardiac monitor indicates sinus tachycardia with frequent premature ventricular contraction. Her blood pressure is 200 /110 mm Hg, and her temperature is 101 F which PRN medication should the nurse administers first?

Furosemide

When finding a client sitting on the floor, the nurse calls for help from the unlicensed assistive personnel (UAP). Which task should the nurse ask the UAP to do? a- Check for any abrasions or bruises. b- Help the client to stand. c- Get a blood pressure cuff. d- Report the fall to the nurse-manager.

Get a blood pressure cuff.

A client's telemetry monitor indicates ventricular fibrillation (VF). After delivering one counter shock, the nurse resumes chest compression, after another minute of compression , the client's rhythm converts to supraventricular tachycardia (SVT) on the monitor, at this point , what is the priority intervention for the nurse?

Give IV dose of adenosine rapidly over 1-2 seconds.

During a staff meeting, a nurse verbally attacks the nurse manager conducting the meeting, stating, "you always let your favorites have holidays off give then easier assignments. You are unfair and prejudiced" how should the nurse-manager respond?

Give me specific examples to support your statements.

The healthcare provider prescribes carboprost tromethamine (Hemabate) 250 mcg IM for a multigravida postpartum client who is experiencing heavy, bright red vaginal bleeding. Prior to administering this medication, which interventions should the RN implement?

Give the prescribed antiemetic.

A middle-aged woman, diagnosed with Graves' disease, asks the nurse about this condition. Which etiological pathology should the nurse include in the teaching plan about hyperthyroidism? (Select all that apply.)

Graves' disease, an autoimmune condition, affects thyroid stimulating hormone receptors. T3 and T4 hormone levels are increased Large protruding eyeballs are a sign of hyperthyroid function

A female client presents in the Emergency Department and tells the nurse that she was raped last night. Which question is most important for the nurse to ask? a. Does she know the person who raped her? b. Has she taken a bath since the raped occurred? c. Is the place where she lived a safe place? d. Did she report the rape to the police Department?

Has she taken a bath since the raped occurred? RATIONALE: The priority action is collected the forensic evidence, so asking if the has taken a bath since the rape occurred is the most important information to obtain. Other options are used by law enforcement to determine the perpetrator and are not vital in providing client care at this time.

Artificial rupture of the membrane of a laboring reveals meconium-stained fluid, what is... the priority? 545. a- Clean the perineal are to prevent infection b- Assess the mother's blood pressure to check for signs of preeclampsia c- Assess the mother temperature to check for development of sepsis. d- Have a meconium aspirator available at delivery.

Have a meconium aspirator available at delivery

An older woman who has difficulty hearing is being discharged from day surgery following a cataract extraction & lens implantation. Which intervention is most important for the nurse to implement to ensure the client's compliance with self-care? a- Speak clearly and face the clients for lip reading b- Provide written instructions for eyes drop administration c- Ensure that someone will stay with the client for 24 hours. d- Have the client vocalize the instructions provided.

Have the client vocalize the instructions provided. Rationale: A client with both hearing and visual sensory deficit should be repeat the instruction provided so the nurse needs to be sure the clients understand the self-care instructions.

An elderly male client is admitted to the urology unit with acute renal failure due to a postrenal obstruction. Which questions best assists the nurse in obtaining relevant historical data?

Have you had any difficulty in starting your urinary stream"

A client with Addison's crisis is admitted for treatment with adrenal cortical supplementation. Based on the client's admitting diagnosis, which findings require immediate action by the nurse? (Select all that apply)

Headache and tremors Postural hypotension Pallor and diaphoresis Irregular heart beat

The nurse receives a newborn within the first minutes after a vaginal delivery and intervenes to establish adequate respirations. What priority issue should the nurse address to ensure the newborn's survival? a. Hypoglycemia b. Fluid balance c. Heat loss d. Bleeding tendencies

Heat loss Rationale: Adequate thermoregulation is the nurse next priority. The newborn is at risk for significant heat loss due to a large surface area exposed to the environment, a thin layer of subcutaneous fat, and distribution of brow fat. Heat loss increases the neonate's metabolic pathway's utilization of oxygen and glucose.

In assessing a client at 34-weeks' gestation, the nurse notes that she has a slightly elevated total T4 with a slightly enlarged thyroid, a hematocrit of 28%, a heart rate of 92 beats per minute, and a systolic murmur. Which finding requires follow-up? a. Elevated thyroid hormone level. b. Hematocrit of 28%. c. Heart rate of 92 beats per minute. d. Systolic murmur.

Hematocrit of 28% Rational: although physiologic anemia is expected in pregnancy, a hematocrit of 28% is below pregnant norms and could signify iron-deficiency anemia. Other options are normal finding pregnancy

A client with arthritis has been receiving treatment with naproxen and now reports ongoing stomach pain, increasing weakness, and fatigue. Which laboratory test should the nurse monitor? a. Sed rate (ESR) b. Hemoglobin c. Calcium d. Osmolality.

Hemoglobin Rational: naproxen can cause gastric bleeding, so the nurse should monitor the client's hemoglobin to assess for possible bleeding. Other options are not likely to be affected by the used of naproxen and are not related to the client's current symptoms.

During a well-baby, 6-month visit, a mother tells the nurse that her infant has had fewer ear infections than her 10-year-old daughter. The nurse should explain that which vaccine is likely to have made the difference in the siblings' incidence of otitis media? a. Varicella Virus Vaccine Live b. Hemophilic Influenza Type B (HiB) vaccine c. Pneumococcal vaccine d. Palivizumab vaccine for RSV

Hemophilic Influenza Type B (HiB) vaccine

A 75-year-old female client is admitted to the orthopedic unit following an open reduction and internal fixation of a hip fracture. On the second postoperative day, the client becomes confused and repeatedly asks the nurse she is. What information for the nurse to obtain?

History of alcohol use,

The nurse plans to administer a schedule dose of metoprolol (Toprol SR) at 0900 to a client with hypertension. At 0800, the nurse notes that client's telemetry pattern shows a second degree heart block with a ventricular rate of 50. What action should the nurse take? a- Administer the Tropol immediately and monitor the client until the heart rate increases. b- Provide the dose of Tropol as scheduled and assign a UAP to monitor the client's BP q30 minutes. c- Give the Tropol as scheduled if the client's systolic blood pressure reading is greater than 180. d- Hold the scheduled dose of Tropol and notify the healthcare provider of the telemetry pattern.

Hold the scheduled dose of Tropol and notify the healthcare provider of the telemetry pattern. Rationale: Beta blockers such as metoprolol (Tropol SR) are contraindicated in clients with second or third-degree heart block because they decrease the heart rate. Therefore, the nurse should hold the medication.

When obtaining a rectal temperature with an electronic thermometer, which action is most important for the nurse to perform? a. Hold the thermometer in place. b. Place the disposable pad under buttocks c. Instruct the client to breathe deeply d. Return the probe to the charger.

Hold the thermometer in place

A client with severe full-thickness burns is scheduled for an allografting procedure. Which information should the nurse provide the client? a. The donor site will be painless a few days after the surgery b. Allografts are made from human and nonhuman material sources. c. Human sources graft require monitor for signs of graft injection d. Human source grafts require monitoring for signs of graft rejection.

Human source grafts require monitoring for signs of graft rejection Rationale: Allograft is a graft created from the client's own skin, which is called harvest site. All types of grafts, from human and nonhuman sources should be monitor for signs of rejection. Graft site are painful. (A). Allografts are obtained from the client, which is a human source (B). scaring does occur under the graft (D)

A client with Addison's disease becomes weak, confused, and dehydrated following the onset of an acute viral infection. The client's laboratory values include; sodium 129 mEq/l (129mmol/l SI), glucose 54 mg/dl (2.97mmol/l SI) and potassium 5.3 mmol/l SI). When reporting the findings to the HCP, the nurse anticipates a prescription for which intravenous medications?

Hydrocortisone

A male client is admitted with burns to his face and neck. Which position should the nurse place the client to prevent contract? a. Flexed with the chin toward the chest. b. Hyperextended with neck supported by a rolled towel. c. Side-lying with the head on a pillow d. Prone with face supported by an inflated rubber ring.

Hyperextended with neck supported by a rolled towel.

A nurse is caring for a client with Diabetes Insipidus. Which assessment finding warrants immediate intervention by the nurse? a- Hypernatremia b- Excessive thirst c- Elevated heart rate d- Poor skin turgor

Hypernatremia

A male client who was hit by a car while dodging through traffic is admitted to the emergency department with intracranial pressure (ICP). A computerized tomography (CT) scan reveals an intracranial bleed. After evacuation of hematoma, postoperative prescription include: intubation with controlled mechanical ventilation to PaCO2...what is the pathophysiological basis for this ventilator settings? a- Hypoxemia reduces ICP. b- Hypocapnea reduces ICP. c- Hyperventilation reduces need for temperature control. d- Controlled ventilation reduces need for oxygen to brain.

Hypocapnea reduces ICP

A male client arrives at the clinic with a severe sunburn and explains that he did not use sun screen because it was an overcast day. Large blisters are noted over his back and chest and his shirt is soaked with serosanguinous fluid. Which assessment finding warrants immediate intervention by the nurse? a. Hypotension. b. Fever and chills c. Dizziness d. Headache

Hypotension

The nurse is assessing a 4-year-old boy admitted to the hospital with the diagnosis of possible nephrotic syndrome. Which statement by the parents indicates a likely correlation to the child's diagnosis? a- I couldn't get my son's socks and shoes on this morning" b- My son has been on amoxicillin/clavulanate for 2 days for an ear infection c- My son has had a red rash over his entire body for the past 4 days. d- I couldn't get my son calm down and sleep last night.

I couldn't get my son's socks and shoes on this morning"

An adolescent, whose mother recently died, comes to the school nurse complain headache. Which statement made by the students should warrant further explanation nurse?

I miss Mom and would like to go see her'".

The nurse is evaluating the health teaching of a female client with condyloma acuminate. Which statement by the client indicates that teaching has been effective? a. These warts are caused by a fungus b. Early treatment is very effective c. I need to have regular pap smears d. I will clean my hot tub better

I need to have regular pap smears

A newly graduated female staff nurse approaches the nurse manager and request reassignment to another client because a male client is asking her for a date and making suggestive comments. Which response is best for the nurse manager to provide? a. I have to call the supervisor o get someone else to transfer to this unit to care for him. b. I know you are good nurse and can handle this client in a professional manner. c. I'll talked to the client about his sexual harassment and I'll insist that he stop it immediately. d. I'll change your assignment, but let's talk about you a nurse should respond to this kind of client.

I'll change your assignment, but let's talk about you a nurse should respond to this kind of client.

An older male client with a history of diabetes mellitus, chronic gout, and osteoarthritis comes to the clinic with a bag of medication bottles. Which intervention should the nurse implement first?

Identify pills in the bag

The nurse walks into a client's room and notices bright red blood on the sheets and on the floor by the IV pole. Which action should the nurse take first? a. Clean up the spilled blood to reduce infection transmission. b. Notify the healthcare provider that the client appears to be bleeding. c. Apply direct pressure to the client's IV site. d. Identify the source and amount of bleeding.

Identify the source and amount of bleeding.

An adult client is exhibit the maniac stage of bipolar disorder is admitted to the psychiatric unit. The client has lost 10 pounds in the last two weeks and has no bathed in a week "I'm trying to start a new business and "I'm too busy to eat". The client is oriented to time, place, person but not situation. Which nursing problem has the greatest priority? a. Hygiene-self-care deficit b. Imbalance nutrition c. Disturbed sleep pattern d. Self-neglect

Imbalance nutrition Rationale: The client's nutritional status has the highest priority at this time, and finger foods are often provided, so the client who is on the maniac phase of bipolar disease can receive adequate nutrition. Other options are nursing problems that should also be addresses with the client's plan of care, but at this stage in the client's treatment, adequate nutrition is a priority

The nurse is reviewing a client's electrocardiogram and determines the PR interval (PRI) is prolonged. What does this finding indicate? a- Initiation of the impulses from a location outside the SA node b- Inability of the SA node to initiate an impulse at the normal rate c- Increased conduction time from the SA node to the AV junction Interference with the conduction through one or both ventricles

Inability of the SA node to initiate an impulse at the normal rate Rationale: A prolonged PRI reflects an increased amount of time for an impulse to travel from the SA node through the AV node and is characteristic of a first-degree heart block.

The nurse is caring for a client with hypovolemic shock who is receiving two units of packed red blood cells (RBCs) through a large bore peripheral IV. What action promotes maintenance of the client's cardiopulmonary stability during the blood transfusion? a- Increase the oxygen flow via nasal cannula if dyspnea is present. b- Place in a Trendelenburg position to increase cerebral blood flow c- Monitor capillary glucose measurements hourly during transfusion. d- Encourage increased intake of oral fluid to improve skin turgor.

Increase the oxygen flow via nasal cannula if dyspnea is present.

A 46-year-old male client who had a myocardial infarction 24-hours ago comes to the nurse's station fully dressed and wanting to go home. He tells the nurse that he is feeling much better at this time. Based on this behavior, which nursing problem should the nurse formulate?

Ineffective coping related to denial

The psychiatric nurse is talking to a newly admitted client when a male client diagnosed with antisocial behavior intrudes on the conversation and tells the nurse, "I have to talk to you right now! It is very important!" how should the nurse respond to this client? a. Put his behavior on extinction and continue talking with the newly admitted. b. Inform him that the nurse is busy admitting a new client and will talk to him later. c. Encourage him to go to the nurse's station and talk with another nurse. d. Introduce him to the newly admitted client and ask him to him to join in the conversation.

Inform him that the nurse is busy admitting a new client and will talk to him later. Rational: the psychiatric nurse must set limits with antisocial behavior so that appropriate behavior is demonstrated. Interrupting a conversation is rude and inappropriate, so telling the client that they can talk later is the best course of action. Other options may cause the client to become angry and they do not address the client's behavior. The nurse should not involve this client with newly admitted client's admission procedure.

A male client is admitted with a bowel obstruction and intractable vomiting for the last several hours despite the use of antiemetics. Which intervention should the nurse implement first? pH 7.50; PaCo2 42; HCO3 33; pO2 92

Infuse 0.9 % sodium chloride 500 ml bolus

Sublingual nitroglycerin is administered to a male client with unstable angina who complains of crushing chest pain. Five minutes later the client becomes nauseated and his bloods pressure drops to 60/40. Which intervention should the nurse implement?

Infuse a rapid IV normal saline bolus

A young adult male is admitted to the emergency department with diabetic ketoacidosis (DKA). His pH is 7.25, HCO3 is 12 mEq/L or 12 mmol/L (SI), and blood glucose is 310 mg/dl or 17.2 mmol/L (SI). Which action should the nurse implement? a- Infuse sodium chloride 0.9% (normal saline) b- Prepare an emergency dose of glucagon c- Determine the last time the client ate d- Check urine for ketone bodies with a dipstick

Infuse sodium chloride 0.9% (normal saline) Rationale DKA an increase in glucose and ketone bodies, result in hyperosmolar dehydration, so is necessary to restore fluid balance.

What action should the school nurse implement to provide secondary prevention to a school-age children? a. Collaborate with a science teacher to prepare a health lesson b. Prepare a presentation on how to prevent the spread of lice c. Initiate a hearing and vision screening program for first-graders d. Observe a person with type 1 diabetes self-administer a dose of insulin

Initiate a hearing and vision screening program for first-grader

A mother brings her 3-week-old son to the clinic because he is vomiting "all the time." In performing a physical assessment, the nurse notes that the infant has poor skin turgor, has lost 20% of his birth weight, and has a small palpable oval-shaped mass in his abdomen. What intervention should the nurse implement first?

Initiate a prescribed IV for parental fluid

After placement of a left subclavian central venous catheter (CVC), the nurse receives report of the x-ray findings that indicate the CVC tip is in the client's superior vena cava. Which action should the nurse implement? a. Initiate intravenous fluid as prescribed b. Notify the HCP of the need to reposition the catheter c. Remove the catheter and apply direct pressure for 5 minutes. d. Secure the catheter using aseptic technique

Initiate intravenous fluid as prescribed Rationale: Venous blood return to the heart and drains from the subclavian vein into the superior vena cava. The X-ray findings indicate proper placement of the CVC, so prescribed intravenous fluid can be started. A and B are not indicated at this time. The catheter should be secure immediate following insertion (C)

An adult is admitted to the emergency department following ingestion of a bottle of antidepressants secondary to chronic paint. A nasogastric tube and a left subclavian venous catheter are placed. The nurse auscultates audible breath sounds on the right side, faint sounds procedure should the nurse prepare for first?

Insertion of a left- sided chest tube.

An adult male who was admitted two days ago following a cerebrovascular accident (CVA) is confused and experiencing left-side weakness. He has tried to get out of bed several times, but is unable to ambulate without assistance. Which intervention is most important for the nurse to implement? a- Ask a family member to sit with the client b- Apply bilateral soft wrist restraints c- Assign staff to check client q15 minutes d- Install a bed exit safety monitoring device

Install a bed exit safety monitoring device

A male infant born at 28-weeks gestation at an outlying hospital is being prepared for transport to a respiration are 92 breaths/minute and his heart rate is 156 beats/minute. Which drug is the transport administration to this infant? a- Give ampicillin 25 mg/kg slow IV push b- Deliver 1:10,000 epinephrine 0.1 ml/kg per endotracheal tube c- Administer digoxin 20 mcg/kg IV d- Instill beractant 100 mg/kg in endotracheal tube.

Instill beractant 100 mg/kg in endotracheal tube

A client with pneumonia has arterial blood gases levels at: PH 7.33; PaCO2 49 mm/hg; HCO3 25 mEq/L; PaO2 95. What intervention should the nurse implement based on these results

Institute coughing and deep breathing protocols

During a left femoral artery aortogram, the healthcare provider inserts an arterial sheath and initiate...through the sheath to dissolve an occluded artery. Which interventions should the nurse implement?

Instruct the client to keep the left leg straight Observe the insertion site for a hematoma Circle first noted drainage on the dressing

An adolescent's mother calls the clinic because the teen is having recurrent vomiting and...Combative in the last 2 days. The mother states that the teen takes vitamins, calcium, mag...With aspirin. Which nursing intervention has highest priority? a. Advise the mother to withhold all medications by mouth. b. Instruct the mother to take the teen to the emergency room c. Recommend that the teen withhold food and fluids for 2 hours d. Suggest that the adolescent breath slowly and deeply.

Instruct the mother to take the teen to the emergency room

The charge nurse observes a new nurse preparing to insert an intravenous (IV) catheter. The new nurse has gathered supplies, including intravenous catheters, an intravenous insertion kit, and a 4x4 sterile gauze dressing to cover and secure the insertion site. What action should the charge nurse take?

Instruct the nurse to use a transparent dressing over the site

During the intraoperative phase of care, the circulating nurse observes that the client is not adequately client's privacy. What is the best initial nursing action for the nurse to implement? a- Document the observation in the client's medical record b- Instruct the scrub nurse to re-drape the client c- Ensure that the client in unaware of the surrounding. d- Consult with operating room manager.

Instruct the scrub nurse to re-drape the client

A child newly diagnosed with sickle cell anemia (SCA) is being discharged from the hospital. Which information is most important for the nurse to provide the parents prior to discharge? a. Instructions about how much fluid the child should drink daily b. information about non-pharmaceutical pain reliever measures c. Referral for social services for the child and family d. Signs of addiction to opioid and medications

Instructions about how much fluid the child should drink daily Rationale: It is essential that the child and family understands the importance of adequate hydration in preventing the stasis-thrombosis-ischemia cycle of a crisis that has a specific plan for hydration is developed so that a crisis can be delayed. Other choices listed are not the most important topics to include in the discharge teaching.

The nurses observes that a postoperative client with a continuous bladder irrigation has a large blood clot in the urinary drainage tubing. What actions should the nurse perform first? a. Tell the UAP to offer more choices during the personal care to prevent anxiety b. Meet with the UAP later to role model more assertive communication techniques c. Assume care of the client to ensure that effective communication is maintained. d. Affirm that the UAP is using and effective strategy to reduce the client's anxiety.

Intravenous administration of thyroid hormone Rationale: Reduction is an effective technique is managing the anxiety of client with Alzheimer's disease, so the nurse should affirm the UAP is using an effective strategy (A). Nurse assertive communication and offering more choices (B) may increase... an agitation (C) is not indicated since the UAP is using redirection, an effective strategy.

The mother of a one-month-old boy born at home brings the infant to his first well...was born two weeks after his due date, and that he is a "good, quiet baby" who almost... hypothyroidism, what question is most important for the nurse to ask the mother? a. Has your son had any immunizations yet? b. Is your son sleepy and difficult to feed? c. Are you breastfeeding or bottle feeding your son? d. Were any relatives born with birth defects?

Is your son sleepy and difficult to feed? Rationale: Like adults with hypothyroidism, excess fatigue is common and a "good" baby is of.... occurs with hypothyroidism and can result in poor sucking.

The nurse is teaching a client about the antiulcer medications ranitidine which was... statement best describes the action of this drug? a- It blocks the effects of histamine, causing decreased secretion of acid b- Ranitidine will neutralize gastric acid and decrease gastric pH c- This drug provides a protective coating over the gastric mucosa d- It effectively blocks 97% of the gastric acid secreted in the stomach

It blocks the effects of histamine, causing decreased secretion of acid

A client with a new diagnosis of Raynaud's disease lives alone. Which instruction should the nurse include in the client's discharged teaching plan?

Keep room temperature 80

A young adult female presents at the emergency center with acute lower abdominal pain. Which assessment finding is most important for the nurse to report to the healthcare provider? a. Pain scale rating at 9 on a 0-10 scale b. Last menstrual period was 7 weeks ago c. Reports white curdy vaginal discharge d. History of irritable bowel syndrome IBS

Last menstrual period was 7 weeks ago Rationale: Acute lower abdominal pain in A young adult female can be indicative of an ectopic pregnancy, which can be life threatening. Since the clients last menstrual period was seven weeks ago a pregnancy test to be obtained to ruled out ectopic pregnancy, which can result in intra-abdominal hemorrhage caused by a ruptured Fallopian tube. Although the severity of pain requires treatment, the most significant finding is the clients last menstrual period. Other options are not the most important concerns.

A client is receiving continuous bladder irrigation via a triple-lumen suprapubic catheter that was placed during prostatectomy. Which report by the unlicensed assistive personnel (UAP) requires intervention by the nurse?

Leakage around catheter insertion site

The nurse inserts an indwelling urinary catheter as seen in the video what action should the nurse take next? a. Remove the catheter and insert into urethral opening b. Observe for urine flow and then inflate the balloon. c. Insert the catheter further and observe for discomfort. d. Leave the catheter in place and obtain a sterile catheter.

Leave the catheter in place and obtain a sterile catheter.

When entering a client's room to administer an 0900 IV antibiotic, the nurse finds that the client is engaged in sexual activity with a visitor. Which actions should the nurse implement?

Leave the room and close the door quietly

A client with Alzheimer's disease falls in the bathroom. The nurse notifies the charge nurse and completes a fall follow-up assessment. What assessment finding warrants immediate intervention by the nurse? a. Urinary incontinence b. Left forearm hematoma c. Disorientation to surroundings d. Dislodge intravenous site

Left forearm hematoma Rationale: The left forearm hematoma may be indicative an injury, such as broken bone, that requires immediate intervention. A may be likely be due to the inability to use the toilet due to the fall. Disorientation is a common symptom of Alzheimer's disease. IV Dislodged is not an urgent concern.

A client is receiving lactulose (Portalac) for signs of hepatic encephalopathy. To evaluate the client's therapeutic response to this medication, which assessment should the nurse obtain? a. Level of consciousness b. Percussion of abdomen c. Serum electrolytes d. Blood glucose.

Level of consciousness Rationale: Colonic bacteria digest lactulose to create a drug-induces acidic and hyperosmotic environment that draws water and blood ammonia into the colon and coverts ammonia to ammonium, which is trapped in the intestines and cannot be reabsorbed into the systemic circulation. This therapeutic action of lactulose is to reduce serum ammonia levels, which improves the client's level of consciousness and mental status.

During a 26-week gestation prenatal exam, a client reports occasional dizziness...What intervention is best for the nurse to recommend to this client? a. Elevate the head with two pillows while sleeping b. Lie on the left or right side when sleeping or resting. c. Increase intake of foods that are high in iron d. Decrease the amount of carbohydrates in the diet.

Lie on the left or right side when sleeping or resting

The nurse is assessing a middle-aged adult who is diagnosed with osteoarthritis. Which factor in this client's history is a contributor to the osteoarthritis? a- Long distance runner since high school. b- Lactose intolerant since childhood c- Photosensitive to a drug currently taking d- Recently treated for deep vein thrombosis.

Long distance runner since high school. Rationale: Osteoarthritis is a degenerative joint disease of the cause by traumatic or repetitive stress to weight-bearing joint such as high impact sport like running.

A client is admitted for cellulitis surrounding an insect bite on the lower, right arm and intravenous (IV) antibiotic therapy is prescribed. Which action should the nurse implement before performing venipuncture?

Lower the left arm below the level of the heart

An adult female client with chronic kidney disease (CKD) asks the nurse if she can continue...Medications. Which medication provides the greatest threat to this client? a. Magnesium hydroxide (Maalox). b. Birth control pills c. Cough syrup containing codeine d. Cold medication containing alcohol

Magnesium hydroxide (Maalox)

A male client who had a small bowel resection acquired methicillin- resistant Staphylococcus aureus (MRSA) while hospitalized. He was treated and released, but is readmitted today because of diarrhea and dehydration. It is most important for the nurse to implement which intervention

Maintain contact transmission precautions

A client who had an emergency appendectomy is being mechanically ventilated, and soft wrist restrain are in place to prevent self extubation. Which outcome is most important for the nurse to include in the client's plan of care? a. Understand pain management scale b. Maintain effective breathing patterns c. Absence of ventilator associated pneumonia d. No injuries refer to soft restrains occur

Maintain effective breathing patterns Rationale: Basic airway management (B) is the priority. Pain management (A), risk of infection (C), and prevention of injury (D) do not have the same priority as (C)

A 16-year-old male is admitted to the pediatric intensive care unit after being involved in a house fire. He has full thickness burns to his lower torso and extremities. Before a dressing change to his legs, which intervention is most important for the nurse to implement? a- Encourage the parents to stay at the bedside b- Use distraction techniques to reduce pain. c- Maintain strict aseptic technique d- Place a drape over the pubic area.

Maintain strict aseptic technique.

A 59-year-old male client comes to the clinic and reports his concern over a lump that, "just popped up on my neck about a week ago." In performing an examination of the lump, the nurse palpates a large, nontender, hardened left subclavian lymph node. There is not overlying tissue inflammation. What do these findings suggest

Malignancy

An adult client comes to the clinic and reports his concern over a lump that "just popped up on my neck about a week ago." In performing an examination of the lump, the nurse palpates a large, non-tender, hardened left subclavian lymph node. There is no overlying tissue inflammation. What do these finding suggest?

Malignancy

What is the nurse's priority goal when providing care for a 2-year-old child experiencing seizure... a- Stop the seizure activity b- Decrease the temperature c- Manage the airway d- Protect the body from injury

Manage the airway

While changing a client's chest tube dressing, the nurse notes a crackling sensation when gentle pressure is applied to the skin at the insertion site. What is the best action for the nurse to take? a. Apply a pressure dressing around the chest tube insertion site. b. Assess the client for allergies to topical cleaning agents. c. Measure the area of swelling and crackling. d. Administer an oral antihistamine per PRN protocol.

Measure the area of swelling and crackling. Rational: a crackling sensation, or crepitus, indicates subcutaneous emphysema, or air leaking into the skin. This area should be measured, and the finding documented. Other options are not indicated for crepitus.

While assisting a client who recently had a hip replacement into a bed pan, the nurse notices that there is a small amount of bloody drainage on the surgical dressing, the client's skin is warm to the touch, and there is a strong odor from the urine. Which action should the nurse take? a. Obtain a urine sample from the bed pan b. Remove dressing and assess surgical site c. Insert an indwelling urinary catheter d. Measure the client's oral temperature

Measure the client's oral temperature Rationale: The strong odor from the urine and skin that is warm to the touch may indicate that the client has a urinary tract infection. Assessing the client's temperature provides objective information regarding infection that can be reported to the healthcare provider. Urine should be obtained via a clean catch, not the bed pan where it has been contaminated. The drainage on the dressing is normal and does not require direct conservation currently. An indwelling catheter should be avoided if possible because it increases the risk of infection.

The nurse is assessing a first day postpartum client. Which finding is most indicative of a postpartum infection? a. White blood count of 19,000 mm3 b. Oral temperature of 100.2 F (37.9 C) c. Moderate amount of foul-smelling lochia. d. Blood pressure 122/74 mm Hg

Moderate amount of foul-smelling lochia.

The nurse is preparing a client for discharge from the hospital following a liver transplant. Which instruction is most important for the nurse to include in this client's discharge teaching plan? a. Monitor for an elevated temperature b. Measure the abdominal girth daily c. Report the onset of sclera jaundice d. Keep a record of daily urinary output

Monitor for an elevated temperature Rationale: The client should be instructed to monitor or elevated temperature because immunosuppressant agents, which are prescribed to reduce rejection after transplantation, place the client at risk for infection. The client should recognize sign of liver rejection, such as sclera jaundice and increasing abdominal girths, but fever may be the only sign of infection. A is not as important and monitoring for signs of infection.

A client with a history of using illicit drugs intravenously is admitted with Kaposi's sarcoma. Which intervention should the nurse include in this client's admission plan of care?

Monitor for secondary infections.

After receiving lactulose, a client with hepatic encephalopathy has several loose stools. What action should the nurse implement? a- Send stool specimen to the lab b- Measure abdominal girth c- Encourage increased fiber in diet. d- Monitor mental status.

Monitor mental status. Rationale: Administer lactulose to a patient hepatic encephalopathy to lower serum ammonia level, so mental status should be improving.

Which intervention should the nurse include in the plan of care for a client with leukocytosis? a. Avoid intramuscular injections b. Monitor temperature regularly c. Assess skin for petechiae or bruising d. Implement protective isolation measures

Monitor temperature regularly

A client with acute renal failure (ARF) is admitted for uncontrolled type 1 diabetes Mellitus and hyperkalemia. The nurse administers an IV dose of regular insulin per sliding scale. Which intervention is the most important for the nurse to include in this client's plan of care? a. Monitor the client's cardiac activity via telemetry. b. Maintain venous access with an infusion of normal saline. c. Assess glucose via fingerstick q4 to 6 hours. Evaluate hourly urine output for return of normal renal function

Monitor the client's cardiac activity via telemetry. Rational: as insulin lowers the blood glucose of a client with diabetic ketoacidosis (DKA), potassium returns to the cell but may not impact hyperkalemia related to acute renal failure. The priority is to monitor the client for cardiac dysrhythmias related to abnormal serum potassium levels. IV access, assessment of glucose level, and monitoring urine output are important interventions, but do not have the priority of monitoring cardiac function.

A male client is returned to the surgical unit following a left nephrectomy and is medicated with morphine. His dressing has a small amount of bloody drainage, and a JacksonPratt bulb surgical drainage device is in place. Which interventions is most important for the nurse to include in this clients plan of care? a- Monitor urine output hourly. b- Assess for back muscle aches c- Record drainage from drain d- Obtain body weight daily

Monitor urine output hourly. Rationale: When one kidney is removed the remained kidney must do all the volume filtering, so A is immediate to postoperative period.

The nurse teaches an adolescent male client how to use a metered dose inhaler. Seen in the picture. What instruction should the nurse provide? a. Secure the mouthpiece under the tongue. b. Press down on the device after breathing in fully c. Move the device one to two inches away from the mouth d. Breathe out slowly and deeply while compressing the device

Move the device one to two inches away from the mouth Rationale: Optimal position of a metered dose inhaler includes placing the inhaler one two inches away from the mouth.

A client with superficial burns to the face, neck, and hands resulting from a house fire... which assessment finding indicates to the nurse that the client should be monitored for carbon monoxide...? a. Expiratory stridor and nasal flaring b. Mucous membranes cherry red color c. Carbonaceous particles in sputum d. Pulse oximetry reading of 80 percent

Mucous membranes cherry red color

A male client with an antisocial personality disorder is admitted to an in-patient mental health unit for multiple substance dependency. When providing a history, the client justifies to the nurse his use of illicit drugs. Based on this pattern of behavior this client's history is most likely to include which finding?

Multiple convictions for misdemeanors and class B felonies.

A client who developed syndrome of inappropriate antidiuretic hormone (SIADH) associated with small carcinoma of the lung is preparing for discharge. When teaching the client about self-management with demeclocycline (Declomycin), the nurse should instruct the client to report which condition to the health care provider? a- Insomnia b- Muscle cramping c- Increase appetite d- Anxiety.

Muscle cramping Rationale: SIADH causes dilution hyponatremia because of the increased release of ADH, which is treated with water restriction and demeclocycline, a tetracycline derivate that blocks the action of ADH. Signs of hyponatremia (normal 136-145), which indicate the need for increasing the dosage of demeclocycline, should be reported to the healthcare provider. The signs include: plasma sodium level less than 120, anorexia, nausea, weight changes related to fluid disturbance, headache, weakness, fatigue, and muscle cramping. AC& D are not related to hyponatremia.

The nurse is interviewing a client with schizophrenia. Which client behavior requires immediate intervention? a. Lip smacking and frequent eye blinking b. Shuffling gait and stooped posture c. Rocks back and forth in the chair d. Muscle spasms of the back and neck

Muscle spasms of the back and neck

The nurse is preparing a teaching plan for an older female client diagnosed with osteoporosis. What expected outcome has the highest priority for this client? a. Identifies 2 treatments for constipation due to immobility. b. Names 3 home safety hazards to be resolve immediately. c. State 4 risk factors for the development of osteoporosis. d. Lists 5 calcium-rich foods to be added to her daily diet.

Names 3 home safety hazards to be resolve immediately Rational: a major teaching goal for an elderly client with osteoporosis is maintenance of safety to prevent falls. Injury due to a fall, usually resulting in a hip fracture, can result in reduced mobility and associated complications. Other goals are also important when teaching clients who have osteoporosis, but they do not have the priority of preventing falls, which relates to safety.

The nurse should observe most closely for drug toxicity when a client receives a medication that has which characteristic? a- Low bioavailability b- Rapid onset of action c- Short half life d- Narrow therapeutic index.

Narrow therapeutic index.

A client with urticaria due to an environmental allergies is taking diphenhydramine... Which complaint should the nurse identify to the client as a side effect of the OTC medication? a. Nausea and indigestion. b. Hyper salivation c. Eyelid and facial twitching d. Increased appetite

Nausea and indigestion.

A client is admitted to isolation with the diagnosis of active tuberculosis (TB). Which infection control measures should the nurse implement? a. Negative pressure environment b. Contact precautions c. Droplet precautions d. Protective environment

Negative pressure environment

When caring for a client with traumatic brain injury (TBI) who had a craniotomy for increased intracranial pressure (ICP), the nurse assesses the client using the Glasgow coma scale (GCS) every two hours. For the past 8 hours the client's GCS score has been 14. What does this GCS finding indicate about the client?

Neurologically stable without indications of an increased IC

The nurse is caring for several clients on a telemetry unit. Which client should the nurse assess first? The client who is demonstrating? a. A paced rhythm with 100% capture after pacemaker replacement b. Normal sinus rhythm and complaining of chest pain c. Atrial fibrillation with congestive heart failure and complaining of fatigue d. Sinus tachycardia 3 days after a myocardial infarction

Normal sinus rhythm and complaining of chest pain

The nurse is caring for a toddler with a severe birth anomaly that is dying. The parents... holding the child as death approaches. Which intervention is most important for the nurse? a- Notify nursing supervisor and hospital chaplain of the child's impending death. b- Verify that the no resuscitate forms are in the child's medical record c- Ask the parents if they have made arrangements with a funeral home d- Provide staff coverage to sit with them as the child's death approaches.

Notify nursing supervisor and hospital chaplain of the child's impending death.

While removing an IV infusion from the hand of a client who has AIDS, the nurse is struck with the needle. After washing the puncture site with soap & water, which action should the nurse take? a. Complete a usual incident report b. Start prophylactic treatment c. Seek psychological resources d. Notify the employee health nurse.

Notify the employee health nurse.

A client with hyperthyroidism who has not been responsive to medications is admitted for evaluation. What action should the nurse implement? (Click on each chart tab for additional information. Please scroll to the bottom right corner of each tab to view all information contained in the client's medical record.)

Notify the healthcare provider

When changing a diaper on a 2-day-old infant, the nurse observes that the baby's legs are... this finding, what action should the nurse take next? a- Notify the healthcare provider b- Continue care since this is a normal finding c- Document the finding in the record d- Perform range of motion to the joint.

Notify the healthcare provider

An adult male with schizophrenia who has been noncompliant in taking oral antipsychotic medications refuses a prescribed IM medication. Which action should the nurse take?

Notify the healthcare provider of the client's refusal

One day after abdominal surgery, an obese client complains of pain and heaviness in the right calf. What action should the nurse implement?

Observe for unilateral swelling

Four hours after surgery, a client reports nausea and begins to vomit. The nurse notes that the client has a scopolamine transdermal patch applied behind the ear. What action should the nurse take? a. Reposition the transdermal patch to the client's trunk. b. Remove the transdermal patch until the vomiting subsides. c. Notify the healthcare provider of the vomiting. d. Explain that this is a side effect of the medication in the patch.

Notify the healthcare provider of the vomiting. Rational: transdermal scopolamine is used to prevent nausea and vomiting from anesthesia and surgery. The nurse should notify the healthcare provider if the medication is ineffective. The patch should be applied behind the ear and should remain in place to reduce the nausea and vomiting. Nausea and vomiting are no side effects of the medication.

A client is admitted for type 2 diabetes mellitus (DM) and chronic Kidney disease (CKD)... which breakfast selection by the client indicates effective learning? a. Scrambled eggs, bacon, one slice of whole wheat toast with butter and jam. b. Oatmeal with butter, artificial sweetener, and strawberries, and 6 ounces' coffee. c. Banana pancake with maple syrup, sausage links, half grapefruit, and low -fat milk d. Orange juice, yogurt with berries, cold cereal with milk, bran muffin with margarine.

Oatmeal with butter, artificial sweetener, and strawberries, and 6 ounces coffee

In evaluating the effectiveness of a postoperative client's intermittent pneumatic compression devices, which assessment is most important for the nurse to complete? a. Evaluate the client's ability to use an incentive spirometer b. Monitor the amount of drainage from the client's incision c. Observe both lower extremities for redness and swelling d. Palpate all peripheral pulse points for volume and strength

Observe both lower extremities for redness and swelling Rationale: Intermittent compression devices (ICDs) are used to reduce venous stasis and prevent venous thrombosis in mobile and postoperative clients and its effectiveness is best assessed by observing the client's lower extremities for early signs of thrombophlebitis.

The nurse is teaching a male adolescent recently diagnosed with type 1 diabetes mellitus (DM) about self-injecting insulin. Which approach is best for the nurse to use to evaluate do you effectiveness of the teaching? a. Ask the adolescent to describe his level of comfort with injecting himself with insulin. b. Observe him as he demonstrates self-injection technique in another diabetic adolescent c. Have the adolescent list the procedural steps for safe insulin administration. d. Review his glycosylated hemoglobin level 3 months after the teaching session.

Observe him as he demonstrates self-injection technique in another diabetic adolescent Rational: watching the adolescent perform the procedure with another adolescent provides peer support the most information regarding his skill with self-injection. Other options do not provide information about the effectiveness of nurse's teaching.

A client with myasthenia Gravis (MG) is receiving immunosuppressive therapy. Review recent laboratory test results show that the client's serum magnesium level has decreased below the normal range. In addition to contacting the healthcare provider, what nursing action is most important? a. Check the visual difficulties b. Note most recent hemoglobin level c. Assessed for he and Hand joint pain d. Observe rhythm on telemetry monitor

Observe rhythm on telemetry monitor Rationale: If not treated a low little Serum magnesium level can affect myocardial depolarization leading to a lethal arrhythmia, and the nurse should assess for dysrhythmias before contacting the healthcare provider. Other choices are common in MG but do not contribute the Safety risk of low magnesium levels.

In caring for a client with a PCA infusion of morphine sulfate through the right cephalic vein, The nurse assesses that the client in lethargic with a blood pressure of 90/60, pulse rate of 118 beats per minute, and respiratory rate of 8 breaths per minutes. What assessment should the nurse perform next? a. Note the appearance and patency of the client's peripheral IV site. b. Palpate the volume of the client's right radial pulse c. Auscultate the client's breath sounds bilaterally. d. Observe the amount and dose of morphine in the PCA pump syringe.

Observe the amount and dose of morphine in the PCA pump syringe

A client presents to the labor and delivery unit, screaming "THE BABY IS COMING" which action should the nurse implement first.

Observe the perineum

The nurse notes an increase in serosanguinous drainage from the abdominal surgical wound from an obese client. What action should the nurse implement? a. Observe the wound for dehiscence b. Teach the client to splint the incision while coughing c. Assess the skin surrounding the wound for maceration d. Obtain a culture of the wound drainage.

Observe the wound for dehiscence

The father of 4-year-old has been battling metastatic lung cancer for the past 2 years. After discussing the remaining options with his healthcare provider, the client requests that all treatment stop and that no heroic measures be taken to save his life. When the client is transferred to the palliative care unit, which action is most important for the nurse working on the palliative care unit to take in facilitating continuity of care? a. Reassure the client that his child will be allowed to visit b. Obtain a detailed report from the nurse transferring the client. c. Mark the chart with client's request for no heroic measure d. Provide the client whitening information about end-of-life care

Obtain a detailed report from the nurse transferring the client. Rationale: To maintain continuity of care, it is important for the nurse working on the palliative care unit to obtain a detailed "situation, background, assessment, recommendation (SBAR) report, which provide clinical and no clinical information, as well as further information about the client may need. A, C and D are important intervention but not have priority at this time.

An older male who is admitted for end stage of chronic obstructive pulmonary disease (COPD) tells the nurse .... The client provides the nurse with a living will and DNR. What action should the nurse implement? a- Inform the family of the client whishes b- Obtain a prescription for DNR c- Clergy consultation d- Ask the patient why he made this choice

Obtain a prescription for DNR

An adult client with schizophrenia begin treatment three days ago with the Antipsychotic risperidone. The client also received prescription for trazodone as needed for sleep and clonazepam as needed for severe anxiety. When the client reports difficulty with swallowing, what action should the nurse take? a. Obtain a prescription for an anticholinergic medication b. Determine how many hours declined slept last night c. Administer the PRN prescription for severe anxiety d. Watch the thyroid cartilage move while the client swallows

Obtain a prescription for an anticholinergic medication Rationale: Antipsychotic medications have an extrapyramidal side effects one of which is difficult to swallowing the nurse should obtain a prescription for an anticholinergic medication which is used for the treatment of extrapyramidal symptoms. Other options are not warranted actions based on the symptoms presented.

A client with pneumonia has an IV of lactated ringer's solution infusing at 30ml/hr current labor....sodium level of 155 mEq/L, a serum potassium level of 4mEq/L.... what nursing intervention is most important? a.Provide a high-potassium snack, such as bananas. b. Obtain a prescription to increase the IV rate c. Administer the next scheduled dose of antibiotic d. Review the report of the most recent chest x-ray.

Obtain a prescription to increase the IV rate

A toddler with a history of an acyanotic heart defect is admitted to the pediatric intensive...rate of 60 breaths/ minute, and a heart rate of 150 beats/minute. What action should the nurse take? a. Obtain a pulse oximeter reading b. Assess the child blood pressure c. Perform a neurological assessment d. Initiate peripheral intravenous access.

Obtain a pulse oximeter reading

A client with a postoperative wound that eviscerated yesterday has an elevated temperature...most important for the nurse to implement? a. Initiate contact isolation b. Obtain a wound swab for culture and sensitivity c. Assess temperature q4 hours d. Use alcohol-based solutions for hand hygiene.

Obtain a wound swab for culture and sensitivity

A client who has a suspected brain tumor is schedules for a computed (CT) scan. When preparing the client for the client for the CT scan, which intervention should the nurse implement?

Obtain the client's food allergy history

A client with possible acute kidney injury (AKI) is admitted to the hospital and mannitol is prescribed as a fluid challenge. Prior to carrying out this prescription, what intervention should the nurse implement? a. Collect a clean catch urine specimen. b. Instruct the client to empty the bladder. c. Obtain vital signs and breath sounds. d. No specific nursing action is required

Obtain vital signs and breath sounds. Rational: the client's baseline cardiovascular status should be determined before conducting the fluid challenge. If the client manifests changes in the vital signs and breath sounds associated with pulmonary edema, the administration of the fluid challenge should be terminating. Other options would not assure a safe administration of the medication.

The nurse instructs an unlicensed assistive personnel (UAP) to turn an immobilized elderly client with an indwelling urinary catheter every two hours. What additional action should the nurse instruct the UAP to take each time the client is turned? a. Empty the urinary drainage bag b. Feed the client a snack c. Offer the client oral fluids d. Assess the breath sounds

Offer the client oral fluids Rationale: Increasing oral fluid intake reduces the risk of problems associated with immobility, so the UAP should be instructed to offer the client oral fluids every two hours, or whenever turning he client. It is not necessary to empty the urinary bag or feed the client every two hours. Assessment is a nursing function, and UAPs do not have the expertise to perform assessment of breath sounds.

The nurse applies a blood pressure cuff around a client's left thigh. To measure the client's blood pressure, where should the diaphragm of the stethoscope be placed? (Mark the location on one of the images.)

On left thigh with arrow pointing to inner thigh"

A-12-years old boy has a body mass index (BMI) of 28, a systolic pressure and a glycosylated hemoglobin (HBA1C) of 7.8%. Which selection indicated that his mother understands the management of his diet

One whole-wheat bagel with cream cheese, two strips of bacon, six ounces of orange juice.

Which needle should the nurse use to administer intravenous fluids (IV) via a client's implanted port?

One with the clamp

A young adult female client with recurrent pelvic pain for 3 year returns to the clinic for relief of severe dysmenorrhea. The nurse reviews her medical record which indicates that the client has endometriosis. Based on this finding, what information should the nurse provide this client?

Oral contraceptives increase the symptoms of endometriosis.

In determine the client position for insertion of an indwelling urinary catheter, it is most important for the nurse to recognize which client condition? a- High urinary PH b- Abdominal Ascites c- Orthopnea d- Fever.

Orthopnea Rationale: If the client is orthopneic, the nurse needs to adapt the insertion position that does not place the client in a supine position (the head of the bed should be elevated as much as possible).

Following an open reduction of the tibia, the nurse notes bleeding on the client's cast. Which action should the nurse implement?

Outline the area with ink and check it every 15 minutes to see if the area has increased

A client on a long-term mental health unit repeatedly takes own pulse regardless of the circumstance. What action should the nurse implement? a. Overlook the client's behavior. b. Distract client to interfere with the ritual. c. Ask why the client checks the pulse. d. Hold client's hand to stop the behavior.

Overlook the client's behavior.

A client with a liver abscess develops septic shock. A sepsis resuscitation bundle protocol is initiated and the client receives a bolus of IV fluids. Which parameter should the nurse monitor to assess effectiveness of the fluid bolus?

Oxygen saturation

To obtain an estimate of a client's systolic B/P. What action should the nurse take first? a. Palpate the client's brachial pulse b. Pump up the blood pressure cuff c. Position the stethoscope diaphragm d. Release the blood pressure cuff valve

Palpate the client's brachial pulse

A native-American male client diagnosed with pneumonia, states that in addition to his prescribed medical treatment of IV antibiotics he wishes to have a spiritual cleaning performed. Which outcome statement indicates that the best plan of care was followed? a. Identifies his ethnocentric values and behaviors b. States an understanding of the medical treatment c. Participated actively in all treatments regimens d. Expresses a desire for cultural assimilation

Participated actively in all treatments regimens Rationale: indicates active participation by the client, which is required for treatment to be successful. The best plan of care should incorporate the valued and treatments of both cultures and in this case there is no apparent cultural clash between the two forms of treatment. The client has already identify he's cultural values (A). (B) Only considers one of the two treatment modalities desired by the client the client has already chosen how he wishes to assimilate his cultural values with the prescribed medical treatment (D).

The nurse has received funding to design a health promotion project for AfricanAmerican women who are at risk for developing breast cancer. Which resource is most important in designing this program? a. A listing of African-American women so live in the community b. Participation of community leaders in planning the program c. Morbidity data for breast cancer in women of all races d. Technical assistance to produce a video on breast self-examination.

Participation of community leaders in planning the program

The nurse is preparing dose # 7 of an IV piggyback infusion of tobramycin for a 73-yearol client with... Infected pseudomonas aeruginosa. Which assessment data warrants further intervention by the nurse? a- Peak and through levels has not been drawn since the tobramycin was started b- Today labs report indicates a white blood cell count of 13,000 cell/mm3 or 13 x 10777/L (S1) c- A serum creatinine level of 1.0 mg/dl or 88 mcmol/L (S1) is documented on yesterday flowsheet. d- The culture growth form the burn areas is sensitive to aminoglycosides.

Peak and through levels has not been drawn since the tobramycin was started

A client who has been in active labor for 12 hours suddenly tells the nurse that she has a strong urge to have a bowel movement. What action should the nurse take? a- Allow the client to use a bedpan. b- Assist the client to the bathroom c- Perform a sterile vaginal exam d- Explain the fetal head is descending.

Perform a sterile vaginal exam Rationale: When a client in active labor suddenly expresses the urge to have a bowel movement, a sterile vaginal exam should be performed to determine if the fetus is descending.

In assessing a pressure ulcer on a client's hip, which action should the nurse include? a. Determine the degree of elasticity surrounding the lesion b. Photograph the lesion with a ruler placed next to the lesion c. Stage the depth of the ulcer using the Braden numeric scale d. Use a gloved finger to palpate for tunneling around the lesion

Photograph the lesion with a ruler placed next to the lesion Rationale: An ulcer extends into the dermis or subcutaneous tissue and is likely to increase in size and depth, so assessment should include photograph with measuring device to document the size of the lesion.

The charge nurse observes the practical nurse (PN) apply sterile gloves in preparation for performing a sterile dressing change. Which action by the PN requires correction by the charge nurse? a- Opening the package b- Picking up the second glove c- Picking up the first glove d- Positioning of the table

Picking up the second glove

The nurse is demonstrating correct transfer procedures to the unlicensed assisted personnel (UAP) working on a rehabilitation unit. The UAPs ask the nurse how to safely move a physically disabled client from the wheelchair to a bed. What action should the nurse recommended? a. Hold the client at arm's length while transferring to better distribute the body weight. b. Apply the gait belt around the client's waits once standing position has been assumed. c. Place a client's locked wheelchair on the client's strong side next to the bed. d. Pull the client into position by reaching from the opposite side of the bed.

Place a client's locked wheelchair on the client's strong side next to the bed. RATIONALE: Placing the wheelchair on the client's strong side offers the greatest stability for the transfer. Holding the client arm's length or pulling from the opposite site of the bed reflect poor body mechanism. Using a gait belt offers additional safety for the client but should be done after the wheelchair has be put into the proper place and the wheels have been locked and before the client has assumed a standing position.

A 17-year -old male is brought to the emergency department by his parents because he has been coughing and running a fever with flu-like symptoms for the past 24 hours. Which intervention should the nurse implement first?

Place a mask on the client's face.

While assisting a male client who has muscular dystrophy (MD) to the bathroom, the nurse observes that he is awkward and clumsy. When he expresses his frustration and complains of hip discomfort, which intervention should the nurse implement?

Place a portable toilet next to the bed

The nurse delegates to an unlicensed assistive personnel (UAP) denture care for a client with...daily leaving. When making this assignment, which instruction is most important for the nurse to do? a- Do not remove the dentures, but instead brush them within the mouth b- Place a washcloth in the sink while cleaning the dentures. c- Use tepid, not hot, water to clean the dentures d- Avoid damaging the dentures using a soft-bristled toothbrush.

Place a washcloth in the sink while cleaning the dentures

At 40 week gestation, a laboring client who is lying is a supine position tells the nurse that she has finally found a comfortable position. What action should the nurse take? a. Encourage the client to turn on her left side. b. Place a pillow under the client's head and knees. c. Explain to the client that her position is not safe. d. Place a wedge under the client's right hip.

Place a wedge under the client's right hip Rationale: Hypotension from pressure on the vena cava is a risk for the full-term client. Placing a wedge under the right hip will relieve pressure on the vena cava. Other options will either not relieve pressure on the vena cava or would not allow the client the remaining her position of choice.

The nurse is caring for a client immediately after inserting a PICC line. Suddenly, the client becomes anxious and tachycardiac, and loud churning is heard over the pericardium upon auscultation. What action should the nurse take first? a. Place client in Trendelenburg position on the left side. b. Administer precordial thump c. Monitor the client with a 12-lead electrocardiogram d. Request a STAT portable chest x-ray.

Place client in Trendelenburg position on the left side

After applying an alcohol-based hand rub to the palms of the hand and rubbing the hand together, what action should the nurse do next? a. Vigorous rub both hands together under running water b. Path both hands dry keeping the fingers lower that the arm c. Place one hand on top of the other and interlace the fingers d. Hold both hand with the fingers pointing upward until dry.

Place one hand on top of the other and interlace the fingers

The nurse is planning to assess a client's oxygen saturation to determine if additional oxygen is needed via nasal cannula. The client has a bilateral below-the-knee amputation and pedal pulses that are weak and threaty. What action should the nurse take? a. Document that an accurate oxygen saturation reading cannot be obtained b. Elevate to client's hands for five minutes prior to obtaining a reading from the finger c. Increase the oxygen based on the clients breathing patterns and lung sounds d. Place the oximeter clip on the ear lobe to obtain the oxygen saturation reading

Place the oximeter clip on the ear lobe to obtain the oxygen saturation reading Rationale: Pulse oximeter clips can be attached to the earlobe to obtain an accurate measurement of oxygen saturation. Other options will not provide the needed assessment.

The nurse enters a client's room and observe the unlicensed assistive personnel (UAP) making an occupied bed as seen in the picture. What action should the nurse take first?

Place the side rails in an up position

Which intervention should the nurse implement for a client with a superficial (first degree) burn? a. Spray an anesthetic agent over the burn every 3 to 4 hours b. Position the burn victim in front of a cool fan to decrease discomfort c. Apply ice pack for 30 mints to lower surface temperature d. Place wet clothes on the burned areas for short periods of time.

Place wet cloths on the burned areas for short periods of time. Rationale: D provides comfort and helps to relive the pain of a first degree burn, which involves only the epidermal layer of the skin.

During discharge teaching, an overweight client heart failure (HF) is asked to make a grocery list for the nurse to review. Which food choices included on the client's list should the nurse encourage? (Select all that apply)

Plain, air-popped popcorn. Natural whole almonds.

A health care provider continuously dismisses the nursing care suggestions made by staff nurses. As a result...dealing with the healthcare provider. What action should the nurse manager implement? a- Confront the health care provider about the perceived lack of respect for the staff nurses. b- Plan an interdisciplinary staff meeting to develop strategies to enhance client care c- Request an investigation about the perceived incivility of the healthcare provider interaction. d- Remind the staff that avoidance behavior is not a professional way to handle the problem.

Plan an interdisciplinary staff meeting to develop strategies to enhance client care

The nurse is collecting sterile sample for culture and sensitivity from a disposable three chamber water-seal drainage system connected to a pleural chest tube. The nurse should obtain the sample from which site on the drainage system?

Plastic tubing located at the chest insertion site

An 11-year-old client is admitted to the mental health unit after trying to run away from home and threatening self-harm. The nurse establishes a goal to promote effective coping, and plans to ask the client to verbalize three ways to deal with stress. Which activity is best to establish rapport and accomplish this therapeutic goal?

Play a board game with the client and begin taking about stressor

The nurse is preparing to gavage feed a premature infant through an orogastric tube. During insertion of the tube, the infant's heart rate drops to 60 beats / minute. Which action should the nurse take?

Postpone the feeding until the infant's vital signs and stable

During a cardiopulmonary resuscitation of an intubated client, the nurse detects a palpable pulse throughout the two minutes cycle chest compression and absent breath sounds over the left lung. What action should the nurse implement? a- Instruct the compressor to stop chest compression. b- Advise ventilator to increase bag-mask ventilation rate. c- Plan to suction endotracheal tube at two-minute check. d- Prepare for the endotracheal tube to be repositioned

Prepare for the endotracheal tube to be repositioned

A female client who was mechanically ventilated for 7 days is extubated. Two hours later...productive cough, and her respirations are rapids and shallow. Which intervention is most important? a- Review record of recent analgesia b- Provide frequent pulmonary toilet c- Prepare the client for intubation d- Obtain STAT arterial blood gases

Prepare the client for intubation

A client's telemetry monitor indicates ventricular fibrillation (VF). What should the nurse do first? a. Administer epinephrine IV b. Give an IV bolus of amiodarone c. Provide immediate defibrillation d. Prepare for synchronized cardioversion

Provide immediate defibrillation

While performing a skin inspection for a female adult client, the nurse observes a rash that is well circumscribed, has silvery scales and plaques, and is located on the elbows and knees. These assessment findings are likely to indicate which condition? a- Tinea corporis b- Herpes zoster c- Psoriasis d- Drug reaction

Psoriasis Rationale: Psoriasis is typically located on the elbow and knees

While taking vital signs, a critically ill male client grabs the nurse's hand and ask the nurse not to leave. What action is best for the nurse to take?

Pull up a chair and sit beside the client's bed

A low-risk primigravida at 28-weeks gestation arrives for her regular antepartal clinic visit. Which assessment finding should the nurse consider within normal limits for this client? a. Pulse increase of 10 beats/minute b. Proteinuria c. Glucosuria d. Fundal height 0f 22 centimeters

Pulse increase of 10 beats/minute Rationale: Blood volume increases 25 to 40 % in pregnancy which increases cardiac output and increases heart rate by approx. 10 to 20 beats/ mints. Proteinuria is for preeclampsia, Glucosuria is for gestational diabetes. A fundal height for 28 weeks should be at 28 cm not 22

The nurse determines that a client's pupils constricts as they change focus from a far object. What documentation should the nurse enter about this finding? a. Pupils reactive to accommodation b. Nystagmus present with pupillary focus. c. Peripheral vision intact d. Consensual pupillary constriction present

Pupils reactive to accommodation

The nurse is preparing a 50 ml dose of 50% dextrose IV for a client with insulin SHOCK... medication? a- Dilute the Dextrose in one liter of 0.9% Normal Saline solution. b- Mix the dextrose in a 50 ml piggyback for a total volume of 100 ml. c- Push the undiluted Dextrose slowly through the currently infusion IV. d- Ask the pharmacist to add the Dextrose to a TPN solution.

Push the undiluted Dextrose slowly through the currently infusion IV Rationale: To reverse life-threatening insulin shock, the nurse should administer the 50% Dextrose infusing IV.

The nurse is assessing a client with a small bowel obstruction who was hospitalized 24 hours ago. Which assessment finding should the nurse report immediately to the healthcare provider? a. Hypoactive bowel sounds in the lower quadrant. b. Rebound tenderness in the upper quadrants. c. Tympani with percussion of the abdomen. d. Light colors gastric aspirate via the nasogastric tube.

Rebound tenderness in the upper quadrants Rationale: Rebound tenderness in the upper quadrant may be indicative of peritonitis. A is a clinical finding associated with bowel obstruction and does not need to be reported D may be something characteristic of the client's condition.

733. The nurse is preparing a community education program on osteoporosis. Which instruction is helpful in preventing bone loss and promoting bone formation

Recommend weigh bearing physical activity

The nurse is preparing a community education program on osteoporosis. Which instruction is helpful in preventing bone loss and promoting bone formation? a. Recommend weigh bearing physical activity b. Reduce intake of foods high in vitamin D c. Decrease intake of foods high in fat d. Minimize heavy lifting and bending.

Recommend weigh bearing physical activity Rationale: Active weight-bearing exercise is a primary preventive measure for osteoporosis. C is indicated for client with cardiac and liver diseases. D may decrease injuries but is not directed toward slowing bone loss and promoting bone formation.

When assessing the surgical dressing of a client who had abdominal surgery the previous day, the nurse observes that a small amount of drainage is present on the dressing and the wound's Hemovac suction device is empty with the plug open. How should the nurse respond? a. Replace the dressing and remove the drainage device b. Reposition the drainage device and keep the plug open c. Notify the healthcare provider that the drain is not working d. Recompress the wound suction device and secure to plug

Recompress the wound suction device and secure to plug Rationale: The plug of a wound suction device, such as a Hemovac, should be closed after compressing the device to apply gentle suction in a closed surgical wound to facilitate the evacuation of subcutaneous fluids into the device. Compressing the device and securing the plug should restore function of the closed wound device. A small amount of drainage should be marked on the dressing, but replacing the dressing is not necessary and the nurse should not remove the device. Other options are not indicated.

The nurse is assessing and elderly bedridden client. Which finding indicates that the turning and positioning schedule is effective in protecting the client's skin?

Reddened skin areas disappear within 15 minutes of being turned and positioned.

Which interventions should the nurse include in a long-term plan of care for a client with COPD? a- Reduce risk factors for infection b- Administer high flow oxygen during sleep c- Limit fluid intake to reduce secretions d- Use diaphragmatic breathing to achieve better exhalation

Reduce risk factors for infection Rationale: Interventions aimed at reducing the risk factors of infections should be included in the plan of care COPD client are at particular risk for respiratory infection. Prevention and early detection of infections are necessary

Following routine diagnostic test, a client who is symptom-free is diagnosed with Paget's disease. Client teaching should be directed toward what important goal for this client? a- Maintain adequate cardiac output b- Promote adequate tissue perfusion c- Promote rest and sleep d- Reduce the risk for injury

Reduce the risk for injury Rationale: Paget's is a metabolic bone disorder which place the client at high risk for injury. Once the client is symptom free the next goal is reducing risk for injury

When checking a third grader's height and weight the school nurse notes that these measurements have not changed in the last year. The child is currently taking daily vitamins, albuterol, and methylphenidate for attention deficit hyperactivity disorder (ADHD). Which intervention should the nurse implement?

Refer child to the family healthcare provider

When gathering for a group therapy session at 1400 hours, a female client complains to the nurse that a smoking break has not been allowed all day. The nurse responds that 15 minute breaks were called over the unit intercom after breakfast and after lunch. The nurse is using what communication technique in responding to the client? a. Doubt b. Observation c. Confrontation d. Reflection

Reflection

Which instruction is most important for the nurse to provide a client who receives a new plan of care to treat osteoporosis? a. Begin a weight-bearing exercise plan b. Increase intake of foods rich in calcium c. Schedule a bone density tests every year. d. Remain upright after taking the medication.

Remain upright after taking the medication Rationale: Risendronate, causes reflux and esophageal erosion.

The nurse observes an adolescent client prepare to administer a prescribed corticosteroid medication using a metered dose inhaler as seen in the picture. What action should the nurse take? a. Remind the client to hold his breath after inhaling the medication b. Confirm that the client has correctly shaken the inhaler c. Affirm that the client has correctly positioned the inhaler d. Ask the client if he has a spacer to use for this medication

Remind the client to hold his breath after inhaling the medication

The nurse is reinforcing home care instructions with a client who is being discharged following...prostate (TURP). Which intervention is most important for the nurse to include in the client... a. Avoid strenuous activity for 6 weeks b. Report fresh blood in the urine. c. Take acetaminophen for fever 101 d. Consume 6 to 8 glasses of water daily.

Report fresh blood in the urine

A client with hypertension receives a prescription for enalapril, an angiotensin... instruction should the nurse include in the medication teaching plan? a. Increase intake of potassium-rich foods b. Report increased bruising of bleeding c. Stop medication if a cough develops d. Limit intake of leafy green vegetables

Report increased bruising of bleeding Rationale: ACEIs can cause thrombocytopenia and increased risk for bruising and bleeding. A is not necessary because is a potassium-sparing

A female client is taking alendronate, a bisphosphate, for postmenopausal osteoporosis. The client tells the nurse that she is experiencing jaw pain. How should the nurse respond? a- Determine how the client is administering the medication b- Confirm that this is a common symptom of osteoporosis c- Report the client's jaw pain to the healthcare provider. d- Advise the client to gargle with warm salt water twice daily.

Report the client's jaw pain to the healthcare provider. Rationale: Bisphosponates, including alendronate, can cause osteonecrosis of jaw, which should be reported to the healthcare provider © for evaluation. Incorrect administration (A) such as failing to remain upright after taking the medication, can contribute to esophageal reactions, but does not causes haw pain. Jaw pain is not a symptom of osteoporosis and is not relieved with saline throat gargles.

The nurse is preparing a 4-day-old I infant with a serum bilirubin level of 19 mg/dl (325 micromol/L) for discharge from the hospital. When teaching the parents about home phototherapy, which instruction should the nurse include in the discharge teaching plan? a. Reposition the infant every 2 hours. b. Perform diaper changes under the light. c. Feed the infant every 4 hours. d. Cover with a receiving blanket.

Reposition the infant every 2 hours. Rational: An infant, who is receiving phototherapy for hyperbilirubinemia, should be repositioned every two hours. The position changes ensure that the phototherapy lights reach all of the body surface areas. Bathing, feedings, and diaper changes are ways for the parents to bond with the infant and can occur away from the treatment. Feedings need to occur more frequently than every 4 hours to prevent dehydration. The infant should wear only a diaper so that the skin is exposed to the phototherapy.

A gravida 2 para 1, at 38-weeks gestation, scheduled for a repeat cesarean section in one week, is brought to the labor and delivery unit complaining of contractions every 10 minutes. While assessing the client, the client's mothers enter the labor suite and says in a loud voice, "I've had 8 children and I know she's in labor. I want her to have her cesarean section right now!" what action should the nurse take?

Request the mother to leave the room

The nurse is preparing to administer an IV dose of ciprofloxacin to a client with urinary tract infection. Which client data requires the most immediate intervention by the nurse?

Serum creatinine of 4.5 mg/dl (398 mcmol/L SI)

To reduce staff nurse role ambiguity, which strategy should the nurse-manager implement?

Review the staff nurse job description to ensure that it is clear, accurate, and current

A client whose wrists are sutured from a recent suicide attempt is been transferred from a medical unit. Which nursing diagnosis is of the highest priority? a- Risk for self-directed violence related to impulsive actions b- Risk for violence related to feeling of guilt and failure c- Low self-esteem related to feeling of loss of control d- Ineffective coping related to violent actions towards self.

Risk for self-directed violence related to impulsive actions

A client who had a gestational trophoblastic disease (GTD) evacuated 2 days ago is being...18 months-old child and lives in a rural area. Her husband takes the family car to work daily...transportation during the day. What intervention is most important for the nurse to implement? a. Teach a client amount the use of a home pregnancy test. b. Schedule a weekly home visit to draw hCG values. c. Make a 5 week follow- up with healthcare provider d. Begin chemotherapy administration during the first home visit

Schedule a weekly home visit to draw hCG values Rationale: To monitor for development of choriocarcinoma, a complication TD, level of hCG should be monitor for negative results.

A client with multiple sclerosis (MS) is admitted to the medical unit. The client reports... which action should the nurse implement to reduce the client's risk for falls?

Schedule frequent rest periods Provide assistance to bedside commode Teach to patch one eye when ambulating

An Insulin infusion for a client with diabetes mellitus who is experiencing hyperglycemic hyperosmolar...in addition to the client's glucose, which laboratory value is most important for the nurse to monitor? a. Urine ketones b. Urine albumin c. Serum protein d. Serum potassium

Serum potassium

A client is admitted to a mental health unit after attempting suicide by taking a handful of medications. In developing a plan of care for this client, which goal has the highest priority?

Signs a no-self-harm contract.

An infant is receiving gavage feedings via nasogastric tube. At the beginning of the feeding, the infant's heart rate drops to 80 beats / minute. What action should the nurse take

Slow the feeding and monitor the infant's response.

To reduce the risk of symptoms exacerbation for a client with multiple sclerosis (MS), which instructions should the nurse include in the client's discharge plan? (Select all that apply).

Space activities to allow for rest periods Take warm baths before starting exercise

The nurse is caring for a 17-year-old male who fell 20 feet 5 months ago while climbing the side of a cliff and has been in a sustained vegetative state since the accident. Which intervention should the nurse implement?

Talk directly to the adolescent while providing care

A nurse stops at the site of a motorcycle accident and finds a young adult male lying face down in the road in a puddle of water. It is raining, no one is available to send for help, and the cell phone is in the car about 50 feet away. What action should the nurse take first? a. Examine the victim's body surfaces for arterial bleeding b. Stabilize the victim's neck and roll over to evaluate his status c. Return to the car to call emergency response 911 for help d. Open the airway and initiate resuscitative measures

Stabilize the victim's neck and roll over to evaluate his status

During the infusion of a second unit of packed red blood cells, the client's temperature increases from 99 to 101.6 f. which intervention should the nurse implement?

Stop the transfusion start a saline

When administering ceftriaxone sodium (Rocephin) intravenously to a client before... most immediate intervention by the nurse? a. Stridor b. Nausea c. Headache d. Pruritus

Stridor Rationale Stridor, a crowing respiration, indicates the client is experiencing bronchospasm, as a reaction to Rocephin, and antibiotic. The finding requires immediate action by the nurse. B and C are side effects that are not life-threatening. Pruritus may be the result as... and need nursing intervention but is of less immediacy than stridor.

A male client reports to the clinic nurse that he has been feeling well and is often "dizzy" his blood pressure is elevated. Based on this findings, this client is at a greatest risk for which pathophysiological condition? a. Pulmonary hypertension b. Left ventricular hypertrophy c. Renal failure d. Stroke

Stroke

Which assessment finding for a client who is experiencing pontine myelinolysis should the nurse report to the healthcare provider? a. Sudden dysphagia b. Blurred visual field c. Gradual weakness d. Profuse diarrhea

Sudden dysphagia Rationale: Osmotic demyelination, also known as pontine myelinolysis, results in destruction of the myelin sheath that covers nerve cell in the brainstem. This condition can be caused by rapid correction of hyponatremia and is often seen in those with syndrome of inappropriate antidiuretic hormone, Symptoms of pontine myelinolysis are sudden and can include dysphagia, para or quadriparesis and dysarthria. Due to the risk of aspiration the healthcare provider should be notified of the client's sudden onset of difficulty swallowing dysphagia (A). Diplopia not blurred vision (B) may be experienced. Weakness occurs suddenly, rather than gradually (C). Constipation, not diarrhea (D), is common due to decreased motility.

The nurse is making a home visit to a male client who is in the moderate stage of Alzheimer's diseases. The client's wife is exhausted and tells the nurse that the family plans to take turns caring for the client in their home, each keeping him for two weeks at a time. How should the nurse respond? a. Advise the client's spouse to consider inpatient hospice care as an alternative b. Suggest that each rotation last one week, rather than two, to prevent caregiver fatigue c. Use active listening to allow the client and spouse to express their feelings about the plan d. Suggest enrolling the client in adult daycare instead of rotating among family.

Suggest enrolling the client in adult daycare instead of rotating among family Rationale: Suggesting a viable alternative, such as adult daycare provides an option to allow the spouse respite the least disruption to routines and environment.

A male client with cancer who has lost 10 pounds during the last months tells the nurse that beef, chicken, and eggs, which used to be his favorite foods, now they taste "bitter". He complains that he simply has no appetite. What action should the nurse implement?

Suggest the use of alternative sources of protein such as dairy products and nuts

The nursing staff on a medical unit includes a registered nurse (RN), practical nurse (LPN), and unlicensed assistant personnel (UAP). Which task should the charge nurse assign to the RN?

Supervised a newly hired graduate nurse during an admission assessment

A male client is having abdominal pain after a left femoral angioplasty and stent, and is asking for additional pain medication for right lower quadrant pain (9/10), two hours ago, he received hydrocodone / acetaminophen 7.5/7.50 mg his vital signs are elevated from reading of a previous hour: temperature 97.8 F, heart rate 102 beats / minute, respiration 20 breaths/minutes. His abdomen is swollen, the groin access site is tender, peripheral pulses are present, but left is greater than right. Preoperatively, clopidrogel was prescribed for a history of previous peripheral stents. Another nurse is holding manual pressure on the femoral arterial access site which may be leaking into the abdomen. What data is needed to make this report complete? a. Client's lungs are clear bilaterally and oxygen saturation is 97% b. Surgeon needs to see client immediately to evaluate the situation c. Left peripheral pulses were present only by Doppler pre-procedure d. Client' history includes multiple back surgeries and chronic pain.

Surgeon needs to see client immediately to evaluate the situation

While attempting to stablish risk reduction strategies in a community, the nurse notes that the regional studies have indicated....persons with irreversible mental deficiencies due to hypothyroidism. The nurse should seek funding to implement which screening measure?

T4 levels in newborns

A female client receives a prescription for alendronate sodium (Fosamax) to treat her newly diagnose osteoporosis. What instruction should the nurse include in the client's teaching plan?

Take on an empty stomach with a full glass of water

A male client returns to the mental health clinic for assistance with his anxiety reaction that is manifested by a rapid heartbeat, sweating, shaking, and nausea while driving over the bay bridge. What action I the treatment plan should the nurse implement? a. Tell the client to drive over the bridge until fear is manageable b. Teach client to listen to music or audio books while driving c. Encourage client to have spouse drive in stressful places. d. Recommend that the client avoid driving over the bridge.

Teach client to listen to music or audio books while driving

An older client is admitted for repair of a broken hip. To reduce the risk for infection in the postoperative period, which nursing care interventions should the nurse include in the client's plan of care? (Select all that apply)

Teach client to use incentive spirometer q2 hours while awake. Remove urinary catheter as soon as possible and encourage voiding.

One year after being discharged from the burn trauma unit, a client with a history of 40% full-thickness burns is admitted with bone pain and muscle weakness. Which intervention should the nurse include in the clients plan of care a. Encourage Progressive active range of motion b. Teach need for dietary and supplementary vitamin D3 c. Explain the need for skin exposure to sunlight without sunscreen d. Instruct the client to use of muscle strengthening exercises

Teach need for dietary and supplementary vitamin D3 Rationale: Burn injury results in the acute loss of bone as well as the development of progressive vitamin D deficiency because burn scar tissue and adjacent normal-appearing skin cannot convert normal quantities of the precursors for vitamin D3 that is synthesized from ultraviolet sun rays which is needed for strong bones. Clients with a history of full thickness burns should increase their dietary resources of vitamin D and supplemental D3 (B). range of motion (A) and muscle strengthening exercises (D) do not treat he is underlying causes of the bone pain and weakness unprotected sunlight (C) should be avoided.

A client with polycystic kidney disease (PKD) receiving antibiotics for an infected cyst is experiencing severe pain. What action should the nurse implement? a. Hold the next dose of antibiotic until contacting the healthcare provider b. Teach the client how to use a dry heating pad over the painful area c. Encourage the client to practice pelvic floor exercises every hour d. Assist the client to splint the site by applying an abdominal binder

Teach the client how to use a dry heating pad over the painful area

An unlicensed assistive personnel (UAP) informs the nurse who is giving medications that a female client is crying. The client was just informed that she has a malignant tumor. What action should the nurse implement first?

Tell the client that the nurse will be back to talk to her after medications are given

For the past 24 hours, an antidiarrheal agent, diphenoxylate, has been administered to a bedridden, older client with infectious gastroenteritis. Which finding requires the nurse to take further action?

Tented skin turgor

A client who is recently diagnosed with type 2 diabetes mellitus (DM) ask the nurse how this type of diabetes leads to high blood sugar. What Pathophysiology mechanism should the nurse explain about the occurrence of hyperglycemia in those who have type 2 DM? a- Immune antibodies attack pancreatic beta cells resulting in no insulin b- The body cells develop resistance to the action of insulin. c- Body organs produce less insulin and more glucagon d- The liver produces excess glucose in response to excess glycotrophic hormones

The body cells develop resistance to the action of insulin

A client with a large pleural effusion undergoes a thoracentesis. Following the procedure, which assessment finding warrants immediate intervention by the nurse? a. The client has asymmetrical chest wall expansion b. The clients complain of pain at the insertion site c. The client chest's x-ray indicates decreased pleural effusion d. The client's arterial blood gases are pH 7.35, PaO2 85, Pa CO2 35, HCO3 26

The client has asymmetrical chest wall expansion Rationale: A potential complication of thoracentesis is a pneumothorax. The symptoms of a pneumothorax are uneven, unequal movement of the chest wall. A is an expected finding after the local anesthetic effects "wear off" B is a desired result of thoracentesis and C is within normal limits.

A 35 years old female client has just been admitted to the post anesthesia recovery unit following a partial thyroidectomy. Which statement reflects the nurse's accurate understanding of the expected outcome for the client following this surgery? a- Supplemental hormonal therapy will probably be unnecessary b- The thyroid will regenerate to a normal size within a few years. c- The client will be restricted from eating seafood d- The remainder of the thyroid will be removed at a later date.

The client will be restricted from eating seafood

A client with bipolar disorder began taking valproic acid (Depakote) 250 mg PO three times daily two months ago. Which finding provides the best indication that the medication regimen is effective? a. The nurse note that no pills remain in the prescription bottle. b. The client serum Depakote level is 125 mcg/ml c. The family reports a great reduction in client's maniac behavior d. The client denies any occurrence of suicidal ideation.

The family reports a great reduction in client's maniac behavior

Following breakfast, the nurse is preparing to administer 0900 medications to clients on a medical floor. Which medication should be held until a later time? a. The loop-diuretic furosemide (Lasix) for a client with a serum potassium level of 4.2 mEq/L b. The mucosal barrier, sucralfate (Carafate), for a client diagnosed with peptic ulcer disease. c. The antiplatelet agent aspirin, for a client who is scheduled to be discharged within the hour d. The antifungal nystatin (mycostatin) suspension, for a client who has just brushed his teeth.

The mucosal barrier, sucralfate (Carafate), for a client diagnosed with peptic ulcer disease Rationale: Carafate coats the mucosal lining prior to eating a meal, so this medication should be held until prior to the next meal.

A male client is discharged from the intensive care unit following a myocardial infarction, and the healthcare provider low-sodium diet. Which lunch selection indicates to the nurse that this client understands the dietary restrictions? a- Turkey salad sandwich. b- Clam chowder c- Macaroni and cheese d- Bacon, lettuce, and tomato sandwich

Turkey salad sandwich.

A female client with rheumatoid arthritis (RA) comes to the clinic complaining of joint pain and swelling. The client has been taking prednisone (Deltasone) and ibuprofen (Motrin Extra Strength) every day. To assist the client with self-management of her pain, which information should the nurse obtain? a. Presence of bruising, weakness, or fatigue b. Therapeutic exercise included in daily routine. c. Average amount of protein eaten daily d. Existence of gastrointestinal discomfort

Therapeutic exercise included in daily routine

A client is admitted to a medical unit with the diagnosis of gastritis and chronic heavy alcohol abuse. What should the nurse administered to prevent the development of Wernicke's syndrome? a. Lorazepam (Ativan) b. Famotidine (Pepcid) c. Thiamine (Vitamin B1) d. Atenolol (Tenormin)

Thiamine (Vitamin B1) Rationale: Thiamine replacement is critical in preventing the onset of Wernickes encephalopathy, an acute triad of confusion, ataxia, and abnormal extraocular movements, such as nystagmus related to excessive alcohol abuse. Other medications are not indicated.

A mother brings her 4-month-old son to the clinic with a quarter taped over his umbilicus, and tells the nurse the quarter is supposed to fix her child's hernia. Which explanations should the nurse provide? a. This hernia is a normal variation that resolves without treatment. b. Restrictive clothing will be adequate to help the hernia go away. c. An abdominal binder can be worn daily to reduce the protrusion. d. The quarter should be secured with an elastic bandage wrap.

This hernia is a normal variation that resolves without treatment. Rational: an umbilical hernia is a normal variation in infants that occurs due to an incomplete fusion of the abdominal musculature through the umbilical ring that usually resolves spontaneously as the child learns to walk. Other choices are ineffective and unnecessary.

A 2-year-old is bleeding from a laceration on the right lower extremity that occurred as the result of a motor vehicle collision. The nurse is selecting supplies to start an IV access. Which assessment finding is most significant in the nurse's selection of catheter size? a. Thready brachial pulse. b. Respirations of 24/minute c. Right foot cool to touch. d. Swelling at the site of injury

Thready brachial pulse.

The RN is assigned to care for four surgical clients. After receiving report, which client should the nurse see first? The client who is: a. Two days postoperative bladder surgery with continuous bladder irrigation infusing. b. One day postoperative laparoscopic cholecystectomy requesting pain medication. c. Three days postoperative colon resection receiving transfusion of packed RBCs. d. Preoperative, in buck's traction, and scheduled for hip arthroplasty within the next 12 hours.

Three days postoperative colon resection receiving transfusion of packed RBCs.

During a visit to the planned parenthood clinic, a young woman tells the nurse that she is going to discontinue taking the oral contraceptives she has taken for three years because she wants to get pregnant. History indicates that her grandfather has adult onset diabetes and that she was treated for chlamydia six months ago, which factor in this client's history poses the greatest risk for this woman's pregnancy?

Three year history of taking oral contraceptives

A client with a history of diabetes and coronary artery disease is admitted with shortness of breath, anxiety, and confusion. The client's blood pressure is 80/60 mmHg, heart rate 120 beats/minute with audible third and fourth heart sounds, and bibasilar crackles. The client's average urinary output is 5 ml/hour. Normal saline is infusing at 124 ml/hour with a secondary infusion of dopamine at mcg/kg/minute per infusion pump. With intervention should the nurse implement? a. Irrigate the indwelling urinary catheter. b. Prepare the client for external pacing. c. Obtain capillary blood glucose measurement. d. Titrate the dopamine infusion to raise the BP.

Titrate the dopamine infusion to raise the BP. Rationale: the client is experiencing cardiogenic shock and requires titration per protocol of the vasoactive secondary infusion, dopamine, to increase the blood pressure. Low hourly urine output is due to shock and does not indicate a need for catheter irrigation. Pacing is not indicated based on the client's capillary blood glucose should be monitored but is not directly indicated at this time.

The nurse is explaining the need to reduce salt intake to a client with primary hypertension. What explanation should the nurse provide? a. High salt can damage the lining of the blood vessels b. Too much salt can cause the kidneys to retain fluid c. Excessive salt can cause blood vessels to constrict d. Salt can cause information inside the blood vessels

Too much salt can cause the kidneys to retain fluid Rationale: Excessive salt intake can contribute to primary hypertension by causing renal salt retention which influence water retention that expands blood volume and pressure (ACD) are not believed to contribute to primary hypertension.

A client with hyperthyroidism is admitted to the postoperative after subtotal thyroidectomy. Which of the client's serum laboratory values requires intervention by the nurse

Total calcium 5.0 mg/dl

A client who is admitted to the intensive care unit with a right chest tube attached to a THORA-SEAL chest drainage unit becomes increasingly anxious and complain of difficulty breathing. The nurse determine the client is tachypneic with absent breath sounds in the client's right lungs fields. Which additional finding indicates that the client has developed a tension pneumothorax? a. Continuous bubbling in the water seal chamber b. Decrease bright red blood drainage c. Tachypnea and difficulty breathing d. Tracheal deviation toward the left lung.

Tracheal deviation toward the left lung. Rationale: Tracheal deviation toward the unaffected left lung with absent breath sounds over the affected right lung are classic late signs of a tension pneumothorax.

Following and gunshot wound, an adult client a hemoglobin level of 4 grams/dl (40 mmol/L SI). The nurse prepares to administer a unit of blood for an emergency transfusion. The client has AB negative blood type and the blood bank sends a unit of type A Rh negative, reporting that there is not type AB negative blood currently available. Which intervention should the nurse implement? a. Transfuse Type A negative blood until type AB negative is available. b. Recheck the client's hemoglobin, blood type and Rh factor. c. Administer normal saline solution until type AB negative is available d. Obtain additional consent for administration of type A negative blood

Transfuse Type A negative blood until type AB negative is available Rationale: those who have type AB blood are considered universal recipients using A or B blood types that is the same Rh factor. The client's hemoglobin is critically low, and the client should receive a unit of blood that is type A, which must be Rh negative blood. Other options are not indicated in this situation.

Following a gun shot wound to the abdomen, a young adult male had an emergency bowel...Multiple blood products while in the operating room. His current blood pressure is 78/52...He is being mechanically ventilated, and his oxygen saturation is 87%. His laboratory values...Grams / dl (70 mmol / L SI), platelets 20,000 / mm 3 (20 x 10 9 / L (SI units), and white blood cells. Based on these assessments findings, which intervention, should the nurse implements first? a. Transfuse packed red blood cells b. Obtain blood and sputum cultures. c. Infuse 1000 ml normal saline d. Titrate oxygen to keep o2 saturation 90%

Transfuse packed red blood cells Rational: The client is exhibiting signs of multiple organ dysfunction syndrome. Transfusion is the first intervention which provide hemoglobin to carry the oxygen to the tissues, is critical.

A client with gestational diabetes is undergoing a non-stress test (NST) at 34-week gestation... is 144 beats/minute. The client is instructed to mark the fetal monitor by pressing a button each time the baby moves. After 20 minutes, the nurse evaluates the fetal monitor strip what? a- The mother perceives and marks at least four fetal movements b- Fetal movements must be elicited with vibroacoustic stimulator c- Two FHR accelerations of 15 beats/minute x 15 seconds are recorded. d- No FHR late deceleration occur in response to fetal movement

Two FHR accelerations of 15 beats/minute x 15 seconds are recorded

A woman who takes pyridostigmine for myasthenia gravis (MG) arrives at the emergency department complaining of extreme muscle weakness. Her adult daughter tells the nurse that since yesterday her mother has been unable to smile, which assessment finding warrants immediate intervention by the nurse? a- Uncontrollable drooling b- Inability to raise voice c- Tingling of extremities d- Eyelid drooling

Uncontrollable drooling

A client admitted with an acute coronary syndrome (ACS) receives eptifibatide, a glycoprotein (GP) IIB IIIA inhibitor, which important finding places the client at greatest risk? a. Blood pressure of 100/60 b. Incontinent with blood in urine c. Unresponsive to painful stimuli d. Presence of hematemesis.

Unresponsive to painful stimuli Rationale: Eptifibatide, is an inhibitor of platelet aggregation, is administer IV for ACS, and bleeding is a significant side effect. A sudden onset of unresponsiveness may indicate intracranial bleeding, which is the life threatening finding related to bleeding. Although hypotension may indicate bleeding, it is not as significant as unresponsiveness to pain. This medication has a short half-life, so B and D are not life threatening findings.

Immediately after extubation, a client who has been mechanically ventilated is placed on a 50% non-rebreather. The client is hoarse and complaining of a sore throat. Which assessment finding should the nurse report to the healthcare provider immediately?

Upper airway stridor

A client with HIV and pulmonary coccidioidomycosis is receiving amphotericin B. which assessment finding should the nurse report to the healthcare provider? a- Urinary output of 25mL per hour b- Hemoglobin level of 10 g/dL or 100 g/L (S1) c- Hyperactive bowel sounds d- Oral temperature of 100.4 F (38 C)

Urinary output of 25mL per hour

A client who is at 36 weeks gestations is admitted with severe preclampsia. After a 6 gram loading dose of magnesium sulfate is administered, an intravenous infusion of magnesium sulfate at a rate of 2 grams/hour is initiated. Which assessment finding warrants immediate intervention by the nurse?

Urine output 20 ml/hour Rational: urinary output of less than 30 ml/hour indicates that the kidneys are being affected by the high level of magnesium, which is excreted through kidneys.

A client in septic shock has a double lumen central venous catheter with one liter of 0.9% Normal Saline Solution infusing at 1 ml/hour through one lumen and TPN infusing at 50 ml/ hr. through one port. The nurse prepared newly prescribed IV antibiotic that should take 45 mints to infuse. What intervention should the nurse implement? a. Use a secondary port of the Normal Saline solution to administer the antibiotic. b. Add the antibiotic to the TPN solution, and continue the normal saline solution. c. Stop the TPN infusion for the time needed to administer the prescribed antibiotic. d. Add the antibiotic to the Normal Saline solution and continue both infusions.

Use a secondary port of the Normal Saline solution to administer the antibiotic.

While caring for a toddler receiving oxygen (02) via face mask, the nurse observes that the child's lips and nares are dry and cracked. Which intervention should the nurse implement

Use a water soluble lubricant on affected oral and nasal mucosa

The nurse is preparing to mix two medications from two different multidose vials, A and B. In which order should these actions be implemented when drawing the solutions from the vials? (Arrange from first on top to last on the bottom)

Verify the drug and dose with the label on the vial Inject the volume of air to be aspirated from each vial Aspirate the desired volume from vial A Aspirate the desired volume from vial B

The practical nurse (PN) is assigned to work with three registered nurses (RN) who are caring for neurologically compromised clients. The client with which change in status is best to assign to the PN?

Viral meningitis whose temperature changed from 101 F to 102 F.

A child with heart failure (HF) is taking digitalis. Which signs indicates to the nurse that the child may be experiencing digitalis toxicity?

Vomiting

The nurse is assessing a female client's blood pressure because she reported feeling dizzy. The blood pressure cuff is inflated to 140 mm hg and as soon as the cuff is deflated a korotkoff sound is heard. Which intervention should the nurse implement next

Wait 1 minute and palpate the systolic pressure before auscultating again

A client is discharged with automated peritoneal dialysis (PD) to be used nightly...which instructions should the nurse include? a. Wash hands before cleaning exit site b. Keep the head of the bed flat at night c. Feel for a thrill and a distal pulse nightly d. Do not get up if fluid is left in the abdomen

Wash hands before cleaning exit site Rationale: meticulous hand hygiene is essential when performing care for a peritoneal dialysis, infections is a common complication of peritoneal dialysis.

A client who is experiencing musculoskeletal pain receives a prescription for ketorolac 15mg IM q6 hours. The medication is depended in a 39mg/ml pre-filled syringe. Which action should the nurse implement when giving the medication? a. Administer the entire pre-filled syringe deep in the dorsogluteal site. b. Use a separate syringe to remove 15mg from the pre-filled syringe and give in the back of the arm. c. Waste 0.5 ml from the pre-filled syringe and inject the medication in the ventrogluteal site. d. Call the healthcare provider to request a prescription change to match the dispensed 30mg dose.

Waste 0.5 ml from the pre-filled syringe and inject the medication in the ventrogluteal site RATIONALE: The pre-filled contain 30mg /1ml, so 0.5ml should be wasted to obtain the correct dosage of 15mg for administration in the preferred IM ventrogluteal site. The nurse is responsible for calculating and preparing the prescribed dose using the available concentration, so other options are not indicated.

In caring for a client who is receiving linezolid IV for nosocomial pneumonia, which assessment finding is most important for the nurse to report to the healthcare provider?

Watery diarrhea

An adult male who fell from a roof and fractures his left femur is admitted for surgical stabilization after having a soft cast applied in the emergency department. Which assessment finding warrants immediate intervention by the nurse?

Weak palpable distal pulses

When teaching a group of school-age children how to reduce the risk of Lyme disease which instruction should the camp nurse include a. Wash hands frequently b. Avoid drinking lake water c. Wear long sleeves and pants d. Do not share personal products

Wear long sleeves and pants Rationale: Lyme disease is it tick bone disorder and is transmitted to a child via a tick bite. Keeping the skin covered reduces the risk of being bitten by a tick. Other options are not reduce the risk for tick bites.

The nurse is conducting the initial assessment of an ill client who is from another culture.... What response should the nurse provide? a- Can you read the written instructions is English? b- "What practices do you believe will help you heal?" c- What prescriptions must be strictly followed to get well. d- You must believe that the medications will help you.

What practices do you believe will help you heal?"

After multiple attempts to stop drinking, an adult male is admitted to the medical intensive care unit (MICU) with delirium tremens. He is tachycardic, diaphoretic, restless, and disoriented. Which finding indicates a life- threatening condition?

Widening QRS complexes and flat waves

When planning care for a client with acute pancreatitis, which nursing intervention has the highest priority? a. Withhold food and fluid intake. b. Initiate IV fluid replacement. c. Administer antiemetic as needed. d. Evaluate intake and output ratio.

Withhold food and fluid intake Rational: The pathophysiologic processes in acute pancreatitis result from oral fluid and ingestion that causes secretion of pancreatic enzymes, which destroy ductal tissue and pancreatic cells, resulting in auto digestion and fibrosis of the pancreas. The main focus of the nursing care is reducing pain caused by pancreatic destruction through interventions that decrease GI activity, such as keeping the client NPO. Other choices are also important intervention but are secondary to pain management.

A client who is schedule for an elective inguinal hernia repair today in day surgery is seem eating in the waiting area. What action should be taken by the nurse who is preparing to administer the preoperative medications? a- Review the surgical consent with the client b- Explain that vomiting can occur during surgery c- Remove the food from the client d- Withhold the preoperative medication

Withhold the preoperative medication

A mother calls the nurse to report that at 0900 she administered a PO dose of digoxin to her 4-month-old infant, but at 0920 the baby vomited the medicine, what instruction should the nurse provide to this mother

Withhold this dose

An African-American man come into the hypertension screening booth at a community fair. The nurse finds that is blood pressure is 170/94 mmHg. The client tells the nurse that he has never been treated for high blood pressure. What response should the nurse make? a. Your blood pressure indicate that you have hypertension. You need to see a physician at once. b. Your blood pressure is quite high. Go to the closest emergency room for immediate treatment. c. Your blood pressure is a little high. You need to have it rechecked within one week. d. Your blood pressure is little high, but it is within the normal range for your age group.

Your blood pressure is a little high. You need to have it rechecked within one weeK

During orientation, a newly hired nurse demonstrates suctioning of a tracheostomy in a skills class, as seen in the video. After the demonstration, the supervising nurse expresses concern that the demonstrated procedure increased the client's risk for which problem?

infection

A nurse receive a shift report about a male client with Obsessive compulsive disorder (OCD). The nurse does morning rounds and reaches the client while he is repeatedly washing the top of the same table. What intervention should the nurse implement? a. Encourage the client to be calm and relax for a little while b. Assist the client to identify stimuli that precipitates the activity. c. Allow time for the behavior and then redirect the clients to other activities. d. Teach the client thought stopping techniques and ways to refocus.

allow time for the behavior and then redirect the clients to other activities

In conducting a health assessment, the nurse determines that both parents of a child with asthma smoke cigarettes. What recommendation is best to the nurse to recommend to the parents? a- avoid smoking in the house b- stop smoking immediately c- decrease the number of cigarettes smoke daily d- obtain nicotine patches to assist in smoking sensation

avoid smoking in the house

A female client with severe renal impairment is receiving enoxaparin (lovenox) 30 mg SUBQ BID. Which laboratory value due to enoxaparin should the nurse report to the healthcare provider? a. creatinine clearance 25 mL/ minute b. calcium 9 mg/dl c. hemoglobin 12 grams/dl d. partial thromboplastin time (PTT) 30 seconds

creatinine clearance 25 mL/ minute

The nurse is assessing a client's nailbeds. Witch appearance indicates further follow-up is needed for problems associated with chronic hypoxia?

clubbing

The nurse prepares to insert an oral airway by first measuring for the correct sized airway. Which picture shows the correct approach to airway size measurement?

corner of the mouth to the tip of the ear

The nurse is triaging clients in an urgent care clinic. The client with which symptoms should be referred to the health care provider immediately? a. headache, photophobia, and nuchal rigidity b. high fever, skin rash, and a productive cough c. nausea, vomiting, and poor skin turgor d. malaise, fever, and stiff, swollen joints

headache, photophobia, and nuchal rigidity Rationale: Headache, photophobia, and nuchal rigidity are classic signs of meningeal infection, so this client should immediately be referred to the health care provider. AC D do not have priority of B

In planning strategies to reduce a client's risk for complications following orthopedic surgery, the nurse recognizes which pathology as the underlying cause of osteomyelitis? a. Infectious process b. Metastatic process c. Autoimmune disorder d. Inflammatory disorder

infectious process

The nurse needs to add a medication to a liter of 5% Dextrose in Water (D5W) that is already infusing into a client. At what location should the nurse inject the medication?

medication port

An adult male is brought to the emergency department by ambulance following a motorcycle accident. He was not wearing a helmet and presents with periorbital bruising and bloody drainage from both ears. Which assessment finding warrants immediate intervention by the nurse? a. Rebound abdominal tenderness b. nausea and projectile vomit c. rib pain with deep inspiration d. diminished bilateral breath sounds

nausea and projectile vomit Rationale: Projective vomiting is indicative of increasing intracranial pressure, which can lead to ischemic brain damage or death, so this finding warrants immediate intervention. Rebound abdominal tenderness may indicate internal bleeding. Diminished breath sound may be related to pain. Rib pain with inspiration may indicate rib fracture.

A preschooler with constipation needs to increase fiber intake. Which snack suggestion should the nurse provide?

oatmeal cookies

A nurse who works in the nursery is attending the vaginal delivery of a term infant. What action should the nurse complete prior to leaving the delivery room?

place the id bands on the infant and mother

The first paddle has been placed on the chest of a client who needs defibrillation. Where should the nurse place the second paddle? (Mark the location where the second paddle should be placed on the image).

right upper chest, left midaxillary

A male client with diabetes mellitus type 2, who is taking pioglitazone PO daily, reports to the nurse the recent onset of nausea, accompanied by dark-colored urine, and a yellowish cast to his skin. What instructions should the nurse provide?

you need to seek immediate medical assistance to evaluate the cause of these symptoms"

During change of shift, the nurse reports that a male client who had abdominal surgery yesterday increasingly confused and disoriented during the night. He wandered into other clients rooms, saying that there are men in his room trying to hurt him. Because of continuing disorientation and the client's multiple attempts to get of bed, soft restrains were applied at 0400. In what order should the nurse who is receiving report implement these interventions? (Arrange from first action on top to last on the bottom) 1. Assess the client's skin and circulation for impairment related to the restrains 2. Evaluate the client's mentation to determine need to continue the restrains 3. Assign unlicensed assistive personnel to remove restrains and remain with client 4. Contact the client's surgeon and primary healthcare provider

1. Assess the client's skin and circulation for impairment related to the restrains 2. Evaluate the client's mentation to determine need to continue the restrains 3. Assign unlicensed assistive personnel to remove restrains and remain with client 4. Contact the client's surgeon and primary healthcare provide

The nurse is caring for a 4-year-old male child who becomes unresponsive as his heart rate decreases to 40 beats/minute. His blood pressure is 88/70 mmHg, and his oxygen saturation is 70% while receiving 100% oxygen by non-rebreather face mask. In what sequence, from first to last, should the nurse implement these actions? (Place the first action on top and last action on the bottom.) 1. Start chest compressions with assisted manual ventilations 2. Administer epinephrine 0.01 mg/kg intraosseous (IO) 3. Apply pads and prepare for transthoracic pacing 4. Review the possible underlying causes for bradycardia.

1. Start chest compressions with assisted manual ventilations 2. Administer epinephrine 0.01 mg/kg intraosseous (IO) 3. Apply pads and prepare for transthoracic pacing 4. Review the possible underlying causes for bradycardia Rationale: The American Heart Association guidelines recommend that the basic life support (BLS) algorithm should be initiated immediately in pediatric clients who are unresponsive or have a heart rate below 60 beats/minutes*** and exhibit signs of poor perfusion. This child is manifesting poor perfusion as evidenced by a low blood pressure and poor oxygenation, so chest compression and assisted manual ventilation should be provided first, followed by administration of drug therapy for persistent bradycardia. Preparation with pad placement for transthoracic pacing should be implemented next, followed by treatment indicated for the underlying cause of the child's bradycardia.

An adult client experiences a gasoline tank fire when riding a motorcycle and is admitted to the emergency department (ED) with full thickness burns to all surfaces of both lower extremities. What percentage of body surface area should the nurse document in the electronic medical record (EMR)?

36% Rational: according to the rule of nines, the anterior and posterior surfaces of one lower extremity is designated as 18 %of total body surface area (TBSA), so both extremities equal 36% TBSA, other options are incorrect.

The nurse is teaching a group of clients with rheumatoid arthritis about the need to modify daily activities. Which goal should the nurse emphasize? a. Protect joint function b. Improve circulation c. Control tremors d. Increase weight bearing.

A Rationale: Primary goal in the management of rheumatoid arthritis is to protect and maintain joint function.

A 56-years-old man shares with the nurse that he is having difficulty making decision about terminating life support for his wife. What is the best initial action by the nurse? a. Provide an opportunity for him to clarify his values related to the decision b. Encourage him to share memories about his life with his wife and family c. Advise him to seek several opinions before making decision d. Offer to contact the hospital chaplain or social worker to offer support.

A Rationale: When a client is faced with a decisional conflict, the nurse should first provide opportunities for the client to clarify values important in the decision. The rest may also be beneficial once the client as clarified the values that are important to him in the decision-making process.

The nurse working in the psychiatric clinic has phone messages from several clients. Which call should the nurse return first? a. A young man with schizophrenia who wants to stop taking his medication b. The mother of a child who was involved in a physical fight at school today. c. A client diagnosed with depression who is experiencing sexual dysfunction. d. A family member of a client with dementia who has been missing for five hours

A family member of a client with dementia who has been missing for five hours Rationale: safety is always the priority concern and the family member of the missing client with dementia needs assistance with contacting authorities as well as psychological support during this time.

A neonate with a congenital heart defect (CHD) is demonstrating symptoms of heart failure (HF). Which interventions should the nurse include in the infant's plan of care? a. Give O2 at 6 L/nasal canula for 3 repeated oximetry screens below 90% b. Administer diuretics via secondary infusion in the morning only c. Evaluate heart rate for effectiveness of cardio tonic medications d. Use high energy formula 30 calories/ounce at Q3 hours feeding via soft nipples e. Ensure Interrupted and frequent rest periods between procedures.

A,C,D,E Rationale: Pulse oximetry screening supports prescribed level of O2. HR provides an evaluative criterion for cardiac medications, which reduce heart rate, increase strength contractions (inotropic effects) and consequently affect systemic circulation and tissue oxygenation. Breast milk or basic formula provide 20 calories/ounce, so frequent feedings with high energy formula. D minimize fatigue is necessary.

A male client is admitted for the removal of an internal fixation that was inserted for the fracture ankle. During the admission history, he tells the nurse he recently received vancomycin (vancomycin) for a methicillin-resistant Staphylococcus aureus (MRSA) wound infection. Which action should the nurse take? (Select all that apply.) a. Collect multiple site screening culture for MRSA b. Call healthcare provider for a prescription for linezolid (Zyrovix) c. Place the client on contact transmission precautions d. Obtain sputum specimen for culture and sensitivity e. Continue to monitor for client sign of infection.

A,C,E Rationale: Until multi-site screening cultures come back negative (A), the client should be maintained on contact isolation(C) to minimize the risk for nosocomial infection. Linezolid (Zyvox), a broad spectrum anti-infecting, is not indicated, unless the client has an active skin structure infection cause by MRSA or multidrug- resistant strains (MDRSP) of Staphylococcus aureus. A sputum culture is not indicated D) based on the client's history is a wound infection.

The nurse is caring for a one week old infant who has a ventriculoperitoneal (VP) shunt that was placed 2 days after birth. Which findings are an indication of a postoperative complication? a. Poor feeding and vomiting b. Leakage of CSF from the incisional site c. Hyperactive bowel sound d. Abdominal distention e. WBC count of 10000/mm3

ABD Rationale: A, B and D are sign of postoperative complications. Shunt malfunction is most often caused by mechanical obstruction, which can result from ventricular exudate, distal end thrombosis or displacement, and/or infection. CNS infection is usually manifested by poor feeding, vomiting, elevated temperature, decreased responsiveness and seizure activity. Incisional leakage should be tested for glucose, an indication of CSF, which place the infant at risk for infection. Abdominal distention is a manifestation of peritonitis or a postoperative ileus from distal catheter placement. C is not a result of a shunt obstruction and E is a normal finding for one-week-old neonate.

While assessing a client's chest tube (CT), the nurse discovers bubbling in the water seal chamber of the chest tube collection device. The client's vital signs are: blood pressure of 80/40 mmHg, heart rate 120 beats/minutes, respiratory rate 32 breaths/minutes, oxygen saturation 88%. Which interventions should the nurse implement? a. Provide supplemental oxygen b. Auscultate bilateral lung fields c. Administer a nebulizer treatment d. Reinforce occlusive CT dressing e. Give PRN dose of pain medication

ABD Rationale: The air bubbles indicate an air leak from the lungs, the chest tube site, or the chest tube collection system. Providing oxygen improves the oxygen saturation until the leak has been resolved. Auscultating the lung fields helps to identify absent or decrease lung sound due to collapsing lung.

While monitoring a client during a seizure, which interventions should the nurse implement? (Select all that apply a. Move obstacle away from client b. Monitor physical movements c. Insert an oral padded tongue blade d. Observe for a patent airway e. Record the duration of the seizure f. Restrain extremity to avoid seizures

ABDE Rationale: Moving this away from the client helps prevent to unnecessary injurie. Observing for the pt airway alert the nurse to provide airway assistance as soon as the seizure stop D and E provide the healthcare provider with an accurate description of the seizure activities. C inserting something on the mouth can obstruct may cause further airway obstruction and is contraindicated even if the client is biting the tongue. F may cause further injury and is contraindicated.

The nurse assesses a client with new onset diarrhea. It is most important for the nurse to question the client about recent use of which type of medication? a. Antibiotics b. Anticoagulants c. Antihypertensive d. Anticholinergics

Antibiotics Rationale: Antibiotic use may be altering the normal flora in the GI tract, resulting in the onset of diarrhea, and several classes of antibiotics result in the overgrowth of Clostridium difficile, resulting in severe diarrhea.

A client is scheduled to receive an IW dose of ondansetron (Zofran) eight hours after receiving chemotherapy. The client has saline lock and is sleeping quietly without any restlessness. The nurse caring for the client is not certified in chemotherapy administration. What action should the nurse take? a. Ask a chemotherapy-certified nurse to administer the Zofran b. Administer the Zofran after flushing the saline lock with saline c. Hold the scheduled dose of Zofran until the client awakens d. Awaken the client to assess the need for administration of the Zofran.

Administer the Zofran after flushing the saline lock with saline Rationale: Zofran is an antiemetic administered before and after chemotherapy to prevent vomiting. The nurse should administer the antiemetic using the accepter technique for IV administration via saline lock. Zofran is not a chemotherapy drug and does not need to be administered by a chemotherapy- certified nurse.

A client diagnosed with calcium kidney stones has a history of gout. A new prescription for aluminum hydroxide (Amphogel) is scheduled to begin at 0730. Which client medication should the nurse bring to the healthcare provider's attention? a. Allopurinol (Zyloprim) b. Aspirin, low dose c. Furosemide (lasix) d. Enalapril (vasote)

Allopurinol (Zyloprim) Rationale: The effectiveness of allopurinol is diminished when aluminum hydroxide is used leading to an increased chance for gout flare ups. The healthcare provider should be alerted about the allopurinol interaction so any changes in medication regimen may be considered.

In early septic shock states, what is the primary cause of hypotension? a. Peripheral vasoconstriction b. Peripheral vasodilation c. Cardiac failure d. A vagal response

Peripheral vasodilation Rationale: Toxins released by bacteria in septic shock create massive peripheral vasodilation and increase microvascular permeability at the site of the bacterial invasion.

A new member joins the nursing team spreads books on the table, puts items on two chairs, and sits on a third chair. The members of the group are forced to move closer and remove their possessions from the table what action should the nurse leader take? a. Move to welcome and accommodate a new person b. Ask the new person to move belonging to accommodate others c. Tell the new person to move belongings because of limited space d. Bring in additional chairs so that all staff members can be seated

Ask the new person to move belonging to accommodate others

Which intervention should the nurse implement during the administration of vesicant chemotherapeutic agent via an IV site in the client's arm? a. Explain the temporary burning of the IV site may occur. b. Assess IV site frequently for signs of extravasation c. Apply a topical anesthetic of the infusion site for burning d. Monitor capillary refill distal to the infusion site.

Assess IV site frequently for signs of extravasation Rationale: Infiltration of a vesicant can cause severe tissue damage and necrosis, so the IV site should be assessed regularly for extravasation (B) of the chemotherapeutic agent. The client should be instructed to report any discomfort at the site (A). If pain and burning occur, the IV should be stopped and C is not indicated. Peripheral pulses, not D, provide the best assessment of perfusion distal to the infusion should the drug extravasate or infiltrate.

A client with angina pectoris is being discharge from the hospital. What instruction should the nurse plan to include in this discharge teaching? a. Engage in physical exercise immediately after eating to help decrease cholesterol levels. b. Walk briskly in cold weather to increase cardiac output c. Keep nitroglycerin in a light-colored plastic bottle and readily available. d. Avoid all isometric exercises but walk regularly.

Avoid all isometric exercises, but walk regularly Rationale: Isometric exercise can raise blood pressure for the duration of the exercise, which may be dangerous for a client with cardiovascular disease, while walking provides aerobic conditioning that improves ling, blood vessel, and muscle function. Client with angina should refrain from physical exercise for 2 hours after meals, but exercising does not decrease cholesterol levels. Cold water cause vasoconstriction that may cause chest pain. Nitroglycerin should be readily available and stored in a dark-colored glass bottle not C, to ensure freshness of the medication.

A client with a history of dementia has become increasingly confused at night and is picking at an abdominal surgical dressing and the tape securing the intravenous (IV) line. The abdominal dressing is no longer occlusive, and the IV insertion site is pink. What intervention should the nurse implement? a. Replace the IV site with a smaller gauge. b. Redress the abdominal incision c. Leave the lights on in the room at night. d. Apply soft bilateral wrist restraints.

B Rationale: The abdominal incision should be redressed using aseptic-techniques. The IV site should be assessed to ensure that it has not been dislodged and a dressing reapplied, if need it. Leaving the light on at night may interfere with the client's sleep and increase confusion. Restraints are not indicated and should only be used as a last resort to keep client from self-harm.

A client who underwent an uncomplicated gastric bypass surgery is having difficult with diet management. What dietary instruction is most important for the nurse to explain to the client? a. Chew food slowly and thoroughly before attempting to swallow b. Plan volume-controlled evenly-space meal thorough the day c. Sip fluid slowly with each meal and between meals d. Eliminate or reduce intake fatty and gas forming food

B Rationale: It is most important for the client to learn how to eat without damaging the surgical site and to keep the digestive system from dumping the food instead of digesting it. Eating volume-control and evenly-space meals thorough the day allows the client to fill full, avoid binging, and eliminate the possibility of eating too much one time. Chewing slowly and thoroughly helps prevent over eating by allowing a filling of fullness to occur. Taking sips, rather than large amounts of fluids keeps the stomach from overfilling and allow for adequate calories to be consumed. Gas forming foods and fatty foods should be avoiding decreasing risk of dumping syndrome and flatulence.

To reduce staff nurse role ambiguity, which strategy should the nurse manager implemented? a. Confirm that all the staff nurses are being assigned to equal number of clients. b. Review the staff nurse job description to ensure that it is clear, accurate, and recurrent. c. Assign each staff nurse a turn unit charge nurse on a regular, rotating basis. d. Analyze the amount of overtime needed by the nursing staff to complete assignments.

B Rationale: Role ambiguity occurs when there is inadequate explanation of job descriptions and assigned tasks, as well as the rapid technological changes that produce uncertainty and frustration. A and D may be implemented if the nurse manager is concerned about role overload, which is the inability to accomplish the tasks related to one's role. C is not related to ambiguity.

The nurse is evaluating the diet teaching of a client with hypertension. What dinner selection indicates that the client understands the dietary recommendation for hypertension? a. Tomato soup, grilled cheese sandwich, pickles, skim milk, and lemon meringue pie. b. Baked pork chop, applesauce, corn on the cob, 2% milk, and key-lime pie. c. Grilled steak, baked potato with sour cream, green beans, coffee and raisin cream pie. d. Beed stir fry, fried rice, egg drop soup, diet coke and pumpkin pie.

Baked pork chop, applesauce, corn on the cob, 2% milk, and key-lime pie Rationale: B is limited in sodium, is high in fiber, and no additional fat is added through cooking, so it is the best choice for an antihypertensive meal. A high in sodium and cholesterol, which should be avoid. C is high in fat and caffeine which can elevate the BP D is high in sodium and cholesterol and includes caffeine.

The nurse is planning care for a client who admits having suicidal thoughts. Which client behavior indicates the highest risk for the client acting on these suicidal thoughts? a. Express feelings of sadness and loneliness b. Neglects personal hygiene and has no appetite c. Lacks interest in the activity of the family and friends d. Begin to show signs of improvement in affect

Begin to show signs of improvement in affect

The mother of a child with cerebral palsy (CP) ask the nurse if her child's impaired movements will worsen as the child grows. Which response provides the best explanation? a. Brain damage with CP is not progressive but does have a variable course b. CP is one of the most common permanent physical disability in children c. Severe motor dysfunction determines the extent of successful habilitation d. Continued development of the brain lesion determines the child's outcome.

Brain damage with CP is not progressive but does have a variable course Rationale: CP is nonprogressive cerebral insult due to asphyxia, brain malformation, or toxicity, such as kernicterus. It is characterized by impair movement, posturing and may include perceptual, expressive and intellectual deficits, but the motor disabilities can vary as the child grows (A) and as interventions are implemented to prevent disuse complications.

Based on principles of asepsis, the nurse should consider which circumstance to be sterile? a. One inch- border around the edge of the sterile field set up in the operating room b. A wrapped unopened, sterile 4x4 gauze placed on a damp table top. c. An open sterile Foley catheter kit set up on a table at the nurse waist level d. Sterile syringe is placed on sterile area as the nurse riches over the sterile field.

C Rationale: A sterile package at or above the waist level is considered sterile. The edge of sterile field is contaminated which include a 1-inch border (A). A sterile object become contaminated by capillary action when sterile objects become in contact with a wet contaminated surface.

A community health nurse is concerned about the spread of communicable diseases among migrant farm workers in a rural community. What action should the nurse take to promote the success of a healthcare program designed to address this problem? a. Conduct face to face prevention education group session is Spanish b. Offer low literacy material that explain respiratory hygiene and handwashing techniques c. Establish trust with community leaders and respect cultural and family values. d. Provide public services announcements advising those who aril o seek prompt medical attention.

C Rationale: Nurses must use respectful, targeted approaches and culturally-competent strategies based on a foundation of trust to work effectively with vulnerable populations such as migrant far, worker. These individuals are often transient and fearful of deportation, so building trust is fundamental to other strategies such as client education media. This client population is not likely to be solely comprised of Hispanic individuals, so A does not target all affected individuals Band D are not likely to address the problem until trust and respect are established with this population.

The nurse performs a prescribed neurological check at the beginning of the shift on a client who was admitted to the hospital with a subarachnoid brain attack (stroke). The client's Glasgow Coma Scale (GCS) score is 9. What information is most important for the nurse to determine? a. When the client's stroke symptoms started b. If the client is oriented to time c. The client's previous GCS score. d. The client's blood pressure and respiration rate.

C Rationale: The normal GCS is 15, and it is most important for the nurse determine if this abnormal score is a sign of improvement or deterioration in the client's conditions. A is irrelevant. B is part of the GCS. The classic vital signs in late or sudden increasing ICP are Cushing's triad (widening pulse pressure, bradycardia with full, bounding pulse, and irregular respirations) Additional vital signs and trending of values are needed to evaluate the current finding(D) and C is a more sensitive, consistent evaluation

In preparing a diabetes education program, which goal should the nurse identify as the primary emphasis for a class on diabetes self-management? a. Prepare the client to independently treat their disease process b. Reduce healthcare costs related to diabetic complications c. Enable clients to become active participating in controlling the disease process d. Increase client's knowledge of the diabetic disease process and treatment options.

C Rationale: The primary goal of diabetic self- management education is to enable the client to become an active participant in the care and control of disease process, matching levels of self- management to the abilities of the individual client. The goal is to place the client in a cooperative or collaborative role with healthcare professional rather than (A)

A preeclamptic client who delivered 24h ago remains in the labor and delivery recovery room. She continues to receive magnesium sulfate at 2 grams per hour. Her total input is limited to 125 ml per hour, and her urinary output for the last hour was 850 ml. What intervention should the nurse implement? a. discontinue the magnesium sulfate immediately b. Decrease the client's iv rate to 50 ml per hour c. Continue with the plan of care for this client d. Change the client's to NPO status

C Rationale: continue with the plan. Diuresis in 24 to 48h after birth is a sign of improvement in the preeclamptic client. As relaxation of arteriolar spasms occurs, kidney perfusion increases. With improvement perfusion, fluid is drawn into the intravascular bed from the interstitial tissue and then cleared by the kidneys

The mother of the 12- month-old with cystic fibrosis reports that her child is experiencing increasing congestion despite the use of chest physical therapy (CPT) twice a day, and has also experiences a loss of appetite. What instruction should the nurse provide? a. Perform CPT after meals to increase appetite and improve food intake. b. CPT should be performed more frequently, but at least an hour before meals. c. Stop using CPT during the daytime until the child has regained an appetite. d. Perform CPT only in the morning, but increase frequency when appetite improves.

CPT should be performed more frequently, but at least an hour before meals. Rationale: CPT with inhalation therapy should be performed several times a day to loosen the secretions and move them from the peripheral airway into the central airways where they can be expectorated. CPT should be done at least one hour before meals or two hours after meals.

A child with heart failure is receiving the diuretic furosemide (Lasix) and has serum potassium level 3.0 mEq/L. Which assessment is most important for the nurse to obtain? a. Cardiac rhythm and heart rate. b. Daily intake of foods rich in potassium. c. Hourly urinary output d. Thirst ad skin turgor.

Cardiac rhythm and heart rate Rationale: Hypokalemia is a side effect of potassium-wasting diuretics, such as Lasix, and manifest as muscle weakness, hypotension, tachycardia, and cardiac dysrhythmias, so changes in the child's heart rate and cardiac rhythm should be reported to the healthcare provider. Although BCD can affect the serum potassium level, the most important finding is the effect of hypokalemia on the child's cardiac rate and rhythm.

An older male client with type 2 diabetes mellitus reports that has experiences legs pain when walking short distances, and that the pain is relieved by rest. Which client behavior indicates an understanding of healthcare teaching to promote more effective arterial circulation? a. Consistently applies TED hose before getting dressed in the morning. b. Frequently elevated legs thorough the day. c. Inspect the leg frequently for any irritation or skin breakdown d. Completely stop cigarette/ cigar smoking.

Completely stop cigarette/ cigar smoking Rationale: Stopping cigarette smoking helps to decrease vasoconstriction and improve arterial circulation to the extremity.

Following an outbreak of measles involving 5 students in an elementary school, which action is most important for the school nurse to take? a. Review the immunization records of all children in the elementary school b. Report the measles outbreak to all community health organizations c. Schedule a mobile public health vehicle to offer measles inoculations to unvaccinated children. d. Restrict unvaccinated children from attending school until measles outbreak is resolved.

Restrict unvaccinated children from attending school until measles outbreak is resolved.

Oral antibiotics are prescribed for an 18-month-old toddler with severe otitis media. An antipyrine and benzocaine-otic also prescribed for pain and inflammation. What instruction should the nurse emphasize concerning the installation of the antipyrine/ benzocaine otic solution? a. Place the dropper on the upper outer ear canal and instill the medication slowly. b. Warm the medication in the microwave for 10 seconds before instilling. c. Keep the medication refrigerated between administrations. d. Have the child lie with the ear up for one to two minutes after installation.

D

The charge nurse in a critical care unit is reviewing clients' conditions to determine who is stable enough to be transferred. Which client status report indicates readiness for transfer from the critical care unit to a medical unit? a. Pulmonary embolus with an intravenous heparin infusion and new onset hematuria b. Myocardial infarction with sinus bradycardia and multiple ectopic beats c. Adult respiratory distress syndrome with pulse oximetry of 85% saturation. d. Chronic liver failure with a hemoglobin of 10.1 and slight bilirubin elevation

D Rationale: A slight bilirubin elevation and anemia are expected finding in a stable client with chronic liver failure who should be transferred to a less-acute medical unit.

A client with a history of chronic pain requests a nonopioid analgesic. The client is alert but has difficulty describing the exact nature and location of the pain to the nurse. What action should the nurse implement next? a. Assess the client's vital signs b. Observe the client's pupils for dilation c. Document the client's drug tolerance d. Administer the analgesic as requested

D Rationale: Chronic pain may be difficult to describe, but should be treated with analgesics as indicated

A 13 years-old client with non-union of a comminuted fracture of the tibia is admitted with osteomyelitis. The healthcare provider collects home aspirate specimens for culture and sensitivity and applies a cast to the adolescent's lower leg. What action should the nurse implement next? a. Administer antiemetic agents b. Bivalve the cast for distal compromise c. Provide high- calorie, high-protein diet d. Begin parenteral antibiotic therapy

D Rationale: The standard of treatment for osteomyelitis is antibiotic therapy and immobilization. After bond and blood aspirate specimens are obtained for culture and sensitivity, the nurse should initiate parenteral antibiotics as prescribed.

An older male client is admitted with the medical diagnosis of possible cerebral vascular accident (CVA). He has facial paralysis and cannot move his left side. When entering the room, the nurse finds the client's wife tearful and trying unsuccessfully to give him a drink of water. What action should the nurse take? a. Give the wife a straw to help facilitate the client's drinking. b. Assist the wife and carefully give the client small sips of water c. Obtain a thickening powder before providing any more fluids. d. Ask the wife to stop and assess the client's swallowing reflex.

D Rationale: Until a swallowing reflex has been established, giving oral fluids can be dangerous, even life-threatening. The nurse should immediately stop the dangerous situation and assess the client. It is most important to determine if the client can swallow before giving him anything by mouth.

The nurse is preparing to administer a histamine 2-receptor antagonist to a client with peptic ulcer disease. What is the primary purpose of this drug classification? a. Neutralize hydrochloric (HCI) acid in the stomach b. Decreases the amount of HCL secretion by the parietal cells in the stomach c. Inhibit action of acetylcholine by blocking parasympathetic nerve endings. d. Destroys microorganisms causing stomach inflammation.

Decreases the amount of HCL secretion by the parietal cells in the stomach Rationale: B correctly describe the action of histamine 2 receptor antagonist in helping to prevent peptic ulcer disease.

Which client should the nurse assess frequently because of the risk for overflow incontinence? A client a. Who is bedfast, with increased serum BUN and creatinine levels b. Who is confused and frequently forgets to go to the bathroom c. With hematuria and decreasing hemoglobin and hematocrit levels d. Who has a history of frequent urinary tract infections.

Who is confused and frequently forgets to go to the bathroom Rationale: Overflow incontinence occurs when the bladder becomes overly distended, which is common in the confused client (B) who does not remember to empty his/her bladder.

An elderly male client is admitted to the mental health unit with a sudden onset of global disorientation and is continuously conversing with his mother, who died 50 years ago. The nurse reviews the multiple prescriptions he is currently taking and assesses his urine specimen, which is cloudy, dark yellow, and has foul odor. These findings suggest that his client is experiencing which condition? a. Delirium b. Depression c. Dementia d. Psychotic episode

Delirium Rationale: The client's clinical findings-polypharmia, urinary tract infection, and possible fluid imbalance are the most common causes of cognition and memory impairment, which is characteristic of delirium.

Which assessment is more important for the nurse to include in the daily plan of care for a client with a burned extremity a. Range of Motion b. Distal pulse intensity c. Extremity sensation d. Presence of exudate

Distal pulse intensity Rationale: Distal pulse intensity assesses the blood flow through the extremity and is the most important assessment because it provides information about adequate circulation to the extremity. Range of motions evaluates the possibility of long term contractures sensation. C evaluates neurological involvement, and exudate. D provides information about wound infection, but this assessment does not have the priority of determining perfusion to the extremity.

A male client's laboratory results include a platelet count of 105,000/ mm3 Based on this finding the nurse should include which action in the client's plan of care? a. Cluster care to conserve energy b. Initiate contact isolation c. Encourage him to use an electric razor d. Asses him for adventitious lung sounds

Encourage him to use an electric razor Rationale: This client is at risk for bleeding based on his platelet count (normal 150,000 to 400,000/ mm3). Safe practices, such as using an electric razor for shaving, should be encouraged to reduce the risk of bleeding.

A 12 year old client who had an appendectomy two days ago is receiving 0.9% normal saline at 50 ml/hour. The client's urine specific gravity is 1.035. What action should the nurse implement? a. Evaluate postural blood pressure measurements b. Obtain specimen for uranalysis c. Encourage popsicles and fluids of choice d. Assess bowel sounds in all quadrants

Encourage popsicles and fluids of choice Rationale: specific gravity of urine is a measurement of hydration status (normal range of 1.010 to 1.025) which is indicative of fluid volume deficit when Sp Gr increases as urine becomes more concentrated.

The nurse note a depressed female client has been more withdrawn and noncommunicative during the past two weeks. Which intervention is most important to include in the updated plan of care for this client? a. Encourage the client's family to visit more often b. Schedule a daily conference with the social worker c. Encourage the client to participate in group activities d. Engage the client in a non-threatening conversation.

Engage the client in a non-threatening conversation. Rationale: Consistent attempts to draw the client into conversations which focus on non-threatening subjects can be an effective means of eliciting a response, thereby decreasing isolation behaviors. There is not sufficient data to support the effectiveness of A as an intervention for this client. Although B may be indicated, nursing interventions can also be used to treat this client. C is too threatening to this client.

Before leaving the room of a confused client, the nurse notes that a half bow knot was used to attach the client's wrist restraints to the movable portion of the client's bed frame. What action should the nurse take before leaving the room? a. Ensure that the knot can be quickly released. b. Tie the knot with a double turn or square knot. c. Move the ties so the restraints are secured to the side rails. d. Ensure that the restraints are snug against the client's wrist.

Ensure that the knot can be quickly released.

A vacuum-assistive closure (VAC) device is being use to provide wound care for a client who has stage III pressure ulcer on a below-the- knee (BKA) residual limb. Which intervention should the nurse implement to ensure maximum effectiveness of the device? a. Empty the device every 8 hours and change the dressing daily ensure sterility b. Extended the transparent film dressing only to edge of wound to prevent tension. c. Ensure the transparent dressing has no tears that might create vacuum leaks d. Use an adhesive remover when changing the dressing to promote comfort.

Ensure the transparent dressing has no tears that might create vacuum leak Rationale: The nurse should ensure that the VAC transparent film is intact, without tears or loose edges C) because a break in the seal resulting in drying the wound and decreasing the vacuum. The vacuum-assisted closure (VAC) device uses an open sponge in the wound bed, sealed with a transparent film dressing and tube extrudes to a suction device that exert negative pressure to remove excess wound fluid, reduce the bacterial count and stimulate granulation. The VAC is changed every other day or third day, not (A) depending on the stage of wound healing and emptied when full or weekly. The transparent wound dressing should extend 3 to 5 cm beyond the wound edges, not (B) to ensure and airtight seal. Adhesive removers leave a reduce that binder transparent film adherence (D)

After receiving the first dose of penicillin, the client begins wheezing and has trouble breathing. The nurse notifies the healthcare provider immediately and received several prescriptions. Which medication prescription should the nurse administer first? a. Epinephrine Injection, USP IV b. Diphenhydramine IV c. Albuterol (Ventolin) inhaler d. Methylprednisolone IV

Epinephrine Injection, USP IV Rationale: Epinephrine should be administered immediately to open the airway and raise the blood pressure by vasoconstricting the blood vessels. All other medications should be administered after the epinephrine is given.

Two clients ring their call bells simultaneously requesting pain medication. What action should the nurse implement first? a. Prepared both client's medication and take to them at once b. Determine when each client last received pain medication. c. Evaluate both client's pain using a standardized pain scale d. Provide non-pharmacologic pain management interventions.

Evaluate both client's pain using a standardized pain scale Rationale: Before administering pain medication, each client' s level of pain should be evaluated using a standardizing scale to determine what type and how much pain medication the clients need.

In monitoring tissue perfusion in a client following an above the knee amputation (aka), which action should the nurse include in the plan of care? a. Evaluate closet proximal pulse. b. Asses skin elasticity of the stump. c. Observe for swelling around the stump. d. Note amount color of wound drainage.

Evaluate closet proximal pulse. Rationale: A primary focus of care for a client with an AKA is monitoring for signs of adequate tissue perfusion, which include evaluating skin color and ongoing assessment of pulse strength.

An unlicensed assistive personnel (UAP) reports that a client's right hand and fingers spasms when taking the blood pressure using the same arm. After confirming the presence of spams what action should the nurse take? a. Ask the UAP to take the blood pressure in the other arm b. Tell the UAP to use a different sphygmomanometer. c. Review the client's serum calcium level d. Administer PRN antianxiety medication.

Review the client's serum calcium level Rationale: Trousseau's sign is indicated by spasms in the distal portion of an extremity that is being used to measure blood pressure and is caused by hypocalcemia (normal level 9.0-10.5 mg/dl, so C should be implemented.

When development a teaching plan for a client newly diagnosed type 1 diabetes, the nurse should explain that an increase thirst is an early sing of diabetes ketoacidosis (DKA), which action should the nurse instruct the client to implement if this sign of DKA occur? a. Resume normal physical activity b. Drink electrolyte fluid replacement c. Give a dose of regular insulin per sliding scale d. Measure urinary output over 24 hours.

Give a dose of regular insulin per sliding scale Rationale: As hyperglycemia persist, ketone body become a fuel source, and the client manifest early signs of DKA that include excessive thirst, frequent urination, headache, nausea and vomiting. Which result in dehydration and loss of electrolyte. The client should determine fingerstick glucose level and self-administer a dose of regular insulin per sliding scale.

The healthcare provider prescribes acarbose (Precose), an alpha-glucosidase inhibitor, for a client with Type 2 diabetes mellitus. Which information provides the best indicator of the drug's effectiveness? a. Body max index (BMI) between 20 and 24 b. Blood pressure reading less than 120/80 mm Hg c. Hemoglobin A1C (HbA1C) reading less than 7% d. Self-reported glucose levels of 120-150 mg/dl.

Hemoglobin A1C (HbA1C) reading less than 7% Rationale: Acarbose (Precose) delays carbohydrate absorption in the GI tract and causes the blood glucose to rise slowly after a meal. The best indicator of acarbose effectiveness is a serum hemoglobin A1 no greater than 7%, an indication of glucose level over time. Acarbose has no effect on pain or blood pressure. Self-reported glucose levels of 120-150 reflect the blood sugar at the time taken and are not the best indicator of drug effectiveness.

A 60-year-old female client asks the nurse about hormones replacement therapy (HRT) as a means preventing osteoporosis. Which factor in the client's history is a possible contraindication for the use of HRT?

Her mother and sister have a history of breast cancer

The nurse is developing the plan of care for a client with pneumonia and includes the nursing diagnosis of "Ineffective airway clearance related to thick pulmonary secretions." Which intervention is most important for the nurse to include in the client's plan of care? a. Increase fluid intake to 3,000 ml/daily b. Administer O2 at 5L/mint per nasal cannula c. Maintain the client in a semi Fowler's position d. Provide frequent rest period.

Increase fluid intake to 3,000 ml/daily Rationale: The plan of care should include an increase in fluid intake (A) to liquefy and thin secretions for easier removal of thick pulmonary secretion which facilitates airway clearance. (B) should be implemented for signs of hypoxia (C) implemented to facilitate lung expansion, and (D) implemented for activity intolerance, but these interventions do not have the priority of (A)

An older adult female admitted to the intensive care unit (ICU) with a possible stroke is intubated with ventilator setting of tidal volume 600, PlO2 40%, and respiratory rate of 12 breaths/minute. The arterial blood gas (ABG) results after intubation are PH 7.31. PaCO2 60, PaO2 104, SPO2 98%, HCO3 23. To normalize the client's ABG finding, which action is required? a. Report the results to the healthcare provider. b. Increase ventilator rate. c. Administer a dose of sodium carbonate. d. Decrease the flow rate of oxygen.

Increase ventilator rate. Rationale: This client is experience respiratory acidosis. Increasing the ventilator rate depletes CO2 a, which returns the PH toward normal. Report findings is important but only after increasing ventilator rate

What is the priority nursing action when initiating morphine therapy via an intravenous patient-controlled analgesia (PCA) pump? a. Assess the client's ability to use a numeric pain scale b. Initiate the dosage lockout mechanism on the PCA pump c. Instruct the client to use the medication before the pain become severe d. Assess the abdomen for bowel sounds

Initiate the dosage lockout mechanism on the PCA pump Rationale: Morphine depress respiration, so ensuring that the client cannot overdose on the medications

If the nurse is initiating IV fluid replacement for a child who has dry, sticky mucous membranes, flushed skin, and fever of 103.6 F. Laboratory finding indicate that the child has a sodium concentration of 156 mEq/L. What physiologic mechanism contributes to this finding? a. The intravenous fluid replacement contains a hypertonic solution of sodium chloride b. Urinary and Gastrointestinal fluid loss reduce blood viscosity and stimulate thirst c. Insensible loss of body fluids contributes to the hemoconcentration of serum solutes d. Hypothalamic resetting of core body temperature causes vasodilation to reduce body heat

Insensible loss of body fluids contributes to the hemoconcentration of serum solutes Rationale: Fever causes insensible fluid loss, which contribute to fluid volume and results in hemoconcentration of sodium (serum sodium greater than 150 mEq/L). Dehydration, which is manifested by dry, sticky mucous membranes, and flushed skin, is often managed by replacing lost fluids and electrolytes with IV fluids that contain varying concentration of sodium chloride. Although other options are consistent with fluid volume deficit, the physiologic response of hypernatremia is explained by hem concentration.

An adult male client is admitted to the emergency room following an automobile collision in which he sustained a head injury. What assessment data would provide the earliest that the client is experiencing increased intracranial pressure (ICP)? a. Lethargy b. Decorticate posturing c. Fixed dilated pupil d. Clear drainage from the ear.

Lethargy Rationale: Lethargy is the earliest sign of ICP along with slowing of speech and response to verbal commands. The most important indicator of increase ICP is the client's level or responsiveness or consciousness. B and C are very late signs of ICP.

A client with a history of cirrhosis and alcoholism is admitted with severe dyspnea and ascites. Which assessment finding warrants immediate intervention by the nurse? a. Jaundice skin tone b. Muffled heart sounds c. Pitting peripheral edema d. Bilateral scleral edema

Muffled heart sounds Rationale: Muffled heart sounds may indicative fluid build-up in the pericardium and is life- threatening. The other one is sign of end stage liver disease related to alcoholism but are not immediately life- threatening.

Which problem reported by a client taking lovastatin requires the most immediate fallow up by the nurse? a. Diarrhea and flatulence b. Abdominal cramps c. Muscle pain d. Altered taste

Muscle pain Rationale: statins can cause rhabdomyolysis, a potentially fatal disease of skeletal muscle characterized by myoglobinuria and manifested with muscle pain, so this symptom should immediately be reported to the HCP.

An older adult client with heart failure (HF) develops cardiac tamponade. The client has muffled, distant, heart sounds, and is anxious and restless. After initiating oxygen therapy and IV hydration, which intervention is most important for the nurse to implement? a. Observe neck for jugular vein distention b. Notify healthcare provider to prepare for pericardiocentesis c. Asses for paradoxical blood pressure d. Monitor oxygen saturation (Sp02) via continuous pulse oximetry

Notify healthcare provider to prepare for pericardiocentesis Rationale: Cardiac tamponade is pressure on the heart that occurs when blood or fluid builds up in the space between the heart muscle (myocardium) and the outer covering sac of the heart (pericardium). In this condition, blood or fluid collects in the pericardium, the sac surrounding the heart. This prevents the heart ventricles from expanding fully. The excess pressure from the fluid prevents the heart from working properly. As a result, the body does not get enough blood.

A mother brings her 3-year-old son to the emergency room and tells the nurse the he has had an upper respiratory infection for the past two days. Assessment of the child reveals a rectal temperature of 102 F. he is drooling and becoming increasingly more restless. What action should the nurse take first? a. Put a cold cloth on his head and administer acetaminophen. b. Listen to lung sounds and place him in a mist tent. c. Notify the healthcare provider and obtain a tracheostomy tray d. Assist the child to lie down and examine his throat.

Notify the healthcare provider and obtain a tracheostomy tray Rationale: This child exhibiting signs and symptoms of epiglottitis, a bacterial infection causing acute airway obstruction, so is the immediate action to take.

An older male client arrives at the clinic complaining that his bladder always feels full. He complains of weak urine flow, frequent dribbling after voiding, and increasing nocturia with difficulty initiating his urine stream. Which action should the nurse implement? a. Obtain a urine specimen for culture and sensitivity b. Palpate the client's suprapubic area for distention c. Advise the client to maintain a voiding diary for one week d. Instruct in effective technique to cleanse the glans penis

Palpate the client's suprapubic area for distention Rationale: the client is exhibiting classic signs of an enlarge prostate gland, which restricts urine flow and cause bothersome lower urinary tract symptoms (LUTS) and urinary retention, which is characterized by the client's voiding patterns and perception of incomplete bladder emptying.

A client is admitted to the hospital after experiencing a brain attack, commonly referred to as a stroke or cerebral vascular accident (CVA). The nurse should request a referral for speech therapy if the client exhibits which finding? a. Abnormal responses for cranial nerves I and II b. Persistent coughing while drinking c. Unilateral facial drooping d. Inappropriate or exaggerated mood swings

Persistent coughing while drinking Rationale: After a stroke, clients may experience dysphagia and an impaired gag reflex that is evaluated by a speech pathology team. Coughing while drinking results from impaired swallowing and gag reflex, so a referral to a speech therapist is indicated to evaluate the coordination of oral movements associated with speech and deglutination. Cranial nerves I and II are sensory nerves for taste and sight and do not require a referral to speech pathology. Unilateral facial drooping is associated with stroke but is not a focus of rehabilitation. D sre not addressed by speech therapy.

A client with rheumatoid arthritis (RA) starts a new prescription of etanercept (Enbrel) subcutaneously once weekly. The nurse should emphasize the importance of reporting problem to the healthcare provider? a. Headache b. Joint stiffness c. Persistent fever d. Increase hunger and thirst

Persistent fever Rationale: Enbrel decrease immune and inflammatory responses, increasing the client's risk of serious infection, so the client should be instructed to report a persistent fever, or other signs of infection to the healthcare provider.

During a Woman's Health fair, which assignment is the best for the Practical Nurse (PN) who is working with a register nurse (RN) a. Encourage the woman at risk for cancer to obtain colonoscopy. b. Present a class of breast-self examination c. Prepare a woman for a bone density screening d. Explain the follow-up need it for a client with prehypertension.

Prepare a woman for a bone density screening Rationale: A bone density screening is a fast, noninvasive screening test for osteoporosis that can be explained by the PN. There is no additional preparation needed (A) required a high level of communication skill to provide teaching and address the client's fear. (B) Requires a higher level of client teaching skill than responding to one client. (D) Requires higher level of knowledge and expertise to provide needed teaching regarding this complex topic.

A client with coronary artery disease who is experiencing syncopal episodes is admitted for an electrophysiology study (EPS) and possible cardiac ablation therapy. Which intervention should the nurse delegate to the unlicensed assistive personnel (UAP)? a. Prepare the skin for procedure. b. Identify client's pulse points c. Witness consent for procedure d. Check telemetry monitoring

Prepare the skin for procedure.

The nurse is planning preoperative teaching plan of a 12-years old child who is scheduled for surgery. To help reduce the child anxiety, which action is the best for the nurse to implement? a. Give the child syringes or hospital mask to play it at home prior to hospitalization. b. Include the child in pay therapy with children who are hospitalized for similar surgery. c. Provide a family tour of the preoperative unit one week before the surgery is scheduled. d. Provide doll an equipment to re-enact feeling associated with painful procedures

Provide a family tour of the preoperative unit one week before the surgery is scheduled Rationale: School age children gain satisfaction from exploring and manipulating their environment, thinking about objectives, situations and events, and making judgments based on what they reason. A tour of the unit allows the child to see the hospital environment and reinforce explanation and conceptual thinking.

While undergoing hemodialysis, a male client suddenly complains of dizziness. He is alert and oriented, but his skin is cool and clammy. His vital signs are: heart rate 128 beats/minute, respirations 18 breaths/minute, and blood pressure 90/60. Which intervention should the nurse implement first? a. Raise the client's legs and feet b. Administer 250 ml saline bolus c. Decrease blood flow from dialyzer d. Stop the hemodialysis procedure.

Raise the client's legs and feet Rationale: To raise the client's blood pressure is the most immediate and easiest intervention for the nurse to implement. B and C should be done asap to add volume to the vascular space by ceasing to pull fluid from the client. If the blood pressure does not increase, then the procedure may be needing to be stopped. (D)

At 1615, prior to ambulating a postoperative client for the first time, the nurse reviews the client's medical record. Based on date contained in the record, what action should the nurse take before assisting the client with ambulation: a. Remove sequential compression devices. b. Apply PRN oxygen per nasal cannula. c. Administer a PRN dose of an antipyretic. d. Reinforce the surgical wound dressing.

Remove sequential compression devices. Rationale: Sequential compression devices should be removed prior to ambulation and there is no indication that this action is contraindicated. The client's oxygen saturation levels have been within normal limits for the previous four hours, so supplemental oxygen is not warranted.

The leg of a client who is receiving hospice care have become mottled in appearance. When the nurse observes the unlicensed assistive personal (UAP) place a heating pad on the mottled areas, what action should the nurse take? a. Remove the heating pads and place a soft blanket over the client's leg and feet. b. Advise the UAP to observe the client's skin while the heating pads are in place. c. Elevate the client's feet on a pillow and monitor the client's pedal pulses frequently. d. Instruct the UAP to reposition the heating pads to the sides of the legs and feet.

Remove the heating pads and place a soft blanket over the client's leg and feet.

A client who had a below the knee amputation is experiencing severe phantom limb pain (PLP) and ask the nurse if mirror therapy will make the pain stop. Which response by the nurse is likely to be most helpful? a. Research indicates that mirror therapy is effective in reducing phantom limb pain b. You can try mirror therapy, but do not expect to complete elimination of the pain c. Transcutaneous electrical nerve stimulators (TENS) have been found to be more effective d. Where did you learn about the use of mirror therapy in treating in treating phantom limb pain?

Research indicates that mirror therapy is effective in reducing phantom limb pain Rationale: pain relief associated with mirror therapy may be due to the activation of neurons in the hemisphere of the brain that is contralateral to the amputated limb when visual input reduces the activity of systems that perceive protopathic pain.

An older adult male is admitted with complications related to chronic obstructive pulmonary disease (COPD). He reports progressive dyspnea that worsens on exertion and his weakness has increased over the past month. The nurse notes that he has dependent edema in both lower legs. Based on these assessment findings, which dietary instruction should the nurse provide? a. Limit the intake of high calorie foods. b. Eat meals at the same time daily. c. Maintain a low protein diet. d. Restrict daily fluid intake.

Restrict daily fluid intake. Rationale: the client is exhibiting signs of Cor pulmonale, a complication of COPD that causes the right side of the heart to fail. Restricting fluid intake to 1000 to 2000 ml/day, eating a high-calorie diet at small frequent meals with foods that are high in protein and low in sodium can help relieve the edema and decrease workload on the right-side of the heart.

An adult client present to the clinic with large draining ulcers on both lower legs that are characteristics of Kaposi's sarcoma lesions. The client is accompanied by two family member. Which action should the nurse take? a. Ask family member to wear gloves when touching the patient b. Send family to the waiting area while the client's history is taking c. Obtain a blood sample to determine is the client is HIV positive d. Complete the head to toes assessment to identify other sign of HIV

Send family to the waiting area while the client's history is taking Rationale: To protect the client privacy, the family member should be asked to wait outside while the client's history is take. Gloves should be worn when touching the client's body fluids if the client is HIV positive and these lesion are actually Kaposi sarcoma lesion. HIV testing cannot legally be done without the client explicit permission. A further assessment can be implemented after the family left the room.

A male client receives a thrombolytic medication following a myocardial infarction. When the client has a bowel movement, what action should the nurse implement? 53- A male client receives a thrombolytic medication following a myocardial infarction. When the client has a bowel movement, what action should the nurse implement? a. Send stool sample to the lab for a guaiac test b. Observe stool for a day-colored appearance. c. Obtain specimen for culture and sensitivity analysis d. Asses for fatty yellow streaks in the client's stool.

Send stool sample to the lab for a guaiac test Rationale: Thrombolytic drugs increase the tendency for bleeding. So, guaiac (occult blood test) test of the stool should be evaluated to detect bleeding in the intestinal tract.

When assessing a 6-month old infant, the nurse determines that the anterior fontanel is bulging. In which situation would this finding be most significant? a. Crying b. Straining on stool c. Vomiting d. Sitting upright.

Sitting upright. Rationale: The anterior fontanel closes at 9 months of age and may bulge when venous return is reduced from the head, but a bulging anterior fontanel is most significant if the infant is sitting up and may indicated an increase in cerebrospinal fluid. Activities that reduce venous return from the head, such as crying, a Valsalva maneuver, vomiting or a dependent position of the head, cause a normal transient increase in intracranial pressure.

The nurse is assisting a new mother with infant feeding. Which information should the nurse provide that is most likely to result in a decrease milk supply for the mother who is breastfeeding? a. Supplemental feedings with formula b. Maternal diet high in protein c. Maternal intake of increased oral fluid d. Breastfeeding every 2 or 3 hours.

Supplemental feedings with formula Rationale: Infant sucking at the breast increases prolactin release and proceeds a feedback mechanism for the production of milk, the nurse should explain that supplemental bottle formula feeding minimizes the infant's time at the breast and decreases milk supply. B promotes milk production and healing after delivery. C support milk production. C is recommended routine for breast feeding that promote adequate milk supply.

The fire alarm goes off while the charge nurse is receiving the shift report. What action should the charge nurse implement first? a. Instruct the client's family member to stay in the visitor waiting area until further notice b. Tell the staff to keep all clients and visitors in the client rooms with the doors closed. c. Direct the nursing staff to evacuate the clients using the stairs in a calm and orderly manner. d. Call the hospital operator to determine if the is indeed a real emergency or a fire drill.

Tell the staff to keep all clients and visitors in the client rooms with the doors closed Rationale: The charge nurse should treat the alarm as an actual fire emergency and instruct all clients and visitors to stay in the clients' room with doors closed until otherwise notified. A should be anxiety producing. Visitors should remain in the rooms with the clients. C is only necessary if the location and severity of the fire make the unit unsafe for inhabitants and would only be implemented after other measures to control de fire had failed. D should not be done until after measures are taken to protect clients and visitors.

An elderly client with degenerative joint disease asks if she should use the rubber jar openers that are available. The nurse's response should be based on which information about assistive devices? a. They can contribute to increased dependency b. They decrease the risk for joint trauma c. They promote muscle strength d. They diminish range of motion ability.

They decrease the risk for joint trauma Rationale: Assistive devices of this kind are very beneficial in reducing joint trauma(B) caused by excessive twisting. These devices promote independence, rather that increasing dependency

The nurse plans to collect a 24- hour urine specimen for a creatinine clearance test. Which instruction should the nurse provide to the adult male client? a. Clearance around the meatus, discard first portion of voiding, and collect the rest in a sterile bottle b. Urinate at specific time, discard the urine, and collect all subsequent urine during the next 24 hours. c. For the next 24 hours, notify the nurse when the bladder is full, and the nurse will collect catheterized specimens. d. Urinate immediately into a urinal, and the lab will collect specimen every 6 hours, for the next 24 hours.

Urinate at specific time, discard the urine, and collect all subsequent urine during the next 24 hours. Rationale: Urinate at specific time, discard the urine, and collect all subsequent urine during the next 24 hours is the correct procedure for collecting 24-hour urine specimen. Discarding even one voided specimen invalidate the test.

When entering a client's room, the nurse discovers that the client is unresponsive and pulseless. The nurse initiate CPR and Calls for assistance. Which action should the nurse take next? a. Prepare to administer atropine 0.4 mg IVP b. Gather emergency tracheostomy equipment c. Prepare to administer lidocaine at 100 mg IVP d. Place cardiac monitor leads on the client's chest.

a. Place cardiac monitor leads on the client's chest. Rationale: Before further interventions can be done, the client's heart rhythm must be determined. This can be done by connecting the client to the monitor. A or C are not a first line drug given for any of the life threatening, pulses dysrhythmias

When providing diet teaching for a client with cholecystitis, which types of food choices the nurse recommend to the client? a. High protein b. Low fat c. Low sodium d. High carbohydrate.

low fat Rationale: A client with cholecystitis is at risk of gall stones that can be move into the biliary tract and cause pain or obstruction. Reducing dietary fat decrease stimulation of the gall bladder, so bile can be expelled, along with possible stones, into the biliary tract and small intestine.


Set pelajaran terkait

Study guide unit 2 Earth and space.

View Set

3.4.21 ICH guidelines, ACRP Key Terms for CCRC exam, CCRC Exam, ACRP CCRC EXAM PREP, CCRC Exam Prep, ACRP CCRC, *CCRC Study Set

View Set

Chap 19 Disorders of Visual Function

View Set

Oceanography Chapter 3: Mapping and Cartography

View Set